You are on page 1of 94

लेखक – अभिषेक भिवारी सूचना एवं संचार प्रौद्योभिकी

लेखक पररचय

अंजनेय अभिषेक भिवारी (BE –EC, GATE, PGDCA & BED)

लेखक विक्रां त इां स्टीट्यूट ऑफ टे क्नॉलरजी एिां मैनेजमेंट, इां दौर से BE


(EC) में स्नरतक वकयर है । इसके अवतररक्त लेखक विछले 7 िर्षों से

अध्ययन एिां अध्यरिन करयय में सांलग्न हैं । लेखक बैंवकांग और SSC िरीक्षर
के 4 इां टरव्यू में एवियर्य हुए हैं । लेखक प्रत्यक्ष और अप्रत्यक्ष रूि से

वसविल सेिरओां से जुड़े रहे हैं । लेखक के मरगयदर्यन में कई छरत्र वसविल
सेिर, बैंवकांग, व्यरिम एिां SSC िरीक्षर में सफल हुए हैं ।

भवभिक घोषणा

 इस िुस्तक में प्रकरवर्त समस्त सूचनरएँ , ज्ञरन एिां तथ्य िूरी तरह से सत्यरवित वकए गए है । वफर भी कोई
जरनकररी यर तथ्य गलत प्रकरवर्त हो तो प्रकरर्क, सांिरदक उससे वकसी व्यक्तक्त विर्ेर्ष यर सांस्थर को िहुां ची

क्षवत के वलए वजम्मेदरर नही है ।


 इस िुस्तक में छिी सरमग्री लेखक ने अिने स्ववििेक के अनुसरर मध्यप्रदे र् की सरकररी वकतरब एिां स्टे ट

गजेवटयर से ली गयी हैं अत: इस िु स्तक में छिी सरमग्री लेखक द्वररर मौवलक रूि से वलखी गई हैं , अगर

करिीररइट उल्लां घन न कर कोई मरमलर सरमने ातर हैं तो प्रकरर्क को वजम्मेदरर नही रहररयर जरएगर ।
 सभी वििरदोां कर वनिटररर इां दौर न्यरवयक क्षे त्र में होगर ।

 ©कॉपीराइट RAKSHA ACADEMIC SOLUTIONS PRIVATE LIMITED, सवााभिकार सुरभिि ।

JOIN OUR TELEGRAM CHANNEL - https://t.me/rakshaacademy


लेखक – अभिषेक भिवारी सूचना एवं संचार प्रौद्योभिकी

आिार

िरमेश्वर की अभ्यथयनर करते हुए वर्ष्य –वर्ष्यरिांद के ात्यक्तिक कल्यरण और ात्मोत्थरन के वलए ही
प्रभु/मरतर-वितर/गुरुदे ि के स्नेहरर्ीर्ष िररणतयूभनट -9 सूचना एवं संचार प्रौद्योभिकी (अब्जेक्टिव )नरमक

िुस्तक कर लेखन एिां प्रकरर्न सांभि हुा हैं । इस उिलक्ति को अहोभरग्य समझ कर अिने को धन्य समझने
िरले ावकांचन्य जन ( अवभर्षेक वतिररी) को वत्रविध ऊजरय स्वत: ही प्ररप्त होने लगती है , क्ोांवक ावकांचन्य जन

जगत को अिनर अहोभरग्य समझने से विद्वतजनोां के ागे नतमस्तक हैं ।

इस िुस्तक के वलए अिने अमूतय ि अमूल्य सुझरिोां से लेखक को अिगत करिरने िरले आदरणीय आनंद भिश्र
जी, एवं रिा अकादिी के सिस्त टीि िेम्बर के प्रवत भी ावकांचन्य विर्ेर्ष ाभरर व्यक्त करते हैं क्ोांवक ररग

– द्वे र्ष –भय से मुक्त विद्वतजनोां ने ात्मररम वनमयल हृदय से ावकांचन्य को द्वै त में विलीन कर वदयर ।

लेखक वजतनर ाभरर अिने गुरुिर/ मरगयदर्य क को करते हैं , उतनर ही धन्यिरद और ाभरर अिने िािा –

भपिा (श्री बसंि कुिार भिवारी और श्रीिभि िायत्री भिवारी ), अिने अनुज (कैवल्य अभिषेक भिवारी ) तथर
अिने वमत्रजन एिां सहिरवरयोां को भी धन्यिरद दे ते हैं , वजन्ोांने लेखक को इस करयय के वलए प्रेररत वकयर और

लेखक अिनी इस लेखन क्षमतर कर उियोग कर मध्यप्रदे र् ररज्यसेिर प्रररक्तिक एिां मुख्य िरीक्षर के अभ्यवथययोां
के वलए अिनर कुछ सहयोग प्रदरन कर सके ।

िन्यवाद/ आिार

अंजनेय अभिषेक भिवारी

JOIN OUR TELEGRAM CHANNEL - https://t.me/rakshaacademy


लेखक – अभिषेक भिवारी सूचना एवं संचार प्रौद्योभिकी

सेट – 1

1. Hard disk के अभवष्कारक ।ै। D. KA- Band


A. रे जॉनसन उत्तर – A

B. विट् ज 6. दृश्य भकरणों की आवतभि परास ।ै


C. वलक होलोनेक A. 1016-1014Hz

D. बनरर्य लेचन B. 1014-1012Hz


उत्तर – A C. 1018-1016Hz

2. िारि का प्रथि स्वदे ीी क्‍्यू टर ।ै। D. 1020-1018Hz


A. फ्लोसरल्िर उत्तर – B

B. िरम – 8000 7. भन्‍न िें से असुिेभलि ।ै


C. िरम िद्म A. गरमर वकरणें – बैंकुरल

D. वमवहर B. एक्स वकरणें – ररन्जन


उत्तर – B C. िररबैंगनी – ररचर

3. प्रॉक्सीभिटी का अथा ।ै D. दृश्य वकरणें – हर्ेल


A. Adaptive display brightness उत्तर – D

B. Phone कर Automatic on off होनर 8. इलेक्टर ॉभनक इं टरकेस की ीु कआि कब की


C. ऑटोमेवटक रोटे र्न िई

D. इनमें से कोई नही A. 26 मरचय 2009


उत्तर – B B. 15 जुलरई 2005

4. VSAT िें प्रयुक्ि टोपोलॉजी ।ै। C. 15 वसतांबर 2004


A. Star D. इनमें से कोई नही

B. Bus उत्तर – B

C. Ring 9. अंिरााष्टर ीय टे लीग्राक संघ का िुायालय क्टस्थि


D. इनमें से कोई नही ।ै

उत्तर - A A. मोनेस्कोां में


5. भन्‍न िें से भकस बड का प्रयोि िूस्थै भिक B. जेनेिर में

उपग्र। िें भकया िया C. वर्करगोां में


A. S-band D. रोम में
B. L-band उत्तर – B
C. K-band

JOIN OUR TELEGRAM CHANNEL - https://t.me/rakshaacademy


लेखक – अभिषेक भिवारी सूचना एवं संचार प्रौद्योभिकी

10. वचुाअल िेिोरी का आकार भनिार करिा ।ै D. प्रचरलक


A. जरनकररी की मरत्रर िर उत्तर – C

B. एर्रेस लरइन िर 15. सिािान एप व सारथी पोटा ल क्रिी: संबंभिि


C. र्रटरबेस िर ।ै

D. वर्स्क स्िे स िर A. चुनरि ायोग ि वदवयरां ग वयक्तक्तयोां के


उत्तर – D सर्क्तक्तकरण से

11. िारि िें सवाप्रथि टे लीकोन सेवा प्रारं ि हुई B. िांचरयत ि मवहलर सर्क्तक्तकरण से
A. 1981 C. िुवलस विभरग ि उद्योग मांत्ररलय से

B. 1881 D. इनमें से कोई नही


C. 1871 उत्तर – A

D. 1971 16. सत्यि इं कोभसस के संथापक ।ै


उत्तर – B A. ररमरदु रई
12. FDI भकस प्रकार के नेटवका के अंििाि आिा B. ररमवलांगम ररजू
।ै C. ररजू िरनिलर

A. स्टरर नेटिकय D. सबीर भरवटयर


B. ररां ग नेटिकय उत्तर – B

C. मैर् नेटिकय 17. भन्‍न िें से कौन भन:ीुल्क ई- िेल सेवा


D. बस नेटिकय प्रदािा ।ै

उत्तर – B A. हॉटमेल
13. प्रथि रोबोभटक सजारी जो भक AIIMS नई B. रे वर्फमेल

भदल्ली िें की िई, के अंििाि भकस ग्रंभथ को C. यरहू


भनकाला िया था। D. उिरोक्त सभी

A. थरयरॉइर् उत्तर – D
B. थॉयमस 18. टाटा सिू। की भ भजटल सेवाओं ।ेिु बनाया

C. एर्रीनल िया ऐप ।ै
D. इनमें से कोई नही A. दियण ऐि

उत्तर – B B. सुिर ऐि
14. ‘न्यूिेभटक’ ।ै। C. मनी मोबरइल ऐि

A. सेंसर D. सेचत ऐि
B. ऊजरय प्रितय क उत्तर – B

C. ऊजरय स्त्रोत 19. िारि भवकास िेटवे िें ीाभिल न।ी ।ै


JOIN OUR TELEGRAM CHANNEL - https://t.me/rakshaacademy
लेखक – अभिषेक भिवारी सूचना एवं संचार प्रौद्योभिकी

A. कवर्ष 24. भन्‍न िें से असुिेभलि ।ै


B. स्िरस््य A. ग्रीन है कर – नये है कर

C. प्ररथवमक वर्क्षर B. िीकर - टे वलफोन है कर


D. िररिहन C. रे र् हे ट है कर – ब्लै क है कर की समरक्तप्त हे तु

उत्तर – D D. होविटरिेस्ट – सरकरर की जरनकररी लोगोां को


20. भदीा एप भकस राज्य के िुायिंत्री ्ारा लांच दे नर

भकया िया उत्तर – D


A. गुजररत 25. इं टरनेट आिजन ।ेिु भकस सन िें उपलब्ि

B. ाां ध्रप्रदे र् हुआ


C. वबहरर A. 1989

D. उत्तरप्रदे र् B. 1994
उत्तर – B C. 1995
21. RASHMI नािक रोबोट ्ारा कौन सी िाषा D. 1996
प्रयुक्ि न।ी की जािी ।ै उत्तर – A

A. अांग्रेजी 26. IPV6 िें भकिने भबट प्रयुक्ि ।ोिे ।ै


B. वहां दी A. 32

C. भोजिुरी B. 64
D. गुजररती C. 128

उत्तर – D D. 8
22. दे ी की प्रथि रोबोभटे क वेिीाला क्टस्थि ।ै उत्तर – C

A. इां दौर 27. Jiomart भकस सन िें ीु क भकया िया


B. उज्जैन A. 2017

C. ररयसेन B. 2018
D. भोिरल C. 2019

उत्तर – B D. 2020
23. पाइरे सी ।ै उत्तर – D

A. िहचरन वछिरकर मेसेज भेजनर 28. Flipkart की स्थापना भकस सन िें की िई


B. बैंक खरते से धन वनकरलनर A. 2006

C. मूल उत्िरद की प्रवतवलिी बरजरर में बेचनर B. 2007


D. वयक्तक्त के वयक्तक्तगत जीिन में घन ुसिैर C. 2008

उत्तर – C D. 2009
JOIN OUR TELEGRAM CHANNEL - https://t.me/rakshaacademy
लेखक – अभिषेक भिवारी सूचना एवं संचार प्रौद्योभिकी

उत्तर – B 34. पेंभसल एप संबंभिि ।ै


29. जैक िां संस्थापक ।ै A. वित्त से

A. अलीबरबर के B. मवहलर वर्क्षर से


B. ई – बे के C. बरलश्रम से

C. इां वर्यरमरटय के D. बरलवर्क्षर से


D. नेटमेट के उत्तर – C-

उत्तर – A 35. ीिुन पोटा ल संबंभिि ।ै


30. आिुभनक रोबोभटक्स के भपिा ।ै A. विदे र् मांत्ररलय

A. जरजय सी र्े िोल B. HRD मांत्ररलय


B. जोसेफ कम्िे क C. ायुर्ष मांत्ररलय

C. जोसेफ एां जेलबगय D. मरनि विकरस मांत्ररलय


D. ईसरफ ावसमोि उत्तर – B
उत्तर – C 36. SWAN Network की स्थापना हुई
31. भन्‍न िें से कौन ।ययूिेनाइ रोबोट न।ी ।ै A. 2002

A. ATLAS B. 2004
B. NAO C. 2005

C. Sophia D. 2010
D. Drone उत्तर – B

उत्तर – D 37. भन्‍न िें से कौन सी सोील नेटवभकगि


32. रोबोट की प्रचालक क।लािी ।ै ्ले टकािा पीटर ि।होलल्ज ्ारा ीु क की िई

A. मरां सिेवर्यरां A. ाकुयट


B. बरां ह B. फेसबुक

C. हरां थ C. ब्लॉग
D. उिरोक्त सभी D. हॉटमेल

उत्तर – B उत्तर – C
33. DRON BHEEM भवकभसि भकया िया 38. भन्‍न िें से सिुिेभलि ।ै

A. IIT खर्गिुर द्वररर A. फेसबुक – मरकयजुकरबगय


B. IIT नरगिुर द्वररर B. हॉटमेल – सबीर भरवटयर

C. IIT करनिुर द्वररर C. सनय – ररमवलांगम ररजू


D. IIT रूढकी द्वररर D. इां फोवसस टे क्नोलॉजी – नरररयण मूवतय

उत्तर – A उत्तर – C-
JOIN OUR TELEGRAM CHANNEL - https://t.me/rakshaacademy
लेखक – अभिषेक भिवारी सूचना एवं संचार प्रौद्योभिकी

39. W3C का िठन भकया िया D. e-bay


A. 1991 उत्तर – B

B. 1992 44. Myntra का िुायालय क्टस्थि ।ै


C. 1993 A. मुांबई

D. 1994 B. वदल्ली
उत्तर – D C. अहमदरबरद

40. Apple pay से Mobile Payment ीु क D. बांगलुरू


भकया िया उत्तर – D

A. 2013 45. व। का ा भजसिें भ भजटल सूचनाएं एकभत्रि


B. 2014 र।िी ।ै।

C. 2015 A. ाई करर्य
D. 2017 B. CRM करर्य
C. वजि करर्य
उत्तर – B
D. स्मरटय करर्य
41. पररश्रि पेाटा ल भकस राज्य से संबंभिि ।ै
उत्तर – D
A. कनरय टक
46. एक े टाबेस िें कील् ।ोिी ।ै
B. ओवर्र्र
A. लेबल
C. ररजस्थरन
B. सूचनर की तरवलकर
D. उत्तर प्रदे र्
C. ररकरर््य स कर समूह
उत्तर – B
D. श्रेणी की जरनकररी
42. स्टीव वाल्िर भकस सचा इं जन के संस्थापक ।ै
उत्तर – D
A. गूगल
47. SMTP Protocol का प्रयोि भकस काया ।ेिु
B. वबांग
।ोिा ।ै
C. यरहू
A. e-mail भेजने हे तु
D. यरनेर्ैक्स
B. e-mail receive करने हे तु
उत्तर – B
C. e-mail में िररितयन हे तु
43. भन्‍न िें से कौन सा ्‍यापार िॉ ल B2C का
D. उिरोक्त सभी
उदा।रण ।ै
उत्तर – B
A. OLX
48. Twitter िें अभिकिि Character ।ोिे ।ै
B. Amazon
A. 120
C. Trip
B. 280
JOIN OUR TELEGRAM CHANNEL - https://t.me/rakshaacademy
लेखक – अभिषेक भिवारी सूचना एवं संचार प्रौद्योभिकी

C. 130 53. े जी ्‍।ील भप्रंटर प्रकार ।ै


D. 180 A. लेसर

उत्तर – B िहले 140 थे अब ितयमरन मे 280 हो B. र्ॉट मेवटक्स


गए हैं C. मैन्यु अल

49. भ भजटल क्‍्यू टर भकस दे ी ्ारा भवकभसि D. इम्िै क्ट


भकया िया उत्तर – D

A. रूस 54. WWW का पूणा कप ।ै


B. विटे न A. Web working window

C. U.S.A. B. Windo world wide


D. जरिरन C. World wide Web

उत्तर – B D. World working web


50. सवााभिक प्रयुक्ि इनपुट भ वाइस ।ै उत्तर – C
A. टर े क बॉल 55. जंक ई- िेल क।लािा ।ै
B. स्केनर A. स्िूफ

C. मरऊस B. स्िू ल
D. इनमें से कोई नही C. स्िै म

उत्तर – C D. वस्नफर क्तिप्ट


51. ऑपरे भटं ि भसस्टि M.S.Dos का भविोचन उत्तर – C

कब भकया िया 56. 8 भबटों के सिू। को क।िे ।ै


A. 1971 A. वनबल

B. 1981 B. बरइट
C. 1991 C. बर्य

D. 2001 D. वकलोबरइट
उत्तर – B उत्तर – B

52. इं टरनेट भसस्टि भन्‍न िें से भकस िकनीक 57. भन्‍न िें से क्‍्यू टर की िाषा न।ी ।ै
का प्रयोि भकया जािा ।ै A. जरिर

A. बस B. सी ++
B. स्टरर C. रोम

C. ररां ग D. िरस्कल
D. टर ी उत्तर – C

उत्तर – D
JOIN OUR TELEGRAM CHANNEL - https://t.me/rakshaacademy
लेखक – अभिषेक भिवारी सूचना एवं संचार प्रौद्योभिकी

58. ई- िेल का Full Form ।ै उत्तर – C


A. इलेक्टर ॉवनक मेल 63. भन्‍न िें से कौन िालवेयर न।ी ।ै

B. इलेक्तररकल मेल A. िरमय


C. इलेक्तररक मेल B. िरयरस

D. उिरोक्त सभी C. कुकीज


उत्तर – A D. टर ोजन हरसय

59. वचुाअल िेिोरी का आकार भनिार करिा ।ै उत्तर – C


A. र्रटरबेस िर 64. M.P. आनलाइन पोटा ल की ीु कआि भकस

B. एर्रेस लरइन िर सन िें हुई।


C. वर्स्क स्िे स िर A. 2004

D. उिरोक्त सभी िर B. 2005


उत्तर – C C. 2006
60. किाण् य स को ले जाने की प्रभक्रया क।लािी ।ै D. 2007
A. फेवचांग उत्तर – C

B. स्टोररां ग 65. नवीा ऐप संबंभिि ।ै


C. वर्कोवर्ां ग A. िररिहन से

D. एक्जीक्यूवटां ग B. िन मांत्ररलय से
उत्तर – A C. सिे ऑफ इां वर्यर से

61. AI University की स्थापना भकस दे ी ्ारा D. रे लिे से


की िई उत्तर – C

A. अमेररकर 66. उत्िि ऐप भकस िंत्रालय के अिीन कायारि


B. सांयुक्त अरब अमीररत ।ै

C. जरिरन A. मरनि विकरस मांत्ररलय


D. रूस B. ऊजरय मांत्ररलय

उत्तर – B C. विदे र् मांत्ररलय


62. भन्‍न िें से क्या एं टीवायरस के भलए प्रयुक्ि D. ायुर्ष मांत्ररलय

।ोिा ।ै उत्तर – B
A. मेलर 67. URL का Full Form ।ै

B. मैक A. Unite resource locator


C. फरयरिरल B. Universal resource locator

D. मैक्ो C. Uniform resource locator


JOIN OUR TELEGRAM CHANNEL - https://t.me/rakshaacademy
लेखक – अभिषेक भिवारी सूचना एवं संचार प्रौद्योभिकी

D. United resource locator D. सहस्त्र – टी


उत्तर – C उत्तर – A

68. वेब पोटा ल DACNET संबंभिि ।ै


A. ई- एग्रीकल्चर से

B. ई – करमसय से
C. ई – वबजनेस से

D. ई – लॉवजक्तस्टक्स से
उत्तर – A

69. 1 TB =
A. 1024 KB

B. 1024 MB
C. 1024 GB
73. िेक ए र े स का आकार ।ै
D. 1024 Bytes
A. 16 Bits
उत्तर – C
B. 32 Bits
70. TRAI की स्थापना भकस सन िें की िई
C. 48 Bits
A. 1997
D. 64 Bits
B. 1998
उत्तर – C
C. 1999
74. बुलेट टर े न भकस भसद्ांि पर आिाररि ।ै
D. 2000
A. अर्द्य चरलक वसर्द्रां त
उत्तर – A
B. अवतचरलकतर
71. िारि िें सुपर क्‍्यू टर भनिााण करने वाली
C. िरइजनर प्रभरि
प्रथि संस्था थी
D. उिरोक्त सभी
A. DRDO
उत्तर – C
B. BARC
75. इलेक्टर ॉभनक आयोि स्थाभपि भकया िया
C. C-DAC
A. 1912 में
D. NAL
B. 1973 में
उत्तर – D
C. 1974 में
72. िारि का प्रथि सुपर क्‍्यू टर था
D. 1975 में
A. फ्लोसरल्िर
उत्तर – B
B. िरम – 1001
76. ओक्टलोस्कोप के अभवष्कारक ।ै
C. अनुिम – 2002
JOIN OUR TELEGRAM CHANNEL - https://t.me/rakshaacademy
लेखक – अभिषेक भिवारी सूचना एवं संचार प्रौद्योभिकी

A. करलय मरक्सय 81. भवश्व के प।ले ्‍यवसाभयक क्‍्यू टर के


B. करलय िरउन भनिाािा ।ै

C. ली.र्ी.फररे स्ट A. एलन ट्यूररां ग


D. जरन बरर्ीन B. कोनररर् ज्यू स

उत्तर – B C. मरइकल र्े ल


77. िारिीय सावाजभनक से वा प्रीासक दू रदीा न D. स्टीि जॉब्स

की स्थापना कब हुई उत्तर – B


A. 1956 82. ऑभिकल काइबर भकस िेत्र िें प्रयुक्ि न।ी

B. 1958 ।ोिा ।ै
C. 1959 A. टर े वफक लरइट में

D. 1960 B. कवर्ष क्षेत्र में


उत्तर – C C. लेसर बीम क्षेत्र में
78. BSNL ने अपनी ब्रा बड सेवा भकस सन िें D. ऑटोमेवटक टोल बूथ में
प्रारं ि की। उत्तर – B

A. 2001 83. भन्‍न िें से सैन्य रोबोट ।ै


B. 1986 A. एइबो

C. 2005 B. क्यू रीओ


D. 1948 C. असीमोां

उत्तर – C D. वबगर्ॉग
79. ए.जे. पालराज सभिभि संबंभिि ।ै उत्तर – D

A. 1 G तकनीक से 84. भवश्व का प।ला भ।ंदी िाषी रोबोट ।ै


B. 4 G तकनीक से A. रक्ति

C. 5 G तकनीक से B. वयोमवमत्र
D. 3 G तकनीक से C. िरिोां

उत्तर – C D. चतुर
80. बेभसक प्रोग्राभिंि िाषा के अभवष्कारक ।ै उत्तर – A

A. ग्रेस हरिर 85. िैककी उदा।रण ।ै


B. जॉन केमेनी A. िरयरस कर

C. जॉन बेकस B. क्तिक हील कर


D. जेक वकल्िी C. एां वटिरयरस कर

उत्तर – B D. फोटोएवर्वटां ग सॉफ्टिेयर कर


JOIN OUR TELEGRAM CHANNEL - https://t.me/rakshaacademy
लेखक – अभिषेक भिवारी सूचना एवं संचार प्रौद्योभिकी

उत्तर – C C. एक्स तरां गे


86. भन्‍न िें से असुिेभलि ।ै D. अिरक्त तरां गे

A. समरधरन ऐि – चुनरि ायोग उत्तर – B


B. बोलो ऐि – अमरनरथ यरवत्र सुरक्षर 91. KU Band की आवतभि ।ोिी ।ै

C. दियण ऐि – र्रक बीमर हे तु A. 8-12.5 GHz


D. सरगरबरनी ऐि – बरगिरनी हे तु B. 12.5-18 GHz

उत्तर – D C. 26.5-40 GHz


87. िॉड्यूलेीन िें भसग्नल पररवभिाि ।ोिा ।ै D. 2-4 GHz

A. मेसेज वसग्नल उत्तर – B


B. केररयर वसग्नल 92. भन्‍न िें से कौन सा टर ांसपोटा र िारि िें

C. मॉड्यूलेटेर् वसग्नल प्रयुक्ि न।ी ।ोिा ।ै


D. उिरोक्त सभी A. S-Band
उत्तर – B B. C-Band
88. टे लीिेंटरी िें भकस प्रकार का िॉड्यूलेीन C. KU-Band

प्रयुक्ि ।ोिा ।ै D. KA-Band


A. ायरम उत्तर – D

B. ािवि 93. भसनकाि – 3 ।ै।


C. कलर A. भूक्तस्थर उिग्रह

D. उिरोक्त सभी B. जैविक उिग्रह


उत्तर – D C. मौसम उिग्रह

89. रे भ यों, टे लीभवजन आभद भकस प्रकार के D. वटयर ....


संचार का उदा।रण ।ै उत्तर – A

A. वसम्प्लेक्स 94. INSAT के संयुक्ि कायाक्रि िें कौन ीाभिल


B. हॉफ र्ु प्लेक्स न।ी ।ै

C. फुल र्ु प्लेक्स A. रक्षर मांत्ररलय


D. उिरोक्त सभी B. विदे र् मांत्ररलय

उत्तर – A C. अांतररक्ष विभरग


90. संचार उपग्र। िें अपभलंक ।ेिु प्रयुक्ि िंरिे D. ाल इां वर्यर रे वर्योां

।ोिी ।ै उत्तर – B
A. रे वर्योां तरां गे 95. INSAT का IST experimental satelite

B. सूक्ष्म तरां गे launce भकया िया


JOIN OUR TELEGRAM CHANNEL - https://t.me/rakshaacademy
लेखक – अभिषेक भिवारी सूचना एवं संचार प्रौद्योभिकी

A. 1980 में 100. 5G अभवष्कार भकस सन िें हुआ


B. 1981 में A. 2017

C. 1982 में B. 2018


D. 1983 में C. 2019

उत्तर – B D. 2020
96. भ ी एन्टीना अवक्टस्थि भकया जािा ।ै उत्तर – B

A. दृवि रे खर से सीधी रे खर में 101. Nokia भकस दे ी की कंपनी ।ै


B. टर रां सररसीिर के सरथ A. भररत

C. टर रां सिोांर्र के सरथ B. जरिरन


D. टर रां सवमटर के सरथ C. चीन

उत्तर – A D. वफनलैंर्
97. DTH िें पुयक्ि टर ांसपों र ।ोिा ।ै उत्तर – D
A. L-Band 102. Toggle Key के अंििाि आिी ।ै
B. X-Band A. Num lock

C. KA-Band B. Caps lock


D. KU-Band C. उिरोक्त दोनोां

उत्तर – D D. दोनोां नहीां


98. िारि िें िुफ़्त DTH सेवा भकस सनय िें प्रारं ि उत्तर – C

की िई 103. C-Band िें प्रयुक्ि आवतभि ।ोिी ।ै


A. 2002 A. 4GHz – 8GHz

B. 2003 B. 8Ghz – 12GHz


C. 2004 C. 12GHz- 15GHz

D. 2005 D. 16GHz – 32GHz


उत्तर – C उत्तर – A

99. Co-axial cable के जनक ।ै 104. प्रथि भ भजटल क्‍्यू टर भकस दे ी ्ारा
A. सैमुअल मरसय भवकभसि भकया िया

B. हे विसरइर् A. जरिरन
C. बैकुरल B. विटे न

D. मरवटय न कूिर C. चीन


उत्तर – B D. अमेररकर

उत्तर –B
JOIN OUR TELEGRAM CHANNEL - https://t.me/rakshaacademy
लेखक – अभिषेक भिवारी सूचना एवं संचार प्रौद्योभिकी

105. National E-governance plan भकस सन C. जांक्र्न


िें लाया िया D. केविलरी बॉण्र्

A. 2005 उत्तर – C
B. 2006 110. सािुदाभयक रे भ यों केंद्र संबंभिि ।ै

C. 2008 A. उत्तर भररत से


D. 2010 B. दवक्षण भररत से

उत्तर – C C. वदल्ली से
106. ज्ञानदू ि प्रोजेक्ट भकस भजले से संबंभिि ।ै D. िूिय – उत्तर क्षेत्र से

A. इां दौर उत्तर – D


B. उज्जैन 111. इलेक्टर ॉभनक भविाि की स्थापना कब हुई

C. धरर A. 1974
D. भोिरल B. 1970
उत्तर – C C. 1976
107. परिीून्य िाप पर अद्ा चालक ।ोिे ।ै D. 1977

A. चरलक उत्तर – B
B. कुचरलक 112. अभिचालकिा के भवषय िें सवाप्रथि खोज

C. कोई प्रभरि नही भकसने की


D. िहले चरलक बरद में कुचरलक A. विवलयम र्रां कली

उत्तर – B B. जॉसेफ जे.थॉमसन


108. िध्यप्रदे ी के ी।र जोभक िारिीय C. केमरवलांग ओस

इलेक्टर ॉभनक िेन्यु केक्चररं ि क्लस्टर ।ेिु D. इनमें से कोई नही


चयभनि भकया िया ।ै उत्तर – C

A. सरगर और ग्िरवलयर 113. 3D िें दृश्यिा ।े िु भकस रं ि के कॉंच चाले


B. भोिरल और जबलिुर चश्िे प्रयुक्ि ।ोिे ।ै

C. इां दौर और भोिरल A. लरल ि हरे


D. इां दौर और उज्जै न B. हरे ि नीले

उत्तर – B C. नीले ि िीले


109. P और N पदाथों के बीच बनने वाली सीिा D. लरल ि नीले

को क।ा जािा ।ै उत्तर – D


A. कूिर बॉण्र्

B. मेयर बॉण्र्
JOIN OUR TELEGRAM CHANNEL - https://t.me/rakshaacademy
लेखक – अभिषेक भिवारी सूचना एवं संचार प्रौद्योभिकी

114. भन्‍न िें से असत्य जो े का चुनाव कीभजए B. बांगलुरू


A. विवलयम ररनिेन – कैथोर् वकरणें C. वत्रचूर

B. ली.र्ी. फररे स्ट – प्रिधयक D. वदल्ली


C. जी.ार. कैरे – प्रकरर् विद् युत सेल उत्तर – C

D. विवलयम र्रकली – अर्द्य चरलक 118. B.C.S. भसद्ांि संबंभिि ।ै


उत्तर – D A. अर्द्य चरलकोां से

115. केबल टे लीभवजन नेटवका भविेयक के भवषय िें B. अवतचरलकतर से


भन्‍न िें से कौन सा कथन सत्य ।ै। C. दोनोां से

D. दोनोां से नही
i. भदसंबर 2015 िक सिी केबल ऑपरे टरों
उत्तर – B
को अपने टर ांसभिीन एनालॉि से भ भजटल
119. D.O.E.A.C.C. का नो ल भविाि कौन सा ।ै
करना था।
A. इलेक्टर ॉवनक्स विभरग
ii. इसके ि।ि केबल ऑपरे टरों को
B. सूचनर और प्रौद्योवगकी िररर्षद
पंजीकरण कराना अभनवाया ।ोिा
C. ग्रह मांत्ररलय

iii.फ्री टू एयर चेनलों का प्रसारण अभनवाया D. मवहलर और बरल विकरस विभरग


उत्तर – A
A. i एिां ii सत्य
120. Wi-Fi की आवतभि परास ।ै
B. ii एिां iii सत्य
A. 2.4GHz

C. i, ii एिां iii सत्य B. 1.62GHz


C. 4.2GHz
D. i एिां iii सत्य
D. 2.3-30MHz
उत्तर – B
उत्तर –A

116. टर ांभजस्टर का अभवष्कार भकसने भकया 121. वा।क िरं ि से िूल संदेी को अलि करने की

A. जॉन िैरीर्ीन प्रभक्रयि क।लािी ।ै

B. िरक्टर िैटेन A. मॉड्यूलेर्न

C. विवलयम र्ॉकली B. वर्मॉड्यूलेर्न

D. इन सभी ने C. प्रिधयन

उत्तर – D D. सांरचण

117. इलेक्टर ॉभनक्स सािाभजक प्रौद्योभिकी केंद्र उत्तर – B

स्थाभपि ।ै
A. मुांबई
JOIN OUR TELEGRAM CHANNEL - https://t.me/rakshaacademy
लेखक – अभिषेक भिवारी सूचना एवं संचार प्रौद्योभिकी

122. VSNL का कायाालय भन्‍न िें से भकस स्थान A. हमयन होलेररथ


पर न।ी ।ै B. चरल्सय बेिेज

A. चैन्नई C. ाइकेन – मरकय


B. कोलकरतर D. उिरोक्त तीनोां द्वररर

C. जयिुर उत्तर – B
D. मेरर 127. COBOL का भवकास भकस वैज्ञाभनक ्ारा

उत्तर – D भकया िया


123. TRAI का िुायालय भकस स्थान पर ।ै A. ग्रेस हॉिर

A. वदल्ली B. जेक वकल्िी


B. मुांबई C. जॉन केमेनी

C. कोलकरतर D. जॉन िेकस


D. बांगलुरू उत्तर – A
उत्तर – A 128. FORTRAN भकसके ्ारा भवकभसि प्रोग्रांभिंि
124. िारि िें टे भलकोन सेवा की ीु कआि भकस िाषा ।ै

स्थान से की िई A. ग्रेस हॉफर


A. वदल्ली B. जरन बेकस

B. कोलकरतर C. जॉन केमेनी


C. मुांबई D. जेक वकल्िी

D. गुर्गरां ि उत्तर – B
उत्तर – B 129. ULSI का प्रयोि क्‍्यू टर की भकस पी ी िें

125. भन्‍न िें से कौन सी भवीेषिा 3rd जनरे ीन भकया िया


की न।ी ।ै A. 3rd

A. IC Circuit कर प्रयोग हुा। B. 4th


B. Small scale Intrigation Technology कर C. 5th

प्रयोग हुा D. उिरोक्त सभी


C. Memory हे तु Megnetic Tap कर प्रयोग उत्तर – C

D. Programming हे तु उच्च स्तरीय भरर्षर कर 130. िेग्नेभटक टे प लेभपि र।िा ।ै


प्रयुक्त
A. ायरन ाक्सरइर् से
उत्तर – C
B. क्ोवमयम र्रई ाक्सरईर् से
126. भ करें स इं जन का अभवष्कार भकसके ्ारा

भकया िया C. उिरोक्त दोनोां से


JOIN OUR TELEGRAM CHANNEL - https://t.me/rakshaacademy
लेखक – अभिषेक भिवारी सूचना एवं संचार प्रौद्योभिकी

D. दोनो से नही C. इयर फोन


D. वर्जीटल कैमरर
उत्तर – C
उत्तर – D
131. िाइक्रोसाफ्ट व ा का उदा।रण न।ी ।ै
135. िारिीय सॉफ्टवेयर उद्योि का जनक भकसे
A. एक ऑिरे वटां ग वसस्टम
क।ा जािा ।ै
B. एक इनिुट वर्िरइस
A. एम.सी.कोहली
C. एक प्रोसेवसांग वर्िरइस
B. दे िरां ग मेहतर
D. एक्तिकेर्न सॉफ्टिेयर
C. नरवसांहन
उत्तर – D
D. एन.ार.नरररयणमूवतय
132. भन्‍न िें से भदए िए कथनों पर भवचार कीभजए
उत्तर – A

136. ऑभिकल काइबर का उत्पादन करने वाला
i. ॉट नेट फ्रेिवका िाइक्रोसॉफ्ट ्ारा भवकभसि
दे ी का प।ला राज्य कौन ।ै
भकया िया
A. ररजस्थरन
ii. जावा सन िाइक्रोभसस्टि ्ारा भवकभसि
B. मध्यप्रदे र्
ओपन सोसा टे क्नोलॉजी ।ै
C. उत्तरप्रदे र्

A. केिल 1 सही D. वबहरर

उत्तर – B
B. केिल 2 सही
137. CAD का पूणा कप ।ै
C. 1 और 2 दोनोां सही
A. कम्प्यूटर एलइर् वर्जरइन

D. कोई सही नही B. कम्प्यू टर एप्लीकेर्न वर्जरइन

C. कम्प्यू टर एल्गोररथम और वर्जरइन


उत्तर – C
D. कम्प्यू टर ऐर्े र् वर्जरइन
133. एक े टाबेस िें कील् ।ोिी ।ै
उत्तर – D
A. लेबल
138. BIOS उपयोि भकया जािा ।ै
B. सूचनर की तरवलकर
A. नेटिकय िर वसस्टम जरनकररी अिर्े ट करनर
C. सांबांवधत ररकरर््य स कर समूह
B. ऑिरे वटां ग वसस्टम लोर् करनर
D. जरनकररी की श्रेणी
C. यह रूवटां ग में सहरयक है
उत्तर – C
D. उिरोक्त में से कोई नही
134. भन्‍न िें से कौन सी आउटपुट युक्टति न।ी ।ै
उत्तर – B
A. र्रम िेन प्लॉटर

B. CRT मॉनीटर

JOIN OUR TELEGRAM CHANNEL - https://t.me/rakshaacademy


लेखक – अभिषेक भिवारी सूचना एवं संचार प्रौद्योभिकी

139. Ethernet िें प्रयुक्ि टोपोलॉजी ।ै। 144. साइबर नाईक क्या ।ै
A. बस टोिोलॉजी A. ररबोट के बरत करने कर तरीकर

B. ररां ग टोिोलॉजी B. रोबोट द्वररर रे वर्यो सजयरी प्रणरली


C. टर ी टोिोलॉजी C. रोबोट द्वररर अटै क

D. ई – टोिोलॉजी D. इनमें से कोई नही


उत्तर – A उत्तर – B

140. िारि के सुपर क्‍्यू टर का नाि ।ै 145. SCARA का Full Form ।ै


A. िरम A. Sustainable compliance artificial

B. IBM robot arm


C. विप्रो B. Selective sompelince artificial robot

D. HCL arm
उत्तर – A C. Selective compliance assembly robot
141. DRDO ्ारा बनाया िया रोबोट ।ै arm
A. दक्ष D. Suspended compliance articultated

B. रक्षक robot arm


C. र्क्तक्त उत्तर – C

D. विनरकर 146. The matrix, terminator, Blade runner


उत्तर – A ।ै।

142. Father of robotics ।ै A. रोबोट


A. जॉन मेक्करथथ B. सेल्फ र्ररइविांग जेट

B. जोसेफ एां जेलबगय C. सुिर कम्प्यू टर


C. िरली एां र्रसन D. AI िर ाधरररत वफल्मे

D. जॉन सरलें उत्तर – D


उत्तर – B 147. Bixby भकस कंपनी का AI Based Vertual

143. International federation of robotics Voice Assistance ।ै।


का िुायालय क।ां ।ै A. अमेजॉन

A. मोनरको B. मरइक्ोसॉफ्ट
B. केवलफोवनययर C. एप्िल

C. जरिरन D. सैमसांग
D. िेंकफटय उत्तर – D

उत्तर – D
JOIN OUR TELEGRAM CHANNEL - https://t.me/rakshaacademy
लेखक – अभिषेक भिवारी सूचना एवं संचार प्रौद्योभिकी

148. सोभकया रोबोट को भकस दे ी ने बनाया ।ै 153. रोबोट िें ीक्टति स्त्रोि के कप िें प्रयुक्ि भकया
A. जरिरन जािा ।ै

B. हरां गकरां ग A. Pb- Acid Battry


C. सउदी अरब B. Ag-cd battry

D. अमेररकर C. दोनोां
उत्तर – B D. दोनो नहीां

149. स्पैि भकस भवषय से संबंभिि ।ै उत्तर – C


A. कम्प्यूटर 154. रोबोभटक्स िें End – Effecter कौन सा अंि

B. कलर ।ोिा ।ै
C. सांगीत A. मरां सिेवर्यर

D. खेल B. बरां ह
उत्तर – A C. हरथ
150. भन्‍न िें से कौन एक पभ।ये वाले रोबोट का D. िैर
उदा।रण ।ै
उत्तर – C
A. रोमेलर
155. DRDO ्ारा बा कदी सुरंि का पिा लिाने
B. र्ररिर
।ेिु भकस रोबोट का अभवष्कार भकया िया
C. बरल्र्
A. FIO
D. होांर्र असीमोां
B. FINSAS
उत्तर – C
C. Helpmate
151. भन्‍न िें से कौन ।ययूिेनाइ रोबोट न।ी ।ै
D. REX
A. ATLAS
उत्तर – B
B. सोवफयर
156. DAKSH का भनिााण भकस संस्थान ्ारा
C. रोवबयर
भकया िया ।ै
D. र्ररिर
A. DRDO
उत्तर – D
B. C-DAC
152. कौन सा नेनोरोबोटय स का उदा।रण न।ी ।ै
C. ICAR
A. जेनिोट् स
D. PACE
B. बरयोवचप्स
उत्तर – A
C. न्यूबोट् स
D. बैक्टीररयर बेस्र्

उत्तर – A
JOIN OUR TELEGRAM CHANNEL - https://t.me/rakshaacademy
लेखक – अभिषेक भिवारी सूचना एवं संचार प्रौद्योभिकी

160. भवश्व का प।ला िानव सद्री रोबोट ।ै


A. फ्यूमूल्स

B. बरबोस – I
C. कररोमेट

D. सोवफयर
उत्तर – B

161. रोबोकाप ।ै
A. विश्ि कर िहलर िुवलस रोबोट अवधकररी

B. ज्िरलरमुखी वनयांत्रक रोबोट


C. Science fiction

157. KIROBO का भनिााण भकस दे ी ्ारा भकया D. उिरोक्त सभी


िया ।ै। उत्तर – A
A. अमेररकर 162. जेनबोटय स के भवषय िें सत्य कथन ।ै
B. जरिरन A. USA द्वररर वनवमयत मरनि सदृश्य रोबोट

C. चीन B. 2019 में विकवसत


D. भररत C. दु बई िुवलस बल में र्रवमल

उत्तर – B D. अिीकी मेंढक प्रजरवत जेलोिस लरविस के


158. रोबोभटक्स के भवषय िें असत्य कथन ।ै नरम िर िर्र

A. रोबोवटक्स र्ब्द कर प्रयोग उिन्यरस Liar में उत्तर – D


वकयर गयर है 163. MANAV ।ै

B. I औद्योवगक रोबोट UNIMATE है । A. भररत कर िलहर रोबोट


C. िरबोट – I कर वनमरय ण जोसेफ एां जेलबगय द्वररर B. जरिरन कर िहलर रोबोट

वकयर गयर C. अमेवकरर कर िहलर रोबोट


D. फेवमसेवियांस कर वनमरय ण 2008 में हुा। D. चीन कर िहलर रोबोट

उत्तर –C उत्तर – A
159. रोबोट जोभक 3600 घूिने िें सिि ।ै 164. भन्‍न िें से कौन – कौन साइबर क्राइि के

A. कोरोमेट कपों िें ीाभिल ।ै


B. फ्चूमूल्स A. इां टरप्र्न

C. बरबोट – I B. फेविकेर्न
D. रौांकी C. इां टरसेप्र्न

उत्तर – B D. मरड्यूलेर्न
JOIN OUR TELEGRAM CHANNEL - https://t.me/rakshaacademy
लेखक – अभिषेक भिवारी सूचना एवं संचार प्रौद्योभिकी

उत्तर – D C. मेक्ो
165. फ्लैि क्या ।ै D. िरलीमरवफयक

A. इां टरनेट कर प्रयोग कर लोगोां को नुकसरन उत्तर – B


िहुां चरनर 170. िालवेयर से बचाव ।ेिु प्रयुक्ि एं भटवायरस ।ै

B. इां टरनेट िर वकसी वयक्तक्त के वलए वलखे गए A. स्िरईिेयर


र्ब्द B. एर्िेयर

C. मूल उत्िरद की प्रवतवलवि को बरजरर में बेंचनर C. फॉयरिरल


D. िहचरन वछिरकर मेसेज भेजनर D. र्रयलसय

उत्तर – B उत्तर – C
166. Root kits प्रयुक्ि ।ोिा ।ै 171. काल्कन – 9 ।ै

A. वकसी वसस्टम की बेवसक जरनकररी कर ितर A. एक रॉकेट


लगरनर B. छोटी रें ज की वमर्रइल
B. वकसी वसस्टम में वनयांवत्रततर प्ररप्त करने हे तु C. मरनि रवहत लर्रकू विमरन
C. झूरे मेल दे खने हे तु D. अांतररक्ष िेधर्रलर

D. अिरां वछत मेसेज भेजने हे तु उत्तर – A


उत्तर – B 172. JUGNU ।ै

167. Ethical Hacker कौन से ।ैकर िाने जािे ।ै A. वमर्रइल


A. White hat B. एां वटनर

B. Blcak hat C. नैनो उिग्रह


C. Gray hat D. दीघन य उिग्रह

D. Cracker उत्तर – C
उत्तर – A 173. BARC क्टस्थि ।ै

168. ।ेक्टिभवस्ट प्रयुक्ि ।ोिा ।ै A. मध्यप्रदे र् में


A. िरवलवटकल एजेंर्र से हे वकांग हे तु B. गुजररत में

B. ब्लैक है कर की समरक्तप्त हे तु C. मुांबई में


C. टे वलफोन है कर D. करनिुर में

D. वबसल ब्लोअर उत्तर – C


उत्तर – A 174. पतथ्वी के पयाावरण िें वािावरणीय दबाव

169. Antiexe virus ।ै A. उच्चतर के सरथ अिररिवतयत रहती है


A. र्रयरे क्ट एक्र्न B. उच्चतर के सरथ बढतर है

B. बूटसेक्टर C. उच्चतर के सरथ कम होतर है


JOIN OUR TELEGRAM CHANNEL - https://t.me/rakshaacademy
लेखक – अभिषेक भिवारी सूचना एवं संचार प्रौद्योभिकी

D. प्रत्येक 6 वकमी िर बढतर है B. 1014-1012HZ


उत्तर – C C. 1016-1014HZ

175. भन्‍न िें से कौन सा कथन संक्रिभणका के D. इनमें से कोई नही


संबंि िें सत्य ।ै उत्तर – A

A. एक ही समुदरय की जरवतयरां उिक्तस्थत 180. िोबाइल सैटेलाइट िें कौन सी बड प्रयुक्ि


B. दो वभन्न समुदरयोां के मध्य सांक्मण क्षेत्र ।ोिी ।ै

C. दो सरमरन समुदरयोां के मध्य सांक्मण क्षेत्र A. L – Band


D. कोई नही B. S – Band

उत्तर – B C. C – Band
176. प्रसारण का भसद्ांि भकस ्‍यक्टति ्ारा भदया D. KU – Band

िया उत्तर – A
A. सैमुअल मरसय 181. 2G जनरे ीन सवाप्रथि भकस दे ी िें प्रारं ि
B. स्टीि जॉब्स हुई
C. ाइकेन A. अमेररकर

D. ग्ररहम बेल B. जरिरन


उत्तर – A C. भररत

177. टे लीकोन का अभवष्कार भकस दे ी िें भकया D. चीन


िया उत्तर – B

A. जरिरन 182. प्रौद्योभिकी भवकास पररषद का िठन हुआ


B. भररत A. 1971 में

C. अमेररकर B. 1972 में


D. वफनलैंर् C. 1973 में

उत्तर – C D. 1974 में


178. ऑक्ट्टकल काइबर के खोजकिाा ।ै उत्तर – C

A. ओवलिर हे विसरइर् 183. िारि िें पोस्टका ा का प्रारं ि भकस सन िें


B. नररां दर वसांह केम्िे नी भकया िया

C. बैकुरल A. 1863
D. इनमें से कोई नही B. 1879

उत्तर – B C. 1880
179. अवरक्ि भकरणों की आवतभि ।ै D. 1911

A. 1012-1010Hz उत्तर – B
JOIN OUR TELEGRAM CHANNEL - https://t.me/rakshaacademy
लेखक – अभिषेक भिवारी सूचना एवं संचार प्रौद्योभिकी

184. प्रथि क्‍्यू टर नेटवका था उत्तर – A


A. ईथरनेट 189. HTML का अभवष्कार भकसके ्ारा भकया

B. गीगरनेट िया
C. वजांजरनेट A. ार.टरमवलांसन

D. कैंिस एररयर नेटिकय B. वटम बनयसय ली


उत्तर – A C. वलक्लरइर्र

185. रोबोट जो भक लीभनयर रोबोट के नाि से D. इनमें से कोई नही


जाना जािा ।ै उत्तर – B

A. स्कररर रोबोट 190. Amazon व Flipkart उदा।रण ।ै


B. करटे वर्यन रोबोट A. B to B कर

C. करटे वर्यरन रोबोट B. B to C कर


D. इनमें से कोई नही C. C to C कर
उत्तर – C D. C to B कर
186. भन्‍न िें से कौन साइबर अपराि िें ीाभिल उत्तर – B

न।ी ।ोिे ।ै 191. दपाण एप संबंभिि ।ै


A. सरइबर िररफेयर A. वर्वजटल िेमेंट हे तु

B. सरइबर िुवलांग B. र्रक बीमर हे तु


C. स्िू वफांग C. बैंकोां में धोखरधर्ी हे तु

D. प्लैम D. इनमें से कोई नहीां


उत्तर – C उत्तर – B

187. फ्रीकर ।ै 192. रोबोट के ।ाथ सािान्यि: क।लािे ।ै


A. टे लीफोन है कर A. प्रितयक

B. नये है कर B. प्रचरलक
C. ब्लैक है कर C. वनयांत्रक

D. इनमें से कोई नही D. एां र्रफेक्टर


उत्तर – A उत्तर – D

188. आरोग्य सेिु एप कब लांच भकया िया 193. WiMax संबंभिि ।ै


A. 2 अप्रैल 2020 A. जैि प्रौद्योवगकी से

B. 2 अप्रैल 2021 B. अांतररक्ष प्रौद्योवगकी से


C. 3 अप्रैल 2020 C. वमसरइल प्रौद्योवगकी से

D. इनमें से कोई नही D. सांचरर प्रौद्योवगकी से


JOIN OUR TELEGRAM CHANNEL - https://t.me/rakshaacademy
लेखक – अभिषेक भिवारी सूचना एवं संचार प्रौद्योभिकी

उत्तर – D D. दोनो नही


194. वेब क्राउलर के कप िें िी जाना जािा ।ै उत्तर – C

A. वलांक र्रयरे क्टरी 199. रा ार के जनक ।ै


B. सचय ाप्टीमरइजर A. ररबटय िरटसन र्ी िरट

C. िेब स्िरइर्र B. गीगर


D. िेब मैनेजर C. M.E.हॉक

उत्तर – C D. उिरोक्त में से कोई नही


195. भनयिों का सेट क।लािा ।ै उत्तर – A

A. प्रोटोकॉल्स 200. िेसर के अभवष्कारक ।ै


B. स्टै ण्र्सय A. गीगरर

C. ार.एफ.सी.एस. B. गोरर्न
D. इनमें से कोई नही C. टरउन्स
उत्तर – A D. उिरोक्त सभी
196. फ्रें य स प्रोजेक्ट भकस राज्य िें Impliment उत्तर – D

हुआ ।ै 201. दे ी का प्रथि क्‍्यू टर सािर भजला ।ै


A. ओवर्र्र A. अनरय कुलम

B. ररजस्थरन B. विल्लुिुरम
C. केरल C. भीरू िल्लूर

D. तवमलनरर्ु D. मल्लिुरम
उत्तर – A उत्तर – D

197. भन्‍न िें से कौन सी कंपनी इं न्वें टरी िा ल के 202. क्‍्यू टर घ ी की स्पी िापी जािी ।ै
अंििाि न।ी आिी ।ै A. गीगर बरइट में

A. अलीबरबर B. वबट में


B. अमेजन C. मेगरबरइट में

C. क्तफ्लिकरटय D. सेकेण्र् में


D. स्नैिर्ील उत्तर – C

उत्तर – A
198. CAIR ्ारा भनभिाि रोबोट ।ै 203. लेसर अद्ा चालक का अभवष्कार कब हुआ

A. रोबोसेन A. 1965
B. स्नेक B. 1961

C. A और B दोनोां C. 1966
JOIN OUR TELEGRAM CHANNEL - https://t.me/rakshaacademy
लेखक – अभिषेक भिवारी सूचना एवं संचार प्रौद्योभिकी

D. 1967 उत्तर – A
उत्तर – B 206. परि ब्र्‍। भकस वषा भनभिाि भकया िया

204. कोटोग्राकी िें सोलर सेल का प्रयोि भकस A. 2003


प्रकार भकया जािा ।ै B. 2002

A. मरिक C. 1991
B. सांसुचन D. 2020

C. कांर्क्तरि उत्तर – D
D. इनमें से कोई नही 207. भन्‍न िें से कौन सा एक क्‍्यू टर ।ा ा वेयर

उत्तर – A न।ी ।ै
205. C-DAC का िुायालय क।ां क्टस्थि ।ै A. मरउस

A. िुणे B. वप्रांटर
B. वदल्ली C. मॉनीटर
C. मुांबई D. एक्से ल
D. कोलकरतर उत्तर – D

सेट – 2

1. स्माटा कोन भ वाइस के आसपास ।ोने वाले

िूविेंट को भ टे ि कर उसके भ।साब से


काया करने वाले सेंसर को क।िे ।ड

a) एक्तक्सलेरोमीटर
b) जरयरोस्कोि

c) प्रॉक्सीवमटी
d) मैग्नोमीटर

उिर - प्रॉक्सीवमटी

2. राष्ट्रीय सुपर कंप्यूटर भिीन के अं ििा ि


सिी सुपरकंप्यूटर को भकस नेटवका से
जोड़ा जाएिा b) ईथरनेट

a) ररिरीय सू चनर विज्ञरन केंद्र नेटिकय c) ररिरीय नरलेज नेटिकय


JOIN OUR TELEGRAM CHANNEL - https://t.me/rakshaacademy
लेखक – अभिषेक भिवारी सूचना एवं संचार प्रौद्योभिकी

d) र्ैक्षवणक एिां अनुसांधरन नेटिकय a) 128 कैरे रर


b) 256 कैरे रर
उिर - ररिरीय नरलेज ने टिकय
c) 102 कैरे रर
3. टर ाई के अनुसार ______ से अभिक िििा
d) 64 कैरे रर
वाले संचार िाध्यि को ब्रा बड की संज्ञा
उिर - 64 कैरे रर
दी जाएिी
a) 128 kbps 7. इं टरनेट ऑक भथंग्स के भलए नीभि जारी

b) 1 mbps करने वाला प।ला दे ी भनम्नभलक्टखि िे से


c) 512 kbps कौन सा ।ड

d) 10 mbps a) भररत
b) अमेररकर
उिर - 512 kbps
c) विटे न
4. OSI प्रोटोकाल िें भकस लेयर िे ाटा
d) रूस
पैकेट की प्राइवेसी को बनाए रखने के
उिर – भररत
भलए एक्टन्क्रिन भकया जािा ।ड –

a) टर रां सिोटय लेयर 8. वषा 2017 िे िारि सरकार ने ाटा


b) नेटिकय लेयर संरिण कानून का िसौदा पेी करने के

c) सेर्न लेयर भलए एक उच्च स्तरीय सभिभि का िठन


d) प्रेज़न्टै र्न लेयर भकया, उसके अध्यि भनम्नभलक्टखि िे से

कौन ।ड –
उिर - प्रेज़न्टै र्न लेयर
a) ररजीि कुमरर
5. भनम्नभलक्टखि आई पी ए र ै स को दे ख कर
b) एस रां गररजन
य। बिाइए भक य। भकस क्लास का आई
c) अरविन्द िरां गररयर
पी ए र ै स ।ड - 223.192.5.8
d) बी एन श्रीकष्णर
a) क्लरस –A
उिर - बी एन श्रीकष्णर
b) क्लरस –B
c) क्लरस –D 9. जब भकसी िािु को भनवााि िे उच्चिाप पर

d) क्लरस –C िरि भकया जािा ।ड िो इसके कलस्व कप


इलेिरानो का उत्सजान ।ोने लििा ।ड, य।
उिर – क्लरस – D
घटना ________ क।लािी ।ड
6. ई- िेल आई ी िे यूजर नेि की
a) ायवनक उत्सजयन
अभिकिि लंबाई भकिनी ।ो सकिी ।ड
b) इलेरररन उत्सजयन
JOIN OUR TELEGRAM CHANNEL - https://t.me/rakshaacademy
लेखक – अभिषेक भिवारी सूचना एवं संचार प्रौद्योभिकी

c) तिरयवनक उत्सजयन c) इलेरररवनक्स के सभी उिक्षेत्रोां


d) उिरोक्त मे से कोई नही मे उद्योग की अगुाई मे

अनुसांधरन एिम विकरस को


उिर - तिरयवनक उत्सजयन
बढ़रिर वदयर जरएगर
10. G-20 भीखर सम्मेलन िे AI को बढ़ावा
d) उिरोक्त सभी सत्य हैं
दे ने के भलए सरकार ने 5I पर जोर भदया ।ड,
उिर - विदे र्ी विवनमरय ण और ायरत को बढ़रिर
इसिे क्या 5I का ििलब न।ी ।ड
वदयर जरएगर
a) INCLUSIVENESS
13. 1936 िे एच ी सेवा की ीु कआि
b) INNOVATION
टे लीभवजन के िेत्र िे भकस दे ी के ्ारा
c) INFRASTRUCTURE
ीु क की िई
d) INTERNATIONAL
a) अमेररकर
COOPERATION
b) विटे न
उिर – INFRASTRUCTURE c) भररत
d) रूस
11. िारि सरकार ्ारा इलेिराभनक
उिर – विटे न
िैन्युकैक्चररं ि क्लस्टर के भलए िध्यप्रदे ी
14. इं टरनेीनल िैरीटाइि सैटेलाइट
के कौन से भजले चुने िए ।ड –
आिेनाइजेीन का िुख्यालय क।ााँ ।ड
a) भोिरल
a) न्यूयरकय
b) जबलिुर
b) ज्यूरीख
c) ग्वरवलयर
c) िरां स
d) A और b दोनोां
d) लांदन
उिर – a और b दोनोां
उिर – लांदन

15. प्रसार िारिी जो िारि का लोक सेवा

12. राष्ट्रीय इलेिराभनक नीभि 2019 के भलए प्रसारक ।ै, का िठन कब भकया िया ।ड

भनम्नभलक्टखि िे से क्या असत्य ।ड - a) 23 निांबर 1997

a) िैवश्वक स्तर िर प्रवतस्पधथ edsm b) 24 अरू बर 1997

सेरर के वलए अनुकूल मरहौल c) 23 वसतांबर 1997

बनरयर जरएगर d) 23 अरू बर 1997

b) विदे र्ी विवनमरय ण और ायरत को उिर – 23 निांबर 1997


बढ़रिर वदयर जरएगर 16. भवश्व के प।ले िीली सािुदाभयक रे भ यो

िध्यप्रदे ी के भकस भजले िे ।ड


JOIN OUR TELEGRAM CHANNEL - https://t.me/rakshaacademy
लेखक – अभिषेक भिवारी सूचना एवं संचार प्रौद्योभिकी

a) झरबुा d) र्ून्य रहती हैं


b) अलीररजिुर उिर – घन टती हैं

c) इां दौर 21. टर ायो वाल्व की खोज कब हुई ।ड


d) धरर a) 1904

उिर – अलीररजिुर b) 1884


17. TRAI के सदस्ों को ।टाने का अभिकार c) 1907

भकसके पास ।ड – d) 1990


a) ररिरिवत उिर – 1907

b) उिररिरिवत 22. भनम्नभलक्टखि p – n संभि भ वाइस न।ी ।ै


c) केंद्र सरकरर a. एलईर्ी

d) सुप्रीम कोटय कर मुख्य न्यरयधीर् b. फ़ोटोर्रयोर्


उिर – केंद्र सरकरर c. सोलर सेल
18. TRAI अभिभनयि िे सुिार भकस वषा भकया d. टर रयोर् िरल्व
िया ।ड उिर – टर रयोर् िरल्व

a) 2020 23. जेनर ायो का प्रयोि भनम्नभलक्टखि िे से


b) 2009 भकसके भलए भकया जािा ।ड

c) 2000 a) िोल्टे ज वनयांत्रक


d) 2021 b) वदिकररी

उिर – 2000 c) प्रिधयक


19. भनम्न िे से कौन से भ वाइस िापायभनक d) प्रकरर् उत्सजयक

उत्सजान पर काया न।ी करिे ।ड – उिर – िोल्टे ज वनयां त्रक


a) र्रयोर् िरल्व 24. टर ांभजस्टर का आभवष्कार करने वाले

b) टर रयोर् िरल्व ीाकले,वर ीन और बेरेलीन को नोबेल


c) अर्द्य चरलक र्रयोर् पुरस्कार कब भदया िया

d) उिरोक्त सभी a) 1948


उिर – अर्द्य चरलक र्रयोर् b) 1956

20. िाप बढ़ाने पर चालक की चालकिा प्र क्या c) 1907


प्रिाव पड़िा ।ड d) 1884

a) बढ़ती है उिर – 1956


b) घन टती है 25. लेजर भसद्ांि का प्रभिपादन भनम्नभलक्टखि

c) न बढ़ती हैं , न घन टती हैं िे से भकसने भकया ।ड –


JOIN OUR TELEGRAM CHANNEL - https://t.me/rakshaacademy
लेखक – अभिषेक भिवारी सूचना एवं संचार प्रौद्योभिकी

a) र्ैलोां d) उिरोक्त सभी


b) प्रो चरल्सय सी टरउन्स उिर – उिरोक्त सभी

c) वथयोर्र मेमेन 28. र ार के जनक भनम्नभलक्टखि िे से कौन से


d) A और b दोनोां ।ड

उिर - A और b दोनोां a) ररबटय िरटसन र्ी िरट


26. आलीयवन ने प।ला िैस लेजर भजसिे b) गीगर

।ीभलयि भनयान िैस का प्रयोि भकया c) र्ॉ र्ी के बॉस


िया,कब बनाया था d) र्ैलोां

a) 1960 उिर – ररबटय िरटसन र्ी िरट


b) 1961 29. इलेिराभनक पाका केंद्र सरकार ्ारा

c) 1962 वैभश्वक स्तर की इलेिराभनक्स


d) 1964 आवश्यकिाओं की पूभिा के भलए कब
उिर – 1960 प्रारं ि की
27. िेसर का आभवष्कार भनम्नभलक्टखि िे से a) 14 वसतांबर 1992

भकसने भकया था b) 14 वसतांबर 1991


a) गीगर c) 14 वसतांबर 1990

b) गॉर्य न d) 14 वसतांबर 1993


c) टरउन्स उिर – 14 वसतांबर 1992

30. भनम्नभलक्टखि कथनों पर भवचार कीभजये:


1. अद्ा चालकों की भवदय युि चालकिा एक कं िर और एक इन्सुलेटर के िध्य ।ोिी ।ै ।

2. भसभलकॉन, जिेभनयि, िैभलयि आसेनाइ अद्ा चालक के उदा।रण ।ड।


उपयुाति कथनों िें से कौन सा/से स।ी ।ै/।ड ?

a) केिल 1
b) केिल 2

c) 1 और 2 दोनोां
d) न तो 1और न ही 2

उिर - C व्यरख्यर: अर्द्य चरलक: z एक कांर्रर (Conductor) और इन्सुलेटर (Insulator) के बीच विद् युत
चरलकतर में मध्यितथ वक्स्टलीय रोस कर कोई भी िगय। अत: कथन 1 सही है ।

अर्द्य चरलकोां कर उियोग र्रयोर्, टर रां वज़स्टर और एकीकत सवकयट सवहत विवभन्न प्रकरर के इलेररॉवनक उिकरणोां के
वनमरय ण में वकयर जरतर है । इस तरह के उिकरणोां को उनकी कॉम्पैरनेस, विश्वसनीयतर, वबजली दक्षतर एिां कम

JOIN OUR TELEGRAM CHANNEL - https://t.me/rakshaacademy


लेखक – अभिषेक भिवारी सूचना एवं संचार प्रौद्योभिकी

लरगत के कररण व्यरिक रूि से प्रयोग में लरयर जरतर है । अर्द्य चरलकोां के कुछ उदरहरण वसवलकॉन, जमेवनयम,
गैवलयम ासेनरइर् हैं । अत: कथन 2 सही है ।

31. ओपन रे भ यो एक्सेस नेटवका (O-RAN) के संबंि िें भनम्नभलक्टखि कथनों पर भवचार कीभजये ?

1. O-RAN एक ऐसी िकनीक ।ै जो भवभिन्न प्रकार के भवक्रेिाओं के उप-घटकों का उपयोि


करके नेटवका बनाने की अनुिभि दे िी ।ै।

2. इसकी प्रिुख अविारणा रे भ यो एक्सेस नेटवका िें भवभिन्न उप-घटकों के बीच प्रोटोकॉल
और इं टरकेस को "खोलना" ।ै।

उपयुाति कथनों िें से कौन-सा/से स।ी न।ी ं ।ै/।ड ?


a) केिल 1

b) केिल 2
c) 1 और 2 दोनोां
d) न तो 1 और न ही 2

उिर - A व्यरख्यर:
 O-RAN एक तकनीक नहीां है , बक्ति मोबरइल नेटिकय ावकयटे क्चर में एक वनरां तर बदलरि है जो विवभन्न

प्रकरर के विक्ेतरओां से उिघन टकोां कर उियोग करके ने टिकय बनरने की अनुमवत दे तर है । अतः कथन 1 सही
नहीां है ।

 O-RAN एकल-विक्ेतर स्वरवमत्व ावकयटे क्चर के वििरीत मोबरइल नेटिकय को प्रसरररत करने के वलये एक
ओिन, बहु-विक्ेतर ावकयटे क्चर प्रणरली है ।

 O-RAN विवभन्न कांिवनयोां द्वररर वनवमयत हरर्य िेयर को एक सरथ करम करने में सक्षम बनरने के वलये
सॉफ्टिेयर कर उियोग करतर है ।

 ओिन RAN की प्रमुख अिधररणर RAN में विवभन्न उि-घन टकोां (रे वर्यो, हरर्य िेयर और सॉफ्टिेयर) के बीच
प्रोटोकॉल एिां इां टरफेस को "खोलनर" है । अत: कथन 2 सही है ।

32. भक्रिोकरें सी के संदिा िें भनम्नभलक्टखि कथनों पर भवचार कीभजये:


1. य। िुद्रा का एक कप ।ै जो भ भजटल या आिासी कप िें पाई जािी ।ै िथा लेन -दे न को

सुरभिि करने के भलये भक्रिोग्राकी का उपयोि करिी ।ै।


2. भक्रिोकरें सी एक केंद्रीकति पीयर-टू -पीयर नेटवका ्ारा सिभथाि ।ै भजसे ब्लॉकचेन क।ा

जािा ।ै।
उपयुाति कथनों िें से कौन-सा/से स।ी ।ै/।ड ?

a) केिल 1
JOIN OUR TELEGRAM CHANNEL - https://t.me/rakshaacademy
लेखक – अभिषेक भिवारी सूचना एवं संचार प्रौद्योभिकी

b) केिल 2
c) 1 और 2 दोनोां

d) न तो 1 और न ही 2

उिर - A व्यरख्यर:
 वक्प्टोकरें सी वजसे कभी-कभी वक्प्टो-मुद्रर यर वक्प्टो कहर जरतर है , मुद्रर कर एक रूि है जो वर्वजटल यर

ाभरसी रूि में िरई जरती है तथर लेन-दे न को सुरवक्षत करने के वलये वक्प्टोग्ररफी कर उियोग करती है ।
अत: कथन 1 सही है ।

 वक्प्टोकरें सी में मुद्रर जररी करने यर विवनयवमत करने िरलर कोई केंद्रीय प्ररवधकरण नहीां है । यह लेन-दे न को
ररकॉर्य करने और नई इकरइयोां को जररी करने के वलये विकेंद्रीकत प्रणरली कर उियोग करती है ।

 यह एक विकेंद्रीकत िीयर-टू -िीयर नेटिकय द्वररर सांचरवलत होतर है वजसे ब्लॉकचेन कहर जरतर है । अत:
कथन 2 सही नहीां है ।

33. ।ाल ।ी िें चचाा िें र।ा e-DAR पोटा ल भनम्नभलक्टखि िें से भकससे संबंभिि ।ै ?

a) मवहलर सुरक्षर
b) जेल सुधरर

c) सड़क सुरक्षर
d) ािदर ज़ोक्तखम लचीलरिन

उिर - C

 व्यरख्यर: हरल ही में सड़क, िररिहन एिां ररजमरगय मांत्ररलय (MoRTH) ने 'e-DAR' (ई-विस्तत दु घन यटनर
ररिोटय ) नरमक िोटय ल विकवसत वकयर है ।

 यह िोटय ल सड़क दु घन यटनरओां की तत्करल जरनकररी प्रदरन करतर है और दु घन यटनर के मुािज़े दरिोां को तेज़ी
से वनिटरने में मदद करतर है तरवक िीवड़त िररिररोां को ररहत वमल सके।

 िोटय ल को अन्य सरकररी िोटय लोां से जोड़र जरएगर वजससे िरहन यर र्ररइविांग लरइसें स सांबांधी वििरण तथर
िरहनोां के िांजीकरण के बररे में जरनकररी प्ररप्त की जरएगी।

 इसके अांतगयत दु घन यटनर हॉटस्पॉट की भी िहचरन की जरएगी तरवक इन हॉटस्पॉट िर दु घन यटनरओां से बचने हे तु
समरधरन प्ररप्त वकयर जर सके। अत: विकल्प C सही हैं

JOIN OUR TELEGRAM CHANNEL - https://t.me/rakshaacademy


लेखक – अभिषेक भिवारी सूचना एवं संचार प्रौद्योभिकी

34. ‘भनयर कील्ड कम्युभनकेीन टे क्नोलॉजी’ के संदिा िें भनम्नभलक्टखि कथनों पर भवचार कीभजये:
1. य। भवदय यु ि चुं बकीय रे भ यो िेत्रों के िाध्यि से े टा प्रसाररि करिा ।ै ।

2. भनयर कील्ड कम्युभनकेीन भ वाइस िें े टा टर ांसभिीन केवल िौभिक स्पीा से ।ोिा ।ै।
उपयुाति कथनों िें से कौन-सा/से स।ी ।ै/।ड ?

a) केिल 1
b) केिल 2

c) 1 और 2 दोनोां
d) न तो 1 और न ही 2

उिर - A
व्यरख्यर: वनयर फील्ड कम्युवनकेर्न टे क्नोलॉजी (NFC) :

 NFC एक छोटी दू री की िरयरलेस कनेक्तरविटी तकनीक है जो एनएफसी-सक्षम उिकरणोां (NFC-


enabled Devices) को एक-दू सरे के सरथ सांिरद करने तथर स्पर्य के मरध्यम से जल्दी और ासरनी से
जरनकररी स्थरनरां तररत करने की अनुमवत दे ती है - चरहे वबलोां कर भुगतरन करनर हो, वबज़नेस करर्य कर
ादरन-प्रदरन करनर हो, कूिन र्रउनलोर् करनर हो यर कोई दस्तरिेज़ सरझर करनर हो।

 NFC दो उिकरणोां के बीच सां चरर को सक्षम करने के वलये विद् युत चुांबकीय रे वर्यो क्षेत्रोां के मरध्यम से र्े टर
प्रसरररत करतर है । इसके वलये दोनोां उिकरणोां में एनएफसी वचप्स होनर ािश्यक है क्ोांवक लेन-दे न एक

वनकटतम दू री िर होतर है । अतः कथन 1 सही है ।


 र्े टर टर रां सफर के वलये एनएफसी-सक्षम वर्िरइस कर एक-दू सरे के सरथ स्पर्य कररनर यर एक-दू सरे को कुछ

सेंटीमीटर की दू री िर रखनर ज़रूरी होतर है । अतः कथन 2 सही है । z िर्षय 2004 में उिभोक्तर इलेररॉवनक्स
कांिवनयोां- नोवकयर, वफवलप्स और सोनी ने वमलकर एनएफसी फोरम कर गरन वकयर, वजसने र्क्तक्तर्रली

नए उिभोक्तर-सांचरवलत उत्परदोां को बनरने के वलये एनएफसी प्रौद्योवगकी हे तु रूिरे खर तैयरर की।


35. ब्रॉ कास्ट सेवा पोटा ल के संदिा िें भनम्नभलक्टखि कथनों पर भवचार कीभजये:

1. य। केवल टीवी चैनल्स और सािुदाभयक रे भ यो चैनल्स को ।ी अपनी सेवाएाँ प्रदान करे िा।
2. य। आवेदनों के टना-अराउं सिय को कि करे िा और साथ ।ी आवेदकों को उनकी प्रिभि को

टर ै क करने िें िदद करे िा। उपयुाति कथनों िें से कौन-सा/से स।ी ।ै/।ड ?
a) केिल 1

b) केिल 2
c) 1 और 2 दोनोां

d) न तो 1 और न ही 2

उिर - B
JOIN OUR TELEGRAM CHANNEL - https://t.me/rakshaacademy
लेखक – अभिषेक भिवारी सूचना एवं संचार प्रौद्योभिकी

व्यरख्यर: िॉर्करस्ट सेिर िोटय ल:


 यह एक 360 वर्ग्री वर्वजटल सल्यूर्न (360 Degree Digital Solution) प्रस्तुत करतर है जो वहतधररकोां

को अनुमवत लेने, िांजीकरण हे तु ािेदन करने , ािेदनोां िर नज़र रखने , र्ुि की गणनर करने तथर
भुगतरन करने की सुविधर प्रदरन करे गर।

 यह िोटय ल सभी वहतधररकोां को वनजी उिग्रह टीिी चैनल्स, टे लीिोटय ऑिरे टसय , मल्टी-सविय सेज़ ऑिरे टसय (
Multi-Services Operators- MSOs), सरमुदरवयक और वनजी रे वर्यो चैनल्स ावद के वलये वर्वजटल

इां वर्यर के व्यरिक प्रयरसोां के तहत अिनी सेिरएँ प्रदरन करे गर। अतः कथन 1 सही नहीां है । ‹ इस िोटय ल के
मरध्यम से 900 सैटेलरइट टीिी चैनल्स, 70 टे लीिोटय ऑिरे टसय , 1,700 एमएसओ, 350 सरमुदरवयक रे वर्यो

स्टे र्न और 380 वनजी एफएम चैनल्स के लरभरक्तित होने की सांभरिनर है ।


 प्रसररण सांबांधी गवतविवधयोां के वलये सूचनर एिां प्रसररण मांत्ररलय द्वररर जररी विवभन्न िांजीकरणोां, अनुमवतयोां

और लरइसेंस हे तु ािेदन करने के वलये वहतधररकोां तथर ािेदकोां को एकल वबांदु (एक की स्थरन िर)
सुविधर प्रदरन करनर।
 यह ािेदनोां के टनय -अररउां र् समय को कम करे गर और सरथ ही ािेदकोां को उनकी प्रगवत को टर ै क करने
में मदद करे गर। अतः कथन 2 सही है ।

36. ‘I-STEM’ पोटा ल के संदिा िें भनम्नभलक्टखि कथनों पर भवचार कीभजये:

1. य। पोटा ल ीोिकिााओ ं को उपकरणों के उपयोि के भलये लॉट िक पहुाँचने और पररणािों के


भववरण, जैसे- पेटेंट, प्रकाीन और प्रौद्योभिभकयों को साझा करने की सुभविा प्रदान करिा ।ै।

2. पोटा ल को प्रिानिंत्री भवज्ञान, प्रौद्योभिकी एवं नवाचार सला।कार पररषद (PM-STIAC) भिीन के
ित्त्वाविान िें लॉन्च भकया िया ।ै। उपयुाति कथनों िें से कौन-सा/से स।ी ।ै/।ड ?

a) केिल 1
b) केिल 2

c) 1 और 2 दोनोां
d) न तो 1 और न ही 2

उिर – C

व्यरख्यर
 I-STEM, अनुसांधरन और विकरस (R&D) सु विधरओां को सरझर करने के वलये एक ररिरीय िे ब िोटय ल है । ‹

िोटय ल र्ोधकिरय ओां को उिकरणोां के उियोग के वलये लॉटॉट तक िहुँ चने के सरथ-सरथ िररणरमोां के वििरण,
जैसे- िेटेंट, प्रकरर्न और प्रौद्योवगवकयोां को सरझर करने की सुविधर प्रदरन करतर है । अतः कथन 1 सही है ।

JOIN OUR TELEGRAM CHANNEL - https://t.me/rakshaacademy


लेखक – अभिषेक भिवारी सूचना एवं संचार प्रौद्योभिकी

 जुलरई 2021 में I-STEM िररयोजनर को िर्षय 2026 तक िरँ च सरल के वलये विस्तरर वदयर गयर थर और इसने
अवतररक्त सुविधरओां के सरथ अिने दू सरे चरण में प्रिे र् वकयर।

 इसे जनिरी 2020 में लॉन्च वकयर गयर थर और यह ‘प्रधरनमांत्री विज्ञरन, प्रौद्योवगकी एिां निरचरर सलरहकरर
िररर्षद’ (PM-STIAC) वमर्न’ के तत्त्वरिधरन में भररत सरकरर के प्रधरन िैज्ञरवनक सलरहकरर करयरय लय की

एक प्रमुख िहल है । अतः कथन 2 सही है ।

37. रे भ यो फ्रीक्वेंसी आइ ें भटभककेीन (RFID) के संदिा िें भनम्नभलक्टखि कथनों पर भवचार कीभजये:
1. RFID एक प्रकार की सभक्रय वायरलेस िकनीक ।ै जो भकसी वस्तु या व्यक्टति की टर ै भकंि या भिलान

की अनुिभि दे िी ।ै।
2. री र ्ारा इन्फ्रारे िरं िों को छोड़ भदया जािा ।ै िथा RFID टै ि ्ारा भसग्नल को वापस प्राप्त

भकया जािा ।ै।


उपयुाति कथनों िें से कौन-सा/से स।ी ।ै/।ड ?
a) केिल 1
b) केिल 2

c) 1 और 2 दोनोां
d) न तो 1 और न ही 2

उिर - D
व्यरख्यर: िीिेंसी ाइर्ें वटवफकेर्न (RFID):

 RFID एक प्रकरर की वनक्तिय िरयरलेस तकनीक है जो वकसी िस्तु यर व्यक्तक्त की टर ै वकांग और मैवचांग की
अनुमवत दे ती है । अतः कथन 1 सही नहीां है । z वसस्टम के दो बुवनयरदी वहस्से हैं : टै ग और रीर्र।

 रीर्र द्वररर रे वर्यो तरां गोां को छोड़ वदयर जरतर है तथर RFID टै ग द्वररर वसग्नल को िरिस प्ररप्त वकयर जरतर है ,
जबवक टै ग अिनी िहचरन एिां अन्य जरनकररी को सांप्रेवर्षत करने के वलये रे वर्यो तरां गोां कर उियोग करतर

है । अतः कथन 2 सही नहीां है । यह टै ग कई फीट दू र से िस्तु की िहचरन कर सकतर है और इसे टर ै क करने
के वलये िस्तु के प्रत्यक्ष ‘लरइन-ऑफ-सरइट’ (Lineof-Sight) के भीतर होने की ािश्यकतर नहीां है ।

 प्रौद्योवगकी को 1970 के दर्क से िहले मांज़ूरी दी गई है , लेवकन हरल के िर्षों में िै वश्वक ािूवतय शांखलर
प्रबांधन और मरइक्ोवचविांग जैसी िस्तुओां में इसके उियोग के कररण यह बहुत अवधक प्रचवलत हो गई है ।

38. ।ाल ।ी िें चचाा िें र।ा ‘सा₹थी’ िोबाइल एप भनम्नभलक्टखि िें से भकससे संबंभिि ।ै ?
a) वक्प्टोकरें सी बरज़रर

b) विदे र् से प्रेर्षण स्थरनरां तररत करनर


c) िूांजी बरज़रर में वनिेर्क को वर्वक्षत करनर

d) मवहलरओां की सुरक्षर
JOIN OUR TELEGRAM CHANNEL - https://t.me/rakshaacademy
लेखक – अभिषेक भिवारी सूचना एवं संचार प्रौद्योभिकी

उिर – C
व्यरख्यर:

 हरल ही में भररतीय प्रवतभूवत और विवनमय बोर्य (SEBI) ने वनिेर्कोां को वर्वक्षत करने िरलर एक मोबरइल
एि सर₹थी लॉन्च वकयर है । 'R' को ''₹’ द्वररर प्रदवर्यत वकयर जरतर है ।

 इसकर उद्दे श्य वनिेर्कोां के बीच प्रवतभूवत बरज़रर की बुवनयरदी अिधररणरओां के बररे में जरगरूकतर िैदर
करनर है ।

 यह एि केिरईसी प्रवक्यर, व्यरिरर और वनिटरन, म्यूचुअल फांर् (एमएफ), हरवलयर बरज़रर के विकरस,
वनिेर्क वर्करयत वनिररण तांत्र ावद के बररे में भी बतरएगर। अत: विकल्प C सही है ।

39. ।ाल ।ी िें चचाा िें र।ा SUPACE, भनम्नभलक्टखि िें से भकससे संबंभिि ।ै ?
a) ICMR द्वररर विकवसत स्वदे र्ी िेसमेकर

b) यूरेनस के वलये नरसर की नई जरँ च


c) भररत के सिोच्च न्यरयरलय द्वररर कवत्रम बुक्तर्द्मिर कर उियोग
d) यूरोिीय सांघन की वमसरइल रक्षर प्रणरली
उिर – C

 SUPACE (सुप्रीम कोटय िोटय ल फॉर अवसस्टें स इन कोटय एवफवर्एां सी): इसे हरल ही में भररत के सिोच्च
न्यरयरलय द्वररर लॉन्च वकयर गयर थर।

 यह न्यरवयक प्रवक्यरओां को समझने हे तु वर्ज़रइन वकयर गयर है वजसमें स्वचरलन की ािश्यकतर होती है ,
वफर यह न्यरवयक प्रवक्यरओां को समरवहत करके दक्षतर में सु धरर तथर लांवबततर को कम करने में न्यरयरलय

की सहरयतर करतर है , इसमें एाई के मरध्यम से स्वचरवलत होने की क्षमतर होती है । अत: विकल्प C सही
है ।

40. सुपरकंप्यूटर के संदिा िें भनम्नभलक्टखि कथनों पर भवचार कीभजये:


1. सुपरकंप्यूटर की िभि को पेटाफ्लॉप्स या टे राफ्लॉप्स िें िापा जािा ।ै।

2. परि रुद्र िारि का प।ला सुपरकंप्यूटर ।ै।


उपयुाति कथनों िें से कौन-सा/से स।ी ।ै/।ड ?

a) केिल 1
b) केिल 2

c) 1 और 2 दोनोां
d) न तो 1 और न ही 2

उिर – A
व्यरख्यर:

JOIN OUR TELEGRAM CHANNEL - https://t.me/rakshaacademy


लेखक – अभिषेक भिवारी सूचना एवं संचार प्रौद्योभिकी

 सुिरकांप्यूटर एक ऐसर कांप्यूटर है जो ितयमरन में कांप्यू टर की उच्चतम िररचरलन दर िर यर उसके वनकट
प्रदर्यन करतर है ।

 ामतौर िर िे टरफ्लॉि एक सुिरकांप्यूटर की प्रसांस्करण गवत कर मरि है और इसे प्रवत से कांर् एक हज़रर
वटर वलयन फ्लोवटां ग िॉइां ट ऑिरे र्न के रूि में व्यक्त वकयर जर सकतर है ।

 FLOPS (फ्लोवटां ग िॉइां ट ऑिरे र्ांस प्रवत सेकांर्) कर उियोग ामतौर िर कांप्यूटर के प्रोसेसर के प्रदर्यन को
मरिने के वलये वकयर जरतर है ।

 फ्लोवटां ग-िॉइां ट एन्कोवर्ां ग कर उियोग करके बहुत लांबी सांख्यरओां को अिेक्षरकत ासरनी से वनयांवत्रत वकयर
जर सकतर है । अतः कथन 1 सही है ।

 भररत कर िहलर सुिरकांप्यूटर िरम 8000 थर। एक िरयलट वसस्टम के तहत 'रुद्र' (Rudra) नरमक एक
स्वदे र्ी रूि से विकवसत सियर िेटफॉमय कर िरीक्षण वकयर जर रहर है वजसमें इां टर-नोर् सांचरर हे तु वत्रनेत्र

(Trinetra) नरमक एक इां टरकनेर भी विकवसत वकयर गयर है । अतः विकल्प 2 सही नहीां है ।
41. ‘रे भ यो िरं िों’ के संदिा िें भनम्नभलक्टखि कथनों पर भवचार कीभजये:
1. इलेिरोिैग्नेभटक स्पेिरि िें रे भ यो िरं िों की िरं िदै ध्या सबसे कि ।ोिी ।ै।
2. इनकी खोज ।ेनररक ।टया ़ ने की थी।

उपयुाति कथनों िें से कौन-सा/से स।ी ।ै/।ड ?


a) केिल 1

b) केिल 2
c) 1 और 2 दोनोां

d) न तो 1 और न ही 2
उिर – B

व्यरख्यर:
 विद् युत चुांबकीय स्पेररम में रे वर्यो तरां गोां की तरां गदै ध्यय सबसे लांबी होती है । अतः कथन 1 सही नहीां है ।

 ये एक फुटबॉल के ाकरर से लेकर िथ्वी (ग्रह) के समरन विर्रल ाकरर तक हो सकती हैं ।
 रे वर्यो तरां गोां की खोज िर्षय 1880 के दर्क के अांत में हे नररक हट्य ज़ (Heinrich Hertz) ने की। अतः कथन

2 सही है ।
 रे वर्यो टे लीस्कोि की मदद से दु बयल रे वर्यो प्रकरर् तरां गोां को एकत्र वकयर जरतर है और उनकी केंद्रीयतर

बढ़रकर इनकर उियोग विश्लेर्षण हे तु वकयर जरतर है ।


 ये तररोां, ाकरर्गांगरओां, ब्लैक होल और अन्य खगोलीय विांर्ोां से प्ररकवतक रूि से उत्पन्न होने िरले रे वर्यो

प्रकरर् कर अध्ययन करने में मददगरर सरवबत होती हैं ।


 ये विर्ेर्ष रूि से वर्ज़रइन वकये गए टे लीस्कोि प्रकरर् की सबसे दीघन य तरां गदै ध्यय कर वनरीक्षण करते हैं , जो 1

वमलीमीटर से लेकर 10 मीटर से अवधक लांबे होते हैं । तुलनर के वलये दृश्यमरन प्रकरर् तरां गें केिल कुछ सौ
JOIN OUR TELEGRAM CHANNEL - https://t.me/rakshaacademy
लेखक – अभिषेक भिवारी सूचना एवं संचार प्रौद्योभिकी

नैनोमीटर लांबी होती हैं । एक नैनोमीटर करगज़ के एक टु कड़े की मोटरई कर केिल 1/10,000िरँ वहस्सर
होतर है ! िरस्ति में हम ामतौर िर रे वर्यो प्रकरर् को उसकी तरां गदै ध्यय से नहीां बक्ति उसकी ािवि से

सांदवभयत करते हैं ।


42. 'राष्ट्रीय कतभष बा़ार (e-NAM)' के संदिा िें भनम्नभलक्टखि कथनों पर भवचार कीभजये:

1. इसे वषा 2020 िें िारि िें कतभष वस्तुओ ं के भलये िौजूदा िंभ यों को ‘एक राष्ट्र एक बा़ार’ िें
एकीकति करने के उद्दे श्य से लॉन्च भकया िया था।

2. य। कतभष वस्तुओ ं के भलये एक एकीकति राष्ट्रीय बा़ार बनाने ।े िु िौजू दा AMPC िंभ यों का
नेटवका बनािा ।ै।

3. लघु भकसान कतभष व्यवसाय संघ (SFAC) इसे लािू करने वाली प्रिुख एजेंसी ।ै।
नीचे भदये िए कूट िें से स।ी भवकल्प का चयन कीभजये:

a) केिल 1 और 2
b) केिल 2 और 3
c) केिल 1 और 3
d) 1, 2 और 3

उिर - B
व्यरख्यर

 ररिरीय कवर्ष बरज़रर (e-NAM) एक अक्तखल भररतीय इलेररॉवनक टर े वर्ां ग िोटय ल है ।


 इसे अप्रैल 2016 में भररत में कवर्ष िस्तुओां के वलये मौजूदर मांवर्योां को ‘एक ररिर एक बरज़रर’ में एकीकत

करने के उद्दे श्य से लॉन्च वकयर गयर थर। अतः कथन 1 सही नहीां है ।
 यह कवर्ष उत्परदोां के वलये एक एकीकत ररिरीय बरज़रर बनरने हे तु मौजू दर APMC (कवर्ष उिज बरज़रर

कमोवर्टीज़) मांवर्योां को एक नेटिकय में एक सरथ लरतर है ।


 एकीकत बरज़ररोां में प्रवक्यरओां को सुव्यिक्तस्थत करके कवर्ष वििणन में एकरूितर को बढ़रिर दे नर। ‹

खरीदररोां एिां विक्ेतरओां के बीच सूचनर विर्षमतर को दू र करनर और िरस्तविक मरां ग िािूवतय के ाधरर िर
िरस्तविक समय मूल्य की खोज को बढ़रिर दे नर। अतः कथन 2 सही है ।

 लघन ु वकसरन कवर्ष व्यिसरय सांघन (SFAC) इसे लरगू करने िरली प्रमुख एजेंसी है । अतः कथन 3 सही है ।
 यह कवर्ष और वकसरन कल्यरण मांत्ररलय के तत्त्वरिधरन में करयय करतर है ।

43. ‘कतभत्रि बुक्टद्ििा’ के संबंि िें भनम्नभलक्टखि कथनों पर भवचार कीभजये:


1. ‘कतभत्रि बुक्टद्ििा’ उन कायों को पूरा करने वाली िीीनों को संदभिाि करिी ।ै भजनके भलये

िानव बुक्टद् की आवश्यकिा ।ोिी ।ै।


2. ‘कतभत्रि बुक्टद्ििा’ िें बड़ी े टा िकनीक िी ीाभिल ।ै।

उपयुाति कथनों िें से कौन-सा/से स।ी ।ै/।ड ?


JOIN OUR TELEGRAM CHANNEL - https://t.me/rakshaacademy
लेखक – अभिषेक भिवारी सूचना एवं संचार प्रौद्योभिकी

a) केिल 1
b) केिल 2

c) 1 और 2 दोनोां
d) न तो 1 और न ही 2

उिर - C
व्यरख्यर: कवत्रम बुक्तर्द्मतर (AI):

 ‘कवत्रम बुक्तर्द्मतर’ उन करयों को िूरर करने िरली मर्ीनोां को सांदवभयत करती है वजनके वलये मरनि बुक्तर्द् की
ािश्यकतर होती है । अतः कथन 1 सही है ।

 इसमें मर्ीन लवनिंग, िैटनय ररकवग्नर्न, वबग र्े टर, न्यूरल नेटिक्सय , सेल्फ एल्गोररथम जैसी प्रौद्योवगवकयरँ
र्रवमल हैं । अतः कथन 2 सही है ।

 इस अिधररणर की उत्पवि ग्रीक िौररवणक कथरओां में दे खी जर सकती है , िरां तु यह केिल ाधुवनक इवतहरस
के दौररन ही दे खी गई है जब सांग्रहीत प्रोग्ररम इलेररॉवनक कांप्यूटर विकवसत वकये गए थे।
 ‘कवत्रम बुक्तर्द्मतर’में जवटल करयय र्रवमल होते हैं , जैसे मर्ीन में वकसी विर्ेर्ष र्े टर को फीर् करनर और
विवभन्न क्तस्थवतयोां के अनुसरर प्रवतवक्यर दे नर।

44. भनम्नभलक्टखि कथनों पर भवचार कीभजये:

1. ग्राकीन दु भनया िें सबसे अभिक भवदय यु ि और िापीय प्रवा।कीय सािग्री ।ै ।


2. ग्राकीन का िीव्रिा के साथ ीएनए अनुक्रिण और दवा भविरण िें अनुप्रयोि ।ोिा ।ै।

उपयुाति कथनों िें से कौन-सा/से स।ी ।ै/।ड ?


a) केिल 1

b) केिल 2
c) 1 और 2 दोनोां

d) न तो 1 और न ही 2
उिर - C

व्यरख्यर:
 ग्ररफीन एक हे क्सरगोनल जरवलकर में व्यिक्तस्थत करबयन के एक रूि ग्रेफरइट से बने ग्ररफीन अणु की मोटरई

िरली सरमरन्य करबयन की एक ितली िरत होती है । कई ाश्चययजनक गुणोां के सरथ ग्ररफीन अिने ाि में
एक उल्लेखनीय िदरथय है ।

 यह लचीलर, िररदर्थ और अविश्वसनीय रूि से मज़बूत होने के सरथ-सरथ दु वनयर में सबसे ितलर, सबसे
अवधक विद् यु त ि तरिीय प्रिरहकीय सरमग्री है । अत: कथन 1 सही है ।

JOIN OUR TELEGRAM CHANNEL - https://t.me/rakshaacademy


लेखक – अभिषेक भिवारी सूचना एवं संचार प्रौद्योभिकी

 इसे प्ररयः अिने असरधररण विद् युत और इलेररॉवनक्स गुणोां के वलये एक अद् भुत सरमग्री के रूि में जरनर
जरतर है , ग्ररफीन इां वर्यम की जगह ले सकतर है और इस तरह स्मरटय फोन में OLED (ऑगेवनक लरइट-

एवमवटां ग र्रयोर्) िीन की लरगत को कम कर सकतर है ।


 ग्ररफीन में अवतररक्त अनुप्रयोगोां के वलये बहुत अवधक विकल्प हैं : एां टी-जांग कोवटां ग्स और िेंट्स, कुर्ल एिां

सटीक सेंसर, तेज़ और कुर्ल इलेररॉवनक्स, लचीलर वर्स्िे , कुर्ल सौर िैनल, ते ज़ र्ीएनए अनुक्मण, दिर
वितरण ावद। अत: कथन 2 सही है ।

45. ‘5G’ के संदिा िें भनम्नभलक्टखि कथनों पर भवचार कीभजये:


1. य। भिलीिीटर-वेव स्पेिरि का उपयोि करिा ।ै।

2. य। उन्नि LTE का उपयोि करिा ।ै। 3. य। 20 Gb/s िक की स्पी प्रदान करिा ।ै।
उपरोति कथनों िें से कौन-से स।ी ।ड ?

a) केिल 1 और 3
b) केिल 1 और 2
c) केिल 2 और 3
d) 1, 2 और 3

उिर – D
व्यरख्यर

5G प्रौद्योवगकी की विर्ेर्षतरएँ :
 वमलीमीटर िेि स्पेररम: 5G नेटिकय वमलीमीटर िेि स्पे ररम (30-300 गीगरहट्य ज़) में करम करे गर। इस

नेटिकय के मरध्यम से तीव्र गवत से अवधक मरत्रर में र्े टर भेजर जर सकतर है क्ोांवक ािवि अवधक होने के
कररण यह ासिरस के सां केतोां से बहुत कम प्रभरवित होगर। अतः कथन 1 सही है ।

 उन्नत LTE: 5G, मोबरइल िॉर्बैंर् नेटिकय में निीनतम दीघन यकरवलक अिग्रेर् (Long-Term Evolution) है ।
अतः कथन 2 सही है ।

 इां टरनेट स्पीर्: 5G के हरई-बैंर् स्पेररम में इां टरनेट की स्पीर् को 20 Gbps (प्रवत सेकांर् गीगरवबट् स) दजय
वकयर गयर है , जबवक 4G में इां टरनेट की स्पीर् 1 Gbps होती है । अतः कथन 3 सही है ।

 5G नेटिकय में र्रउनवलांक (Downlink) की स्पीर् 20 Gb/s और अिवलांक (Uplink) की स्पीर् 10 Gb/s
होगी।

46. अंिरााष्ट्रीय दू रसंचार सं घ (ITU) के संबंि िें भनम्नभलक्टखि कथनों पर भवचार कीभजये:
1. य। संयुति राष्ट्र की पं द्र।वी ं भवभीष्ट् एजें सी बन िई।

2. य। भजनेवा, क्टस्वटय जरलड िें क्टस्थि ।ै।


उपयुाति कथनों िें से कौन-सा/से स।ी ।ै/।ड ?

a) केिल 1
JOIN OUR TELEGRAM CHANNEL - https://t.me/rakshaacademy
लेखक – अभिषेक भिवारी सूचना एवं संचार प्रौद्योभिकी

b) केिल 2
c) दोनोां 1 और 2

d) न तो 1 और न ही 2
उिर - B

व्यरख्यर:
अांतररय िरीय दू रसांचरर सांघन :

 अांतररय िरीय दू रसांचरर सांघन (International Telecommunication Union- ITU) सूचनर एिां सांचरर
प्रौद्योवगकी के वलये सांयुक्त ररिर की एक विर्े र्ष एजें सी है ।

 यह सां युक्त ररिर की सभी 15 विवर्ि एजेंवसयोां में सबसे िुररनी है । अत: कथन 1 सही नहीां है ।
 वजनेिर, क्तस्वट् जरलैंर् में क्तस्थत, यह सां युक्त ररिर विकरस समूह कर सदस्य भी है तथर विश्व में इसके 12 क्षेत्रीय

और क्षेत्रीय करयरय लय हैं । अत: कथन 2 सही है ।


 इसकी सदस्यतर में 193 सदस्य ररज्य और लगभग 800 सरियजवनक और वनजी क्षेत्र की कांिवनयरां ँँ और
र्ैक्षवणक सांस्थरन के सरथ-सरथ अां तररय िरीय और क्षे त्रीय दू रसांचरर सांस्थरएां ँँ र्रवमल हैं ।
 भररत को अगले 4 िर्षों की अिवध (2019-2022) के वलये िुनः अां तररय िरीय दू रसां चरर सां घन (ITU) िररर्षद कर

सदस्य चुनर गयर है ।


 भररत िर्षय 1952 से इसकर एक वनयवमत सदस्य बनर हुा है ।

47. ओपन सोसासॉफ्टवेयर के संदिा िें भनम्नभलक्टखि कथनों पर भवचार कीभजये:


1. य। एक सॉफ्टवेयर ।ै भजसे इसके सोसा को के साथ भविररि भकया जािा ।ै।

2. य। भकसी को िी अपनी आवश्यकिाओं के अनुसार सवहोलिि कप से सॉफ्टवेयर को संीोभिि


करने की अनुिभि दे िा ।ै।

उपयुाति कथनों िें से कौन-सा/से स।ी ।ै/।ड ?


a) केिल 1

b) केिल 2
c) 1 और 2 दोनोां

d) न तो 1 और न ही 2
उिर - C

व्यरख्यर:
ओिन सोसयसॉफ्टिेयर:

 ओिन सोसयसॉफ्टिेयर (ओएसएस) िह सॉफ्टिेयर है अिने सोसय कोर् के सरथ वितररत वकयर जरतर है , जो
इसे अिने मूल अवधकररोां के सरथ उियोग, सांर्ोधन और वितरण के वलये उिलि कररतर है । अतः कथन 1

सही है ।
JOIN OUR TELEGRAM CHANNEL - https://t.me/rakshaacademy
लेखक – अभिषेक भिवारी सूचना एवं संचार प्रौद्योभिकी

 सोसय कोर् सॉफ्टिेयर कर िह वहस्सर है वजसे अवधकरां र् कांप्यूटर उियोगकिरय कभी नहीां दे खते हैं ।
 इस कोर् कर प्रयोग कांप्यूटर प्रोग्ररमर प्रोग्ररम यर एिीकेर्न के व्यिहरर को वनयांवत्रत करने हे तु हे रफेर के

वलये वकयर है ।
 ओएसएस में ामतौर िर एक लरइसेंस र्रवमल होतर है जो प्रोग्ररमर को उसकी ािश्यकतरओां के अनुसरर

सबसे सुलभ तरीके से सॉफ्टिेयर को सांर्ोवधत करने की अनुमवत दे तर है और यह वनयांवत्रत करतर है वक


सॉफ्टिेयर कैसे वितररत वकयर जर सकतर है । अतः कथन 2 सही है ।

 सोसय कोर् को स्वतांत्र रूि से उिलि कररने कर विचरर िर्षय 1983 में MIT के एक प्रोग्ररमर ररचर्य स्टॉलमैन
द्वररर अनौिचरररक रूि से स्थरवित एक िैचरररक ाां दोलन से उत्पन्न हुा थर।

 वलनक्स, मोवज़लर फरयरफॉक्स, िीएलसी मीवर्यर िे यर, सुगर CRM ावद प्रमुख उदरहरण हैं ।

48. भनम्नभलक्टखि िें से कौन अद्ा चालक की भवीेषिा ।ै/।ड ?


1. आयभनक बंिन के कारण इसका भनिााण ।ोिा ।ै
2. िनात्मक िाप भनयिांक
3. सबसे बा।री किा िें केवल चार संयोजी इलेिरॉन ।ोिे ।ड।

नीचे भदये िए कूट का प्रयोि कर स।ी उिर चुभनये:


a) केिल 1 और 3

b) केिल 2 और 3
c) केिल 3

d) 1, 2 और 3
उिर - C

व्यरख्यर:
 अर्द्य चरलक एक ऐसी सरमग्री है वजसमें सुचरलक (ामतौर िर धरतु) और कुचरलक यर ऊष्मररोधी (जैसे-

अवधकरां र् वसरे वमक) के बीच चरलन की क्षमतर होती है ।


 अर्द्य चरलक र्ुर्द् तत्त्व हो सकते हैं , जैसे- वसवलकॉन यर जमेवनयम, यर यौवगक जैसे गैवलयम ासेनरइर् यर

कैर्वमयम से लेनरइर्।
 सेमीकांर्रर वचि एक विद् युत िररिथ है , वजसमें कई घन टक होते हैं जैसे वक- टर रां वज़स्टर और अर्द्य चरलक

िेफर िर बनने िरली िरयररां ग। इन घन टकोां में से कई से युक्त एक इलेररॉवनक उिकरण को एकीकत सवकयट
(IC) कहर जरतर है और इसे कांप्यूटर, स्मरटय फोन, उिकरण, गेवमांग हरर्य िेयर और वचवकत्सर उिकरण जैसे

इलेररॉवनक उिकरणोां में िरयर जर सकतर है ।


 उनकी सबसे बरहरी कक्षर में चरर सांयोजी इलेररॉन होते हैं और सहसांयोजी बांध के कररण बनते हैं । उनकी

सबसे बरहरी कक्षर में चरर सांयोजी इलेररॉन होते हैं जो सहसांयोजक बांध के कररण होते हैं ।
JOIN OUR TELEGRAM CHANNEL - https://t.me/rakshaacademy
लेखक – अभिषेक भिवारी सूचना एवं संचार प्रौद्योभिकी

 अर्द्य चरलक में सांयोजकतर और चरलन बैंर् 1.1eV के वनवर्षर्द् ऊजरय अांतररल द्वररर अलग वकये जरते हैं ।
 अर्द्य चरलकोां में प्रवतरोध अवधक होतर है जबवक कांर्ररोां में यह बहुत छोटर और कुचरलक में बहुत अवधक

होतर है । अर्द्य चरलकोां में ऋणरत्मक तरि वनयतरां क होते हैं । अतः विकल्प C सही है ।
49. भनम्नभलक्टखि िें से कौन ‘सबारस’ (Cerberus) का सबसे अच्छा वणान करिा ।ै ?

a) य। एक बडभकंि टर ोजन ।ै।


b) य। ‘भ नायल ऑक सभवास’ अटै क (DoS) ।ै।

c) य। एक ‘ई्‍स र ॉभपंि’ (िैन-इन-द-भिभ ल) अटै क ।ै।


d) य। एक क्रॉस-साइट क्टिभिंि (XSS) सॉफ्टवेयर अटै क ।ै।

उिर - : A
व्यरख्यर:

 ‘सबयरस’ एक बैंवकांग टर ोजन है जो मुख्य रूि से क्ेवर्ट करर्य नांबर जैसे वििीय र्े टर चोरी करने हे तु उियोग
वकयर जरतर है ।
 टर ोजन एक प्रकरर कर कोर् यर सॉफ़्टिेयर है , जो र्े टर यर नेटिकय को नु कसरन िहुँ चरतर है , बरवधत करतर है ,
चोरी करतर है यर हरवनकररक कररय िरई करतर है ।

 बैंवकांग टर ोजन एक ऐसर प्रोग्ररम है , वजसकर उियोग ऑनलरइन बैंवकांग और भुगतरन प्रणरवलयोां कर उियोग
करने िरले ग्ररहकोां के बररे में गोिनीय जरनकररी प्ररप्त करने हे तु वकयर जरतर है ।

 ‘सबयरस’ कर करयय:
 यह कोविर् -19 महरमररी कर लरभ उररतर है और एक उियोगकिरय को दु भरय िनरिूणय सॉफ़्टिेयर िरले

वलांक को र्रउनलोर् करने हे तु लुभरने के वलये एसएमएस भेजतर है ।


 यह अिने एक्तिकेर्न को ामतौर िर वफवर्ांग अवभयरनोां के मरध्यम से फैलरतर है , तरवक

उियोगकिरय ओां को इसे अिने स्मरटय फोन िर इां स्टॉल करने के वलये प्रेररत वकयर जर सके।
 ऐसर प्रतीत होतर है वक वलांक िरलर ईमेल यर टे क्स्ट सां देर् बैंक जैसे वकसी विश्वसनीय स्रोत से ायर है ।

 वलांक उियोगकिरय को एक जरली िेबसरइट िर ले जरतर है और एक बरर लॉवगन नरम और िरसिर्य जैसे
वििरण दजय करने के बरद, लॉवगन क्ेर्ें वर्यल है कर तक िहुँ च जरते हैं । अतः विकल्प A सही है ।

50. इलेिरॉभनक-अपभीष्ट्/कचरा के संदिा िें भनम्नभलक्टखि कथनों पर भवचार कीभजये:

1. बेसल कन्वेंीन की नैरोबी घोषणा िें इलेिरॉभनक कचरे के प्रबंिन का उल्लेख ।ै।
2. घरे लू और व्यावसाभयक इकाइयों से कचरे के पतथक्करण, प्रसंस्करण और भनपटान के भलये िारि

का प।ला ई-कचरा क्टक्लभनक िोपाल िें स्थाभपि भकया िया ।ै।


उपयुाति कथनों िें से कौन-सा/से स।ी ।ै/।ड ?

a) केिल 1
JOIN OUR TELEGRAM CHANNEL - https://t.me/rakshaacademy
लेखक – अभिषेक भिवारी सूचना एवं संचार प्रौद्योभिकी

b) केिल 2
c) 1 और 2 दोनोां

d) न तो 1 और न ही 2
उिर - C

व्यरख्यर:
 ई-कचरर इलेररॉवनक-अिवर्ि कर सांवक्षप्त रूि है और इस र्ब्द कर प्रयोग चलन से बरहर हो चुके िुररने

इलेररॉवनक उिकरणोां कर िणयन करने के वलये वकयर जरतर है । इसमें उनके घन टक, उिभोग्य िस्तुएांँँ और
िुजे र्रवमल होते हैं ।

 इसे दो व्यरिक श्रेवणयोां के अांतगयत 21 प्रकररोां में िगथकत वकयर गयर है : „ सूचनर प्रौद्योवगकी
और सांचरर उिकरण।

 उिभोक्तर इलेक्तररकल और इलेररॉवनक्स।


 भररत में ई-कचरे के प्रबांधन के वलये िर्षय 2011 से करनू न लरगू है , जो यह अवनिरयय करतर है वक अवधकत
विघन टनकिरय और िुनचयक्णकिरय द्वररर ही ई-कचरर एकत्र वकयर जरए। इसके वलये िर्षय 2017 में ई-कचरर
(प्रबांधन) वनयम, 2016 अवधवनयवमत वकयर गयर थर।

 घन रे लू और व्यरिसरवयक इकरइयोां से कचरे को


अलग करने , प्रसांस्करण और वनिटरन के वलये

भररत कर िहलर ई-कचरर क्तक्लवनक भोिरल, मध्य


प्रदे र् में स्थरवित वकयर गयर है । अत: कथन 2 सही

है ।
 मूल रूि से बेसल किेंर्न (1992) ने ई-कचरे कर

उल्लेख नहीां वकयर थर, लेवकन बरद में इसने 2006


(COP8) में ई-कचरे के मुद्दोां को सांबोवधत वकयर।

 नैरोबी घन ोर्षणर को खतरनरक कचरे के सीमर िरर


ािरगमन के वनयां त्रण िर बेसल किेंर्न के COP9 में अिनरयर गयर थर। इसकर उद्दे श्य इलेररॉवनक कचरे

के ियरय िरण अनुकूल प्रबांधन के वलये अवभनि समरधरन तैयरर करनर है । अत: कथन 1 सही है ।
51. 'भ जी सिि कायाक्रि' के संदिा िें भनम्नभलक्टखि कथनों पर भवचार कीभजये:

1. इसे केंद्रीय श्रि िंत्रालय और िाइक्रोसॉफ्ट इं भ या ्ारा संयुति कप से लॉन्च भकया िया ।ै।
2. य। वंभचि सिुदायों से संबंभिि अद्ा -ी।री िेत्रों के रो़िार चा।ने वालों को भ भजटल कौील

प्रभीिण प्रदान करे िा।


उपयुाति कथनों िें से कौन-सा/से स।ी ।ै/।ड ?

a) केिल 1
JOIN OUR TELEGRAM CHANNEL - https://t.me/rakshaacademy
लेखक – अभिषेक भिवारी सूचना एवं संचार प्रौद्योभिकी

b) केिल 2
c) 1 और 2 दोनोां

d) न तो 1 और न ही 2
उिर - C

व्यरख्यर:
 हरल ही में केंद्रीय श्रम मांत्ररलय और मरइक्ोसॉफ्ट इां वर्यर ने सांयुक्त रूि से युिरओां की रोज़गरर क्षमतर बढ़रने

के वलये एक वर्वजटल कौर्ल मांच 'वर्जी सक्षम' (DigiSaksham) कर र्ुभररां भ वकयर है । अत: कथन 1 सही
है ।

 यह सांयुक्त िहल ग्ररमीण और अर्द्य -र्हरी क्षेत्रोां के यु िरओां को प्रोत्सरहन दे ने के वलये सरकरर द्वररर सांचरवलत
करययक्मोां कर विस्तरर है ।

 वर्जी सक्षम िहल के मरध्यम से िहले िर्षय में 3 लरख से अवधक युिरओां को बुवनयरदी कौर्ल के सरथ-सरथ
उन्नत कांप्यूवटां ग (Advanced Computing) सवहत वर्वजटल कौर्ल में मुफ्त प्रवर्क्षण प्रदरन वकयर जरएगर।
 इस िहल में िांवचत समुदरयोां से सांबांवधत अर्द्य -र्हरी क्षे त्रोां के रोज़गरर चरहने िरले लोगोां को प्ररथवमकतर दी
जरएगी, इनमें िे लोग भी र्रवमल होांगे वजन्ोांने कोविर्-19 महरमररी के कररण अिनी नौकरी गँिर दी है ।

अत: कथन 2 सही है ।


 दे र् भर में अनुसूवचत जरवत/अनुसूवचत जनजरवत के वलये मॉर्ल कॅररयर केंद्रोां (MCC) और ररिरीय कॅररयर

सेिर केंद्रोां (NCSC) में प्रवर्क्षण कर ायोजन वकयर जरएगर।


 करयरय ियन: वर्जी सक्षम को ागर खरन रूरल सिोटय प्रोग्ररम इां वर्यर (AKRSP-I) द्वररर क्षेत्र में लरगू वकयर

जरएगर।
 AKJRSP-I एक गैर-सरां प्रदरवयक, गैर-सरकररी विकरस सांगरन है । यह स्थरनीय समुदरयोां को प्रत्यक्ष सहरयतर

प्रदरन कर ग्ररमीण समुदरयोां की बेहतरी के वलये एक उत्प्रेरक के रूि में करयय करतर है ।
52 सूचना प्रद्योभिकी अभिभनयि के भकस संीोिन 53 भनम्नभलक्टखि िे से कौन सा वायरस अपने

िे ott प्लेटकािा और सोील िीभ या के आपको बार – बार एनको करके अपनी एक से
भवभनयिन को रखा िया ।ड – ज्यादा कापी बनािा ।ड –

a) 2022 a) मैक्ो िरयरस

b) 2019 b) िेब क्तिवप्टांग िरयरस


c) 2009 c) िरलीमरवफयक िरयरस

d) 2021 d) मल्टी िरयररइट िरयरस

उिर – d उिर - पालीिाभकाक वायरस

JOIN OUR TELEGRAM CHANNEL - https://t.me/rakshaacademy


लेखक – अभिषेक भिवारी सूचना एवं संचार प्रौद्योभिकी

54. भनम्नभलक्टखि िे से भकस दे ी ने भक्रिोग्राकी का b) चीन र्रैगन


प्रयोि सवाप्रथि भसक्युररटी उद्दे श्य से भकया िया – c) स्टोन िरां र्र

d) सोलर विांर्
a) भररत

b) जरिरन उिर – स्टोन पां ा


c) अमेररकर
58. ई –ीासन िे भीलांि घोषणा पत्र कब जारी
d) चीन
हुआ था

उिर – अिेररका
a) 2019
55. आईटी ऐि की कौन सी िारा कंप्यूटर b) 2020

संसािनों से छे ड़छाड़ को प्रदभीाि करिी ।ड – c) 2021


d) 2022
a) 65
b) 66 उिर – 2019 – 22वां राष्ट्रीय सम्मे लन – भवषय –

c) 67 भ भजटल इं भ या : सकलिा से उत्कतष्ट्िा


d) 73
59. राष्ट्रीय ई िवनेन्स पु रस्कार भकस वषा से प्रदान
उिर – 65 भकए जा र।े ।ै –

56. साइबर आिंकवाद से संबंभिि जुिा िे दं का a) 2002


प्राविान भकस िारा िे ।ड – b) 2001

c) 2003
a) ाईटी ऐर 2000 धररर 65
d) 2020
b) ाईटी ऐर धररर 2000 धररर
66f उिर -2003

c) ाईिीसी धररर 500


60. ई पासपोटा सेवा भवदे ी िंत्रालय व भकस
d) ाईिीसी धररर 463
प्राइवेट कंपनी का संयुति कायाक्रि ।ड

उिर - आईटी ऐि िारा 2000 िारा 66f


a) मरइक्ोसॉफ्ट

57. चीन के भकस ।ैकर सिू। ने िारि के िारि b) गूगल


बायोटे क व सीरि इं स्टीट्यूट ऑक इं भ या की c) इन्फोवसस

सूचना प्रौद्योभिकी व सप्लाई चैन को ।ैक करने d) टरटर कांसल्टें सी सवियसेज


की कोभीी की थी – वलवमटे र्

a) रे र् इको उिर – टाटा कंसल्टें सी सभवासेज भलभिटे

JOIN OUR TELEGRAM CHANNEL - https://t.me/rakshaacademy


लेखक – अभिषेक भिवारी सूचना एवं संचार प्रौद्योभिकी

61. िध्यप्रदे ी के भकस पोटा ल को भ भजटल b) भोिरल


इं भ या अवा ा 2020 से सम्माभनि भकया िया ।ड c) ग्वरवलयर

d) जबलिुर
a) एमिी अनलरइन िोटय ल

b) रोजगरर सेतु िोटय ल उिर – िोपाल


c) ई सांजीिनी िोटय ल
63. ई कािसा पर उपिोति ा संरिण अभिभनयि
d) MPTASS िोटय ल (MP TRIBAL
कब अभिसूभचि भकया िया –
AFFAIRS AUTOMATION
a) 2019
SYSTEM)
b) 2018
उिर – रोजिार सेिु पोटा ल
c) 2020
62 िध्यप्रदे ी राज्य ाटा केंद्र का िुख्यालय क।ा d) 2021

पर क्टस्थि ।ड –
उिर – 2020

a) इां दौर
64. िवनािेंट ई-िाकेटप्लेस (GeM) भसस्टि के संदिा िें भनम्नभलक्टखि कथनों पर भवचार कीभजये:

1. GeM एक वन-स्टॉप राष्ट्रीय सावाजभनक खरीद पोटा ल ।ै ।

2. िंत्रालयों व केंद्रीय सावाजभनक िेत्र के उद्यिों (CPSE) के भलए GeM से वस्तुओ ं और सेवाओं की खरीद
करना अभनवाया ।ै। 3. GeM के रखरखाव के भलये वाभणज्य और उद्योि िंत्रालय नो ल एजेंसी ।ै।

उपयुाति कथनों िें से कौन-सा/से स।ी ।ै/।ड ?

a) केिल 1
b) केिल 2

c) केिल 2 और 3
d) 1, 2 और 3

उिर – D

व्यरख्यर:

 GeM विवभन्न केंद्रीय और ररज्य सरकररोां के विभरगोां/सां गरनोां/सरियजवनक क्षेत्र के उिक्मोां (PSU) द्वररर
ािश्यक सरमरन्य उियोग की िस्तुओां और सेिरओां की ऑनलरइन खरीद की सुविधर के वलये िन-स्टॉि
ररिरीय सरियजवनक खरीद िोटय ल है । अत: कथन 1 सही है ।

JOIN OUR TELEGRAM CHANNEL - https://t.me/rakshaacademy


लेखक – अभिषेक भिवारी सूचना एवं संचार प्रौद्योभिकी

 GeM िर उिलि िस्तुओां और सेिरओां के वलये मांत्ररलयोां ि केंद्रीय सरियजवनक क्षे त्र के उद्यमोां (CPSEs) द्वररर
िस्तुओां एिां सेिरओां की खरीद करनर अवनिरयय है । अत: कथन 2 सही है ।

 यह सरकररी उियोगकिरय ओां को उनके िैसे कर सिोिम मूल्य प्ररप्त करने की सु विधर के वलये ई-बोली और
ररिसय ई-नीलरमी जैसे उिकरण भी प्रदरन करतर है । z इसे िर्षय 2016 में सरकररी खरीद प्रवक्यर में

िररदवर्यतर और दक्षतर लरने के वलये लॉन्च वकयर गयर थर।


 GeM के रखरखरि के वलये िरवणज्य और उद्योग मांत्ररलय नोर्ल एजेंसी है । अत: कथन 3 सही है ।

65. िारिनेट प्रोजेि के संदिा िें भनम्नभलक्टखि कथनों पर भवचार कीभजये:

1. य। ऑभिकल काइबर का उपयोि कर भवश्व का सबसे बड़ा ग्रािीण ब्रॉ बड कनेक्टिभवटी कायाक्रि ।ै।

2. इसका कायााक्टन्वि इलेिरॉभनक्स और सूचना प्रौद्योभिकी िंत्रालय ्ारा भकया जा र।ा ।ै।

उपयुाति कथनों िें से कौन-सा/से स।ी ।ै/।ड ?

a) केिल 1
b) केिल 2

c) 1 और 2 दोनोां
d) न तो 1 और न ही 2

उिर - A

व्यरख्यर:

 यह ऑवप्टकल फरइबर कर उियोग कर विश्व कर सबसे बड़र ग्ररमीण िॉर्बैंर् कनेक्तरविटी करययक्म है और
भररत िॉर्बैंर् नेटिकयवलवमटे र् (BBNL) द्वररर करयरय क्तित एक प्रमुख वमर्न भी। अत: कथन 1 सही है ।
 BBNL, भररत सरकरर द्वररर कांिनी अवधवनयम, 1956 के तहत 1000 करोड़ रुिए की अवधकत िूांजी के सरथ

स्थरवित एक विर्ेर्ष प्रयोजन िरहन (SPV) है ।


 इसे सांचरर मांत्ररलय के तहत दू रसांचरर विभरग द्वररर करयरय क्तित वकयर जर रहर है ।अत: कथन 2 सही नहीां है ।

 अक्तूबर 2011 में नेर्नल ऑवप्टकल फरइबर नेटिकय (National Optical Fibre Network- NOFN) को
लॉन्च वकयर गयर थर, िर्षय 2015 में इसकर नरम बदलकर भररत नेट प्रोजेर (Bharat Net Project) कर

वदयर गयर। ‹ NOFN को ग्ररम िांचरयतोां (Gram Panchayats) तक िॉर्बैंर् कनेक्तरविटी िहुां ँँचरने हे तु एक
मज़बूत बुवनयरदी ढरां ँँचे के वनमरय ण के मरध्यम से एक सू चनर सुिरहरइिे के रूि में िररकक्तल्पत वकयर गयर

थर।

66. 'ऐक्टम्बटै ि’ (AmbiTag) के संदिा िें भनम्नभलक्टखि कथनों पर भवचार कीभजये:

JOIN OUR TELEGRAM CHANNEL - https://t.me/rakshaacademy


लेखक – अभिषेक भिवारी सूचना एवं संचार प्रौद्योभिकी

1. य। इं टरनेट ऑक भथं ग्स (IoT) पर आिाररि भ वाइस ।ै।

2. य। कोल्ड चेन प्रबंिन के भलये िापिान े टा लॉिर ।ै।

उपयुाति कथनों िें से कौन-सा/से स।ी ।ै/।ड ?

a) केिल 1

b) केिल 2
c) 1 और 2 दोनोां

d) न तो 1 और 2 न ही

उिर – C

व्यरख्यर:

 भररतीय प्रौद्योवगकी सांस्थरन (IIT) रोिड़, िांजरब ने अिनी तरह कर िहलर इां टरनेट ऑफ वथांग्स (Internet of

Thing) उिकरण ऐक्तिटै ग (AmbiTag) विकवसत वकयर है । अतः कथन 1 सही है ।

यह वर्िरइस खररब होने िरले उत्परदोां, टीकोां और यहरँ तक वक र्रीर के अांगोां ि रक्त के सांचरर के दौररन
उनके ासिरस कर ररयल टरइम तरिमरन दजय करतर है ।

 इस वर्िरइस को प्रौद्योवगकी निरचरर केंद्र अिध (AWADH- कवर्ष एिां जल तकनीकी विकरस केंद्र) और

उसके स्टरटय अि िैचनेस्ट (ScratchNest) के अांतगयत विकवसत वकयर गयर है ।

 यह यूवनिसयल सीररयल बस (Universal Serial Bus- USB) वर्िरइस के ाकरर कर है । z यह कोल्ड चेन
प्रबांधन के वलये भररत कर िहलर स्वदे र्ी तरिमरन र्े टर लॉगर है । अतः कथन 2 सही है ।
o यह एक बरर चरजय होकर िू रे 90 वदनोां तक वकसी भी टरइम ज़ोन में और -40 से +80 वर्ग्री सेंटीग्रेट
तक के िरतरिरण में वनरां तर तरिमरन दजय करतर रहतर है ।

o ितय मरन में अां तररय िरीय बरज़रर में उिलि इस तरह की वर्िरइस वसफय 30-60 वदनोां तक की अिवध
के वलये तरिमरन दजय करती है ।

o जब तरिमरन िूिय वनधरय ररत सीमर से ऊिर जरतर है तो यह एक अलटय जररी करतर है ।
o दजयवकये गए र्े टर को वकसी कांप्यूटर को USB से जोड़कर हरवसल वकयर जर सकतर है ।

67. 'राष्ट्रीय कतभत्रि बु क्टद्ििा पोटा ल' के संदिा िें भनम्नभलक्टखि कथनों पर भवचार कीभजये

1. य। इलेिरॉभनक्स और सूचना प्रौद्योभिकी िंत्रालय (MeitY) िथा नीभि आयोि की एक संयुति प।ल ।ै।

2. य। िारि और उसके बा।र कतभत्रि बुक्टद्ििा (AI) से संबंभिि िभिभवभियों के भलये एक केंद्रीय ।ब के
कप िें काया करिा ।ै।

JOIN OUR TELEGRAM CHANNEL - https://t.me/rakshaacademy


लेखक – अभिषेक भिवारी सूचना एवं संचार प्रौद्योभिकी

उपयुाति कथनों िें से कौन-सा/से स।ी ।ै/।ड ?

a) केिल 1
b) केिल 2

c) 1 और 2 दोनोां
d) न तो 1 और न ही 2

उिर - B

व्यरख्यर:

 28 मई, 2021 को 'ररिरीय कवत्रम बुक्तर्द्मिर िोटय ल' (National AI Portal) ने अिनी िहली िर्षयगरँ र मनरई।
 यह इलेररॉवनक्स और सूचनर प्रौद्योवगकी मांत्ररलय (Ministry of Electronics and IT- MeitY), ररिरीय ई-

गिनेंस वर्िीजन (National e-Governance Division- NeGD) और नैसकॉम (NASSCOM) की एक


सांयुक्त िहल है । अतः कथन 1 सही नहीां है ।

 यह भररत और उसके बरहर कवत्रम बुक्तर्द्मिर (AI) से सां बांवधत समरचरर, सीखने, लेख, घन टनरओां और
गवतविवधयोां ावद के वलये एक केंद्रीय हब (Hub) के रूि में करयय करतर है । अतः कथन 2 सही है ।

 ररिरीय ई-गिनेंस वर्िीज़न: िर्षय 2009 में वर्वजटल इां वर्यर कॉरिोरे र्न (MeitY द्वररर स्थरवित एक गैर-
लरभकररी कांिनी) के तहत NeGD को एक स्वतां त्र व्यरिरर प्रभरग के रूि में स्थरवित वकयर गयर थर।

 NASSCOM एक गैर-लरभकररी औद्योवगक सांघन है जो भररत में IT उद्योग के वलये सिोच्च वनकरय है ।

68. अपराि और आपराभिक टर ै भकंि नेटवका एवं भसस्टि (CCTNS) के संबंि िें भनम्नभलक्टखि कथनों पर

भवचार कीभजये:

1. इसे राष्ट्रीय अपराि ररकॉ ा ब्यू रो (NCRB) ्ारा भवकभसि भकया िया ।ै।

2. य। भवज्ञान और प्रौद्योभिकी िंत्रालय की नेीनल ई-िवनेंस योजना के ि।ि एक भिीन िो पररयोजना


।ै।

3. इसका उद्दे श्य पुभलस के कािकाज को नािररक भ।िैषी बनाना ।ै।

उपयुाति कथनों िें से कौन-सा/से स।ी ।ै/।ड ?

a) केिल 2 और 3
b) केिल 1 और 2

c) केिल 1 और 3
d) 1, 2 और 3

JOIN OUR TELEGRAM CHANNEL - https://t.me/rakshaacademy


लेखक – अभिषेक भिवारी सूचना एवं संचार प्रौद्योभिकी

उिर –C

व्यरख्यर:

 अिररध और ािररवधक टर ै वकांग नेटिकय एिां वसस्टम (Crime and Criminal Tracking Network and
SystemsCCTNS) एक केंद्रीय विििोवर्षत योजनर है , वजसे ररिरीय अिररध ररकॉर्य ब्यू रो (National Crime

Records Bureau) द्वररर विकवसत वकयर गयर है । अतः कथन 1 सही है ।


o यह गह मांत्ररलय के नेर्नल ई-गिनेंस िरन (National e-Governance Plan) के तहत स्थरवित

एक वमर्न मोर् प्रोजेर है । अतः कथन 2 सही नहीां है ।


 इसे िर्षय 2009 में मांज़ूरी दी गई थी। ‹ यह एक सुरवक्षत एिीकेर्न है जो दे र् के 97% से अवधक िुवलस

स्टे र्नोां को जोड़तर है ।


 उद्दे श्य: िुवलस थरनोां के करमकरज को िररदर्थ करके िुवलस के करमकरज को नरगररक वहतैर्षी और अवधक

िररदर्थ बनरनर। अतः कथन 3 सही है । ाईसीटी के प्रभरिी उियोग के मरध्यम से नरगररक केंवद्रत सेिरओां
के वितरण में सुधरर लरनर। अिररध और अिररवधयोां की सटीक एिां तीव्र जरँ च के वलये जरँ च अवधकरररयोां को

अद्यवतत उिकरण, तकनीक और जरनकरररयरँ प्रदरन करनर।

69. भनम्नभलक्टखि कथनों पर भवचार कीभजये:

1. 2G िकनीक भ भजटल रे भ यो भसग्नल का उपयोि करिा ।ै।

2. 4G िकनीक को 100 Mbps से 1 Gbps की अभिकिि स्पी के साथ लॉन्च भकया िया था।

3. 5G िकनीक केवल ।ाई-बड स्पेिरि िें काया करिी ।ै भजसिें उच्च िभि के साथ-साथ अभिकिि कवरे ़
िी ।ोिा ।ै। उपयुाति कथनों िें से कौन-सा/से स।ी ।ै/।ड ?

a) केिल 1 और 2

b) केिल 2
c) केिल 2 और 3

d) 1, 2 और 3

उिर - B

व्यरख्यर:

 2जी को 1990 के दर्क में लॉन्च वकयर गयर थर, जो वर्वजटल रे वर्यो वसग्नल कर उियोग करतर है और 64

केबीिीएस की बैंर्विर्् थ के सरथ िॉयस और र्े टर टर रां सवमर्न दोनोां कर करयय करतर है । अतः कथन 1 सही
नहीां है ।

JOIN OUR TELEGRAM CHANNEL - https://t.me/rakshaacademy


लेखक – अभिषेक भिवारी सूचना एवं संचार प्रौद्योभिकी

 1जी को 1980 के दर्क में लॉन्च वकयर गयर थर और इसने एनरलॉग रे वर्यो वसग्नल िर करययवकयर तथर
केिल िॉयस कॉल को सांभि बनरयर। z 4जी को िर्षय 2009 में 100 एमबीिीएस से 1 जीबीिीएस की

अवधकतम स्पीर् के सरथ लॉन्च वकयर गयर थर और यह 3र्ी ाभरसी िरस्तविकतर को भी सक्षम करतर है ।
अतः कथन 2 सही है ।

 5जी ‘लॉन्ग-टमय एिोलूर्न’ (LTE) मोबरइल िॉर्बैंर् नेटिकय में सबसे निीनतम अिग्रेर् है ।
 5G मुख्य रूि से 3 बैंर् (लो, वमर् और हरई बैंर् स्पेररम) में करयय करतर है , वजसमें सभी के बैंर््स के कुछ

विवर्ि उियोग और कुछ विवर्ि सीमरएँ हैं ।


 लो बैंर् स्पेररम (Low Band Spectrum): इसमें इां टरनेट की गवत और र्े टर के इां टरै क्शन-प्रदरन की

अवधकतम गवत 100Mbps (प्रवत सेकांर् मेगरवबट् स) तक होती है ।


 वमर् बैंर् स्पेररम (Mid-Band Spectrum): इसमें लो बैंर् के स्पेररम की तु लनर में इां टरनेट की गवत

अवधक होती है , वफर भी इसके किरे ज क्षेत्र और वसग्नलोां की कुछ सीमरएँ हैं ।
 हरई बैंर् स्पेररम (High-Band Spectrum): इसमें उिरोक्त अन्य दो बैंर््स की तुलनर में उच्च गवत होती है ,
लेवकन किरे ज और वसग्नल भेदन की क्षमतर बेहद सीवमत होती है । अतः कथन 3 सही नहीां है ।

70. भनम्नभलक्टखि कथनों पर भवचार कीभजये:

1. ‘इं टरनेट एक्सचेंज प्वाइं ट’ एक ऐसी सुभविा ।ै जो इं टरनेट सेवा प्रदािाओं को आपस िें भिलाने और

टर ै भ़िक का आदान-प्रदान करने की अनुिभि दे िी ।ै।

2. ‘िारि का राष्ट्रीय इं टरनेट एक्सचें ज’ ोिेन को के नाि, इं टरनेट एक्सचेंज और इं टरनेट प्रोटोकॉल का
प्रबंिन करिा ।ै। उपयुाति कथनों िें से कौन-सा/से स।ी ।ै/।ड ?

a) केिल 1

b) केिल 2
c) 1 और 2 दोनोां

d) न तो 1 और न ही 2

उिर – C

व्यरख्यर

 एक इां टरनेट एक्सचेंज प्वरइां ट एक सुविधर है जो इां टरनेट सेिर प्रदरतरओां को ािस में वमलरने और टर ै वफ़क

कर ादरन-प्रदरन करने की अनुमवत दे तर है , वजसे ‘वियररां ग’ भी कहर जरतर है ।


o इां टरनेट एक्सचेंज िॉइां ट्स (IXPs) इां टरनेट के इन्िरस्टर क्चर कर सबसे महत्त्विूणयवहस्सर होते हैं ।

JOIN OUR TELEGRAM CHANNEL - https://t.me/rakshaacademy


लेखक – अभिषेक भिवारी सूचना एवं संचार प्रौद्योभिकी

o यह ISPs के वलये अां तररय िरीय बैंर्विर्् थ िर िै से बचरतर है और दे री को कम करके अिने ग्ररहकोां के
वलये कनेक्तरविटी में सुधरर करतर है । अतः कथन 1 सही है ।

 NIXI वनम्रां वकत गवतविवधयोां के मरध्यम से इां टरनेट की बुवनयरदी अिसांरचनर तक भररत के नरगररकोां की
िहुँ च स्थरवित करने के वलये िर्षय 2003 से करम कर रही एक गैर-लरभकररी सांस्थर (कांिनी अवधवनयम 2013

की धररर 8 के तहत) है :
o इां टरनेट एक्सचेंज के मरध्यम से इां टरनेट सेिर प्रदरतरओां (ISP’s), र्े टर केंद्रोां और सरमग्री वितरण

नेटिकय (CDNs) के बीच इां टरनेट र्े टर कर ादरन-प्रदरन करनर।


o .IN रवजस्टर ी, .IN कांटर ी कोर् र्ोमेन और .BHARAT IDN (अांतररय िरीय र्ोमेन नरम) र्ोमेन कर

िांजीकरण, प्रबांधन और सांचरलन। इां टरनेट नरम और सांख्यर के वलये भररतीय रवजस्टर ी (IRINN),
इां टरनेट प्रोटोकॉल (IPv4/IPv6) कर प्रबांधन और सांचरलन। अतः कथन 2 सही है ।

71 साइबर अपरािों के संदिा िेंभनम्नभलक्टखि कथनों पर भवचार कीभजये:

1. स्पूभकंि ।िलों को भकसी केकंप्यूटर िक पहुाँच या िभि पहुाँचानेऔर व्यक्टति िि जानकारी एकत्र करनेके

भलये भ ़ाइन भकया िया ।ै।

2. इं टरनेीनल टे लीकम्युभनकेीन यूभनयन (ITU) साइबर सु रिा िुद्दों से भनपटनेके भलयेसंयुति राष्ट्र की एक

भवभीष्ट् एजेंसी ।ै। 3. िारि साइबर अपराि पर बु ापेस्ट अभिसिय का ।स्तािकिाा न।ी ं ।ै।

उपयुाति कथनों िेंकौन-सा/सेस।ी ।ै/।ड ?

a) केिल 1

b) केिल 2 और 3
c) केिल 3

d) केिल 1 और 3

उिर - A

व्यरख्यर:

सरइबर हमलर वकसी कांप्यूटर और कांप्यूटर नेटिकय केअनवधकत उियोग तथर उसेउजरगर करने , बदलने , अक्षम

करने , नि करने , चोरी करने यर उस तक अनवधकत िहुँ च प्ररप्त करनेकर प्रयरस है ।

सरइबर हमलर वकसी भी प्रकरर की ऐसी ाक्रमक युक्तक्त है जो कांप्यूटर सूचनर प्रणरली, इन्िरस्टर क्चर, कांप्यूटर
नेटिकय यर व्यक्तक्तगत कांप्यूटर उिकरणोां को लवक्षत करती है ।

सरइबर हमले के तरीके-

JOIN OUR TELEGRAM CHANNEL - https://t.me/rakshaacademy


लेखक – अभिषेक भिवारी सूचना एवं संचार प्रौद्योभिकी

 वफवर्ांग यर स्पूवफांग हमले: स्पूवफांग में हमलरिर अिनी असल िहचरन को वछिरकर खुद को एक विश्वसनीय
स्रोत केरूि मेंप्रस्तुत करते हैं अथरय त् िह िैध उियोगकिरय की िहचरन कर उियोग करनेकी कोवर्र् करतर

है ।
 वफवर्ांग िह प्रवक्यर है वजसमेंकोई व्यक्तक्त उियोगकिरय की सांिेदनर्ील जरनकररी जैसे- बैंक खरतर वििरण

ावद को चुररतर है । अतः कथन 1 सही नहीां है ।


 मैलिेयर यर स्परइिेयर: स्परइिेयर एक प्रकरर कर मैलिे यर है जो वर्वज़टल वर्िरइस जैसे- कांप्यूटर, मोबरइल,

टे बलेट ावद सेगुप्त एिां वनजी जरनकरररयरँ चुररतर है । यह जीमेल अकरउां ट, बैंक वर्टे ल्स, सोर्ल मीवर्यर
सेलेकर टे क्स्ट मैसेज जैसी गवतविवधयोां िर नज़र रखतर है एिां िहरँ सेर्ेटर चोरी कर अिनेऑिरे टर तक

िहुँ चरतर है ।
 वसम स्वैि (SIM Swap): इसमेंमूल वसम कर एक क्लोन बनरकर मूल वसम को अमरन्य कर वदयर जरतर है और

र्ु क्तिकेट वसम कर उियोग उियोगकिरय केऑनलरइन बैंक खरतेसेधनररवर् स्थरनरां तररत करनेके वलये वकयर
जर सकतर है ।

अांतररय िरीय तां त्र: अांतररय िरीय दू रसांचरर सां घन (ITU): यह सांयुक्त ररिर की एक विर्े र्ष एजें सी है जो दू रसां चरर और सरइबर

सुरक्षर मुद्दोां केमरनकीकरण तथर विकरस मेंअग्रणी भूवमकर वनभरती है । अतः कथन 2 सही है ।

सरइबर अिररध िर बुर्रिे स्ट अवभसमय:

 बुर्रिेस्ट किेंर्न सरइबर क्रइम िर एक किेंर्न है , वजसेसरइबर अिररध िर बुर्रिेस्ट किेंर्न यर

बुर्रिेस्ट किेंर्न केनरम सेजरनर जरतर है ।


 यह अिनी तरह की िहली ऐसी अां तररय िरीय सां वध है वजसकेअां तगयत ररिरीय करनू नोां को सु व्यिक्तस्थत कर जरँ च-

िड़तरल की तकनीकोां में सुधरर करनेतथर इस सांबांध में विश्व केअन्य दे र्ोां केबीच सहयोग बढ़रने हे तुइांटरनेट
और कांप्यूटर अिररधोां िर रोक लगरनेकी मरां ग की गई है । यह 1 जुलरई, 2004 को लरगूहुा। भररत इस

सम्मेलन कर हस्तरक्षरकिरय नहीां है । अतः कथन 3 सही है ।

इां टरनेट गिनेंस फोरम (IGF): यह इां टरनेट गिनेंस वर्बेट िर सभी वहतधररकोां यरनी सरकरर, वनजी क्षेत्र और
नरगररक समरज को एक सरथ लरतर है ।

72. इं टरनेट ए र ै स कौन दे िा ।ड –

a) इन्टरनेट ावकयटे क्चर


b) केंद्र सरकरर

c) प्रदे र् सरकरर
d) मरइक्ोसॉफ्ट

JOIN OUR TELEGRAM CHANNEL - https://t.me/rakshaacademy


लेखक – अभिषेक भिवारी सूचना एवं संचार प्रौद्योभिकी

उिर - इन्टरनेट ावकयटे क्चर

73. भ भजटल कंप्यूटर भकस भसद्ांि पर काया करिा ।ड

a) गणनर
b) मरिन

c) विद् युत
d) लॉजीकल वसर्द्रां त

उिर – लॉजीकल वसर्द्रां त

सेट – 3

1. भकसी िी िाध्यि का प्रयोि कर सूचना B. 1952

का आदान-प्रदान करना क।लािा ।ै C. 1980


A. सूचनर D. 1964

B. सांचरर
उत्तर -(b)
C. बरत-चीत
4. भिस्टे पेयर केबल भबना ररपीटर के
D. इनमें से कोई नहीां।
भकिनी दू री िक ाटा का स्थानांिरण
उत्तर -(b)
करिा ।ै

2. व। िाध्यि भजसके ्ारा सेण्डर (Sender) A. 1k m


ररसीवर को िैसेज या ाटा िेजिा ।ै B. 10 km

क।िें ।ै C. 10 m
A. मरध्यम D. 100 mtr.

B. केबल
उत्तर -(a)
C. सांचरर मरध्यम
5. ाटा संचरण की िभि िापने की इकाई
D. इनमें से कोई नहीां।
क्या ।ै
उत्तर (c)
A. sec
3. ऑभिकल काइबर भजसका प्रयोि संचार B. Bit

िाध्यि िें भकया जािा ।ै उसका C. Baud


आभवष्कार कब हुआ? D. Band
A. 1950
उत्तर -(d)

JOIN OUR TELEGRAM CHANNEL - https://t.me/rakshaacademy


लेखक – अभिषेक भिवारी सूचना एवं संचार प्रौद्योभिकी

10. दे ी की प।ली सेल्युलर सेवा कब प्रारं ि


की ।ै
6. इलेिरोिैग्नेभटक वेब का कान्सेि
A. Aug 1995
भनम्नभलक्टखि िें से भकसने भदया ।ै।
B. Aug 1996
A. सैमुअल मरसय
C. July 1995
B. फ्लेवमांग
D. इनमें से कोई नहीां।
C. मैक्सिेल
D. इनमें से कोई नहीां। उत्तर -(a)

उत्तर -(c) 11. FDMA -

A. Fine division Multiple Access


7. िािा भकरणों की खोज भकसने की ।ै
B. Frequency division Multiple Access
A. ररजन
C. Frequency divide Manipulation
B. बैकुरल
Access
C. ररटर
D. इनमें से कोई नहीां।
D. इनमें से कोई नहीां।
उत्तर -(b)
उत्तर -(b)
12. प्रथि पीढ़ी का के िोबाइल भनम्नभलक्टखि िें
8. को।रे िें कोटोग्राकी के भलये भनम्नभलक्टखि
से भकस दे ी ने भवकभसि भकये ।ै
िें से कौन सी ।ै?
A. जरिरन
A. अिरक्त वकरणें
B. चीन
B. रे वर्यो बेि
C. अमेररकर
C. मरइक्ो बेि
D. विटे न
D. इनमें से कोई नहीां।
उत्तर -14-(a)
उत्तर -(a)
13. टे लीकोन का आभवष्कार भकस दे ष िें हुआ
9. भनम्नभलक्टखि िें से क्या एनालाि भसग्नल का
था?
उदा।रण ।ै
A. जरिरन
A. मरनि ािरज
B. भररत
B. असतत वकरण
C. अमेररकर
C. स्क्वरयर बेि
D. वफनलैण्ड
D. इनमें से कोई नहीां।
उत्तर -15-(d)
उत्तर --(a)

JOIN OUR TELEGRAM CHANNEL - https://t.me/rakshaacademy


लेखक – अभिषेक भिवारी सूचना एवं संचार प्रौद्योभिकी

14. GPRS - उत्तर -(a)


A. General Packet Radio Service Cellular
18. ICTAI िे T ििलब क्या ।ै
Technology
A. Technology
B. General Point Radio Service Cellular
B. Transformative
Technology
C. Transform
C. General Packet Radio Service Cell
D. इनमें से कोई नहीां।
Technology
उत्तर --(b)
D. इनमें से कोई नहीां।

19. भवश्व के भकस दे ी ने AI की ीु कआि की


उत्तर -(a)
A. जरिरन
15. भनम्नभलक्टखि िें से कौन सी Generation
B. ऑक्तस्टरयर
IOT के भलये उपयुति िोबाइल जनरे ीन ।ै
C. चीन
A. 2 G
D. अमेररकर
B. 3 G
उत्तर -(a)
C. 4 G

D. 5 G 20. भनम्नभलक्टखि िें से कौन सा पूणा

प्रभिभक्रयात्मक AI का उदा।रण ।ै ?
उत्तर --(d)
A. Deep Blue
16. Smart Phone का जनक कौन सा ।ै
B. Watson
A. सैमुअल मरसय
C. Siri
B. मरवटय न कूिर
D. इनमें से कोई नहीां।
C. ग्ररहम बेल
उत्तर -(a)
D. इनमें से कोई नहीां।

21. प।ले इलेिराभनक प्रविाक (Amplifier)


उत्तर -(b)
की खोज भकसने की थी?
17. विािान िें Smart Phone िें कौन सा
A. विल्हे न रां ग्जेन
Android Version Install ।ै
B. जोसेफ जे भरमसन
A. Android 12.0
C. जरन एिुर्ष फ्लेवमांग
B. Android 9.0
D. ली.र्ी. फोरस्ट
C. Android 10.0
उत्तर -(d)
D. इनमें से कोई नहीां।

JOIN OUR TELEGRAM CHANNEL - https://t.me/rakshaacademy


लेखक – अभिषेक भिवारी सूचना एवं संचार प्रौद्योभिकी

22. प्रकाी िंिु का उत्पादन करने वाला दे ी 26. सबसे प।ला वाल्ड रोबोट का भनिााण
का प।ला राज्य कौन ।ै भकसने भकया था

A. ररजस्थरन A. केविन िरवनयक


B. मध्यप्रदे र् B. प्रो. ररल्फ होवलस

C. उिरप्रदे र् C. जरजय र्े िेल


D. वबहरर D. र्े विर् हे नसन

उत्तर -(b) उत्तर -(b)

23. DTH की ीु कआि िारि िें कब हुयी थी 27. रोबोट के ।ाथों को सािान्यिः क।ा जािा

A. Oct 2003 ।ै
B. Oct 2005 A. प्रियतक

C. Jan 2003 B. प्रचरलक


D. Jan 2005 C. वनयांत्रक

D. एां र्इफेरर
उत्तर -(a)
उत्तर -(d)
24. िारि की दू रसं चार भनयािक बा ी TRAI
के विािान अध्यि कौन ।ै 28. भवश्व के प।ले रोबोट किाचारी युति ।ोटल

A. िी र्ी िरघन ेलर की ीु कआि भकस दे ष िें हुआ ।ै


B. र्ॉ. नांदन A. अमेररकर

C. र्र. िररष्ठ र्मरय B. जरिरन


D. इनमें से कोई नहीां। C. चीन

D. दवक्षण कोररयर
उत्तर --(a)
उत्तर -(b)
25. सबसे प।ले Android ऑपरे भटं ि भसस्टि
पर आिाररि िोबाइल कोन भकस कंपनी 29. सेन्टर कार आभटा भकभषयल इन्टे लीजेंस

ने लांच भकया था? एण्ड रोबोभटक्स संस्थान क।ां क्टस्थि ।ै


A. HTC A. बेंगलुरू

B. Samsung B. चेन्नई
C. Nokia C. कोलकरतर

D. Vivo D. अहमदरबरद

उत्तर -(a) उत्तर -(a)

JOIN OUR TELEGRAM CHANNEL - https://t.me/rakshaacademy


लेखक – अभिषेक भिवारी सूचना एवं संचार प्रौद्योभिकी

30. Automatic vehicle िें प्रयुति ।ोने वाला 34. भनम्न िें से भकसे टोिल (Toggle) कंु जी
AI भनम्नभलक्टखि िें से कौन सा ।ै - क।िे ।ै

A. सरमरन्य AI A. एण्टर
B. सीवमत स्मवत B. नमलॉक कुांजी

C. ात्म चेतन C. कैप्सलॉक कुजी


D. कमजोर AI D. b और c दोनो

उत्तर -(b) उत्तर --(d)

31. AI के भलए बनाई िई प्रोग्राभिंि िाषा कौन 35. DPI दीाािा ।ै

सी ।ै A. dot per inch


A. LISP B. dot pikel ink

B. PASCAL C. (c) Digits per unit


C. COBOL D. Diagra per inch

D. ALGOL
उत्तर --(a)
उत्तर -(a)
36. OCR प्रकाष स्त्रोि की स।ायिा से
32. E- कािसा का िात्पया ।ै कैरे िर के ......को प।चानना ।ै। के

A. इां टरनेट कर व्यरिरररक लेखर जोखर करनर A. सरइज


B. इां क्तिर् में लेखर जोखर करनर B. कलर

C. इां टरनेट में एकरां उट सीखनर C. र्ेि


D. उिरोक्त सभी D. यूज वलांक

उत्तर -(a) उत्तर --(d)

33. विािान िें दु भनया िें सबसे ज्यादा रे वेन्यु 37. भनम्नभलक्टखि िें से कौन CPU का एक िाि
जेनरे ट करने वाली ई-कािसा कंपनी कौन ।ै

सी ।ै A. की बोर्य
A. Alibaba B. वप्रांटर

B. Amazon C. टे ि
C. Jabong D. ए.एल.यू

D. e-bay
उत्तर -(d)
उत्तर -(b)
38. क्‍प्यूटर िें ाटा भकसे क।िे ।ै

JOIN OUR TELEGRAM CHANNEL - https://t.me/rakshaacademy


लेखक – अभिषेक भिवारी सूचना एवं संचार प्रौद्योभिकी

A. सांख्यर को D. 2006
B. सूचनरओां को
उत्तर --(c)
C. वचन् को
D. वचन् एिां सांख्यरत्मक सूचनर को

43. NIC -?
उत्तर --(d)
A. National Information Center
39. क्‍प्यूटर क्या ।ै
B. Novel Industrial Center
A. इलेरररवनक मर्ीन
C. Nodal Industrial Center
B. मरनि मर्ीन
D. National Industrial Center
C. िरिर मर्ीन
उत्तर --(a)
D. इनमें से कोई नहीां।

44. IT Act 2000_________िें प्रिाव िें आया?


उत्तर -(a)
A. 12 July 2000
40. NIC NET िें लांच हुआ था?
B. 11 Dec 2000
A. 1985
C. 17 Oct 2000
B. 1987
D. 14 Sect 2000
C. 1983
उत्तर -(b)
D. 1984

45. क्‍प्यूटर के काया करने का भसद्ांि ।ै


उत्तर --(b)
A. Input
41. SWANs -
B. Output
A. State wide Area Networks
C. Process
B. State wise Area Networks
D. इनमें से कोई नहीां।
C. Secured wide Area Network
उत्तर -(c)
D. इनमें से कोई नहीां।

46. क्‍प्यूटर ्ारा भदया िया पररणाि ।ै


उत्तर -(a)
A. र्रटर
42. RTI Act सन…………. िें इ्‍प्लीिेंट हुआ
B. मेमोरी
था
C. ाउटिुट
A. 2004
D. इनिुट
B. 2007
उत्तर -(c)
C. 2005

JOIN OUR TELEGRAM CHANNEL - https://t.me/rakshaacademy


लेखक – अभिषेक भिवारी सूचना एवं संचार प्रौद्योभिकी

47. बुभनयादी क्‍प्यूटर प्रोसेभसंि चक्र िें 51. िाइक्रो क्‍प्यूटर को टे लीकोन से कौन सी
ीाभिल ।ै व्यावस्था जोड़िी ।ै ?

A. इनिुट, प्रोसेवसांग ाउटिुट A. VDU


B. वसस्टम अिीकेर्न B. यूवनक्स

C. र्रटर, सूचनर और अिीकेर्न C. मॉर्े म


D. हरर्य िेयर, सॉफ्टिेयर, स्टोरे ज D. सी.िी.यू

उत्तर -(a) उत्तर --(c)

48. क्‍प्यूटर घड़ी के स्पी की िणना की 52. भनकनेट (NIC NET) ।ै

जािी ।ै A. एक अांतररय िरीय नेटिकय


A. गीगर बरइट B. विर्े र्ष तरर कर बुनर जरल

B. वबट C. इां टरनेट कर दू सरर नरम


C. मेगर हरटय ज D. इनमें से कोई नहीां।

D. सेकेन्ड
उत्तर --(d)
उत्तर -(c)
53. संचार नेटवका भजसका प्रयोि बड़ी
49. BIOS संस्थाओं ्ारा प्रादे भीक, राष्ट्रीय और

A. Basic Intra operating system. वैभश्वक िेत्र िें भकया जािा ।ै क।लािा ।ै
B. Basic Internal organ system. A. LAN

C. Basic Input output system. B. WAN


D. इनमें से कोई नहीां। C. MAN

D. VAN
उत्तर -(c)
उत्तर -(b)
50. क्‍प्यूटर के िािों को जोड़ने वाला िुख्य
सभकाट बो ा कौन सा ।ोिा ।ै ? 54. क्‍प्यूटर का नेटवभकगि करना

A. मदर बोर्य A. खतरोां के अिसरोां में बढ़ोिरी करतर है


B. फरदर बोर्य B. कम्प्यूटर की उियोवगतर बढ़रतर है

C. मैच बोर्य C. सूचनर अवभगमन की सांभरिनर बढ़रतर है .


D. इनमें से कोई नहीां। D. उिरोक्त सभी

उत्तर --(a) उत्तर -(c)

JOIN OUR TELEGRAM CHANNEL - https://t.me/rakshaacademy


लेखक – अभिषेक भिवारी सूचना एवं संचार प्रौद्योभिकी

55. Wi- Max भनम्नभलक्टखि िें से भकससे 59. 2G स्पेिरि िें G ीब्द के भलये प्रयुति ।ै
संबंभिि ।ै A. िोबल

A. जैि प्रौद्योवगकी B. गिनयमेंट


B. अांतररक्ष प्रौद्योवगकी C. जनरे र्न

C. वमसरइल प्रौद्योवगकी D. गूगल


D. सांचरर प्रौद्योवगकी
उत्तर -(c)
उत्तर -(d)
60. प्रकाी ।ेिु (Optical fiber) भजस
56. SIM? भसद्ांि पर काया करिा ।ै

A. Subscribers Identity Module. A. िूणय ाां तररक िररितयन


B. Subscribers Identity Machine. B. अिितयन

C. Self Identity Machine. C. प्रकीणयन


D. Self Identity Module. D. व्यवतकरण

उत्तर --(a) उत्तर --(a)

57. भसग्नल की ीक्टति कि हुये भबना नेटवका


की लंबाई बढ़ाने के भलये उपयोि करिे ।ै

A. ररिीटर
B. ररउटर

C. गेटिे
D. क्तस्वच

उत्तर -(a)

58. भकसी कंपनी के किाचाररयों ्ारा एक ।ी


स्थान िें उपयोि भकया जाने वाला अनन्य

कप से भनजी नेटवका ।ोिा


61. WWW के आभवष्कारक और प्रवािक ।ै
A. इां टरनेट
A. वबल गेट्स
B. लोकल एररयर नेटिकय
B. ली. एन. वफांसेंग
C. िरइर् एररयर नेटिकय
C. एन रसल
D. ािरय नेट
D. वटम बनयस ली
उत्तर --(b)
उत्तर -(d)

JOIN OUR TELEGRAM CHANNEL - https://t.me/rakshaacademy


लेखक – अभिषेक भिवारी सूचना एवं संचार प्रौद्योभिकी

62. याहू िूिल व MSN ।ै 66. बेब साइट अक्टस्तत्व िें आया
A. इां टरनेट सरइट A. अमेररकर

B. कम्प्यूटर िरां र् B. भररत


C. क्तस्वटजर लैण्ड में बनने िरली घन ड़ी C. जरिरन

D. इनमें से कोई नहीां। D. क्तस्वटजरलैंर्

उत्तर -(a) उत्तर --(a)

63. इं टरनेट क्या ।ै? 67. यभद भकसी ोिेन नेि के आक्टखर िें edu,
A. समुद्र में मछली िकड़ने कर जरल us, ।ै िो य। ।ै

B. बॉस्केट बॉल की अां तररय िरीय स्पधरय A. एक अां तररय िरीय सां स्थर
C. रे ल लरइनोां में रे ल के वर्ब्ोां कर वहसरब रखने B. एक गैर लरभकररी सांस्थर

िरली िर्द्वत C. ऑस्टर े वलयर की एक र्ैक्षवणक सांस्थर


D. कम्प्यूटर िर ाधरररत अांतररय िरीय सू चनरओां D. USA की एक र्ैक्षवणक सांस्थर

कर तांत्र
उत्तर --(d)
उत्तर -(d)
68. सूचना राजपथ भकसे क।िे ।ै
64. इनिें से कौन सा सचा इं जन न।ी ं ।ै A. ई-मेल

A. गूगल B. िेजर को
B. सरइां स र्रयरे र C. सेल्युलर फोन

C. अस्टर विस्टर D. इां टरनेट


D. ारकुट
उत्तर -(d)

उत्तर -(d)
69. क्‍प्यूटर वायरस ।ोिा ।ै -एक

65. Advance Research Project Agency A. फफूांद


भनम्नभलक्टखि िें से भकसके भवकास के भलए B. बैरीररयर

उिरदायी ।ै C. IC-7344
A. बेबसरइट D. सरफ्टिेयर प्रोग्ररम

B. ऑनलरइन
उत्तर -(d)
C. ई-मेल
70. Spam भकस भवषय से संबंभिि ीब्द ।ै
D. इां टरनेट
A. कम्प्यूटर
उत्तर --(d)
B. क्लब

JOIN OUR TELEGRAM CHANNEL - https://t.me/rakshaacademy


लेखक – अभिषेक भिवारी सूचना एवं संचार प्रौद्योभिकी

C. सांगीत 74. वीभ यो िेल से ।ि क्या िेज सकिे ।ै।


D. खेल A. ग्ररवफक्स

B. िीवर्यो क्तक्लप्स
उत्तर --(a)
C. िीवर्यो मैसेज
71. क्‍प्यूटर वायरस ।ै
D. ये सभी
A. ऐसर कम्प्यूटर प्रोग्ररम जो स्वयां की प्रवतवलवि
उत्तर -(d)
बनर सके

B. ऐसर कम्प्यूटर िरयरस जो मनुष्य के स्वरस्थ्य 75. भकसी पररपथ या Circuit िें इलेिरान के
को प्रभरवित करे भनयंभत्रि प्रवा। से िात्पया ।ै

C. उियुयक्त दोनो A. इलेरररवनकी


D. उियुयक्त मे से कोई नही B. इलेक्तररकल

C. मैकेवनकल
उत्तर -(a)
D. इनमें से कोई नहीां।
72. व। िोपनीय को जो कुछ प्रोग्रािों िें
उत्तर -(a)
प्रभवष्ट् प्रभिबंभिि करिा ।ै?

A. िरसिर्य 76. आिुभनक ठोस अवस्था अद्ा चालक


B. िरसिोटय इले िराभनकी का प्रादुा िाव कब हुआ ।ै

C. एां टर ी कोर् A. 1930


D. एक्सेस कोर् B. 1931

C. 1939
उत्तर (a)
D. 1940
73. क्‍प्यूटर ।ैकर ।ै
उत्तर -(a)
A. एक व्यक्तक्त जो Computer की सुरक्षर बनरये

रखतर है 77. िापायभनक उल्साजन की खोज कब हुआ


B. एक व्यक्तक्त जो व्यक्तक्तगत लरभ के दू वर्षत ।ै?

इररदोां से कम्प्यूटर सुरक्षर िरलन नहीां करतर A. 1895


है । B. 1884

C. एक व्यक्तक्त जो कम्प्यूटर के सुरवक्षत C. 1984


िररचरलन हे तु उिरदरयी होतर है । D. 1930

D. कम्प्यूटर सुधररने िरलर व्यक्तक्त


उत्तर -(b)
उत्तर -(b)

JOIN OUR TELEGRAM CHANNEL - https://t.me/rakshaacademy


लेखक – अभिषेक भिवारी सूचना एवं संचार प्रौद्योभिकी

78. िापायभनक उल्साजन भसद्ांि पर 82. ायो वाल्व िें प्रयुति कैथो भकस िािु
आिाररि भनम्नभलक्टखि िें से कौन सी का बना ।ोिा ।ै

भ वाइस ।ै A. वनकल
A. र्रयोर् िरल्व B. टां गस्टन

B. टर रयोर् िरल्व C. तरां बर


C. कैथोर्वकरण नवलकर D. थोररयम

D. उिरोक्त सभी
उत्तर -(a)

उत्तर --(d)
83. भदष्ट्कारी (Rectifier) िें भकसका प्रयोि

79. िापायभनक उत्साजन आिाररि भ वाइस िें ।ोिा ।ै?


कौन-सी िािु का प्रयोि भकया जािा ।ै A. र्रयोर्

A. थोररयम B. र्रयोर् िरल्व


B. वनकल C. a और b दोनोां

C. टां गस्टन D. a और b दोनोां नहीां।


D. तरां बर
उत्तर --(c)

उत्तर --(c)
84. ऐसे अद्ा चालक जो प्रकतभि से प्राप्त ।ोिे ।ै,

80. ायो वाल्व की खोज भकसने की ।ै उन्हे क।िे ।ै


A. जरन एिोसय फ्लेवमांग A. बरहय अर्द्य चरलक

B. थॉमस एल्वर एर्ीसन B. वनज अर्द्य चरलक


C. र्र. ली. र्ी. फररे स्ट C. N टरइि अर्द्य चरलक

D. उिरोक्त में से कोई नहीां। D. Pटरइि अर्द्य चरलक

उत्तर -(a) उत्तर -(b)

81. टर ायो बाल्ब की खोज कब हुयी ।ै 85. LED

A. 1907 A. Light emitting diode


B. 1904 B. Light emitter diode

C. 1908 C. Live emitting diode


D. 1910 D. उिरोक्त में से कोई नहीां।

उत्तर -(a) उत्तर --(a)

86. भ भजटल क्‍प्यूटर भवकभसि भकया िया ?

JOIN OUR TELEGRAM CHANNEL - https://t.me/rakshaacademy


लेखक – अभिषेक भिवारी सूचना एवं संचार प्रौद्योभिकी

A. रूस उत्तर --(c)


B. विटे न
91. वाभणक्टज्यक उपयोि के भलए उपलब्ध
C. जरिरन
कराया िया प।ला क्‍प्यूटर था?
D. यू.एस.ए.
A. MANAIC
उत्तर --(b) B. UNIVAC

C. ENIAC
87. िाइक्रोप्रोसेसर का आभवष्कार भकया था।
D. EDVAC
A. मरइक्ोसॉफ्ट
B. IBM उत्तर --(b)

C. Intel
92. इलेिराभनक क्‍प्यूटर का आभवष्कार
D. Google
भकया ।ै?

उत्तर --(c) A. चरल्सय बैबेज


B. मरकोनी
88. क्‍प्यूटर की पांचवी पीढ़ी का प्रिीक ।ै?
C. ग्ररटम बेल
A. मरइक्ोप्रोसेसर
D. इनमें से कोई नहीां
B. वमनी कम्प्यूटर
C. सुिर कम्प्यूटर उत्तर --(d)

D. मरइक्ो कम्प्यूटर

उत्तर -(c)
93. क्‍प्यूटर की प।ली पीढ़ी िें कौन सी
89. भवश्व का प।ला इलेिराभनक क्‍प्यूटर ।ै? भ वाइस का प्रयोि ।ोिा था?

A. एवनएक A. इन्टीग्रेटेर् सवकयट


B. Univae B. मरइक्ो प्रोसेसर

C. मरकय 1 C. िैक्ूम ट्यू ब


D. इनमें से कोई नहीां D. टर रां वजस्टर

उत्तर --(a) उत्तर --(c)

90. िारि िें भसभलकान वैली क्टस्थि ।ै ? 94. IBM PC कब भवकभसि हुआ था ?
A. चेन्नई A. 1990

B. वदल्ली B. 1981
C. बेंगलुरू C. 1959

D. मुिई D. 1979

JOIN OUR TELEGRAM CHANNEL - https://t.me/rakshaacademy


लेखक – अभिषेक भिवारी सूचना एवं संचार प्रौद्योभिकी

उत्तर --(b) 99. प्रथि पीढ़ी के क्‍प्यूटर िें __________का


उपयोि करके प्रोग्राभिंि की जािी थी।
95. सबसे प।ला भिनी क्‍प्यूटर था ?
A. असेिली लैंग्वेज
A. PDP-1
B. मर्ीन लैंग्वेज
B. BNIAC
C. सोसय कोर्
C. UNISAC
D. ऑब्जेिर कोर्
D. PC XT
उत्तर -(b)
उत्तर -(a)
100. IBM का पूरा नाि ।ै?
96. ितिीय पीढ़ी के क्‍प्यूटर इनके ्ारा बनाये
A. इक्तण्डयन विजनेस मर्ीन
िये?
B. इण्टरनेर्नल वबजनेस मर्ीन
A. वचस
C. इण्टरनेर्नल विजनेस मॉर्ल
B. IC
D. इनमें से कोई नहीां।
C. टर रां वजस्टर

D. िैक्ूम ट्यू ब उत्तर -(b)

उत्तर --(b) 101. क्‍प्यूटर का बुभनयादी संरचना का


भवकास ने भकया था?
97. िाइक्रो क्‍प्यूटर की ीु कआि भकस पीढ़ी
A. जरन िरन न्यूमैन
से हुयी ?
B. चरल्सय बैबेज
A. प्रथम िीढ़ी
C. ब्लेज िरस्कल
B. वद्वतीय िीढ़ी
D. गरर्य न मुरे
C. ततीय िीढ़ी

D. चतुथय िीढ़ी उत्तर -(a)

उत्तर --(d) 102. उस फ्रांभससी का क्या नाि था,


भजसने वस्त्र बनाने वाली िीीन की
98. टर ांभजस्टर क्या ।ै ?
भ जाइन से पंचका ा बनायी ?
A. सेमी कांर्रर
A. स्टीवलन
B. सैर् कण्डरर
B. जोसेफ जेकरर्य
C. बैर् कांर्रर
C. हमयन होलोररथ
D. इनमें से कोई नहीां।
D. जरन िरन न्यूमैन
उत्तर --(a)
उत्तर (b)

JOIN OUR TELEGRAM CHANNEL - https://t.me/rakshaacademy


लेखक – अभिषेक भिवारी सूचना एवं संचार प्रौद्योभिकी

103. प।ली िैकेभनकल एभ ि िीीन 107. Transistor का आभवष्कार करने


भजसिें जोड़ ।ो सकिा था? वालों िें से जान वार ीन, भवभलयि ीाकले

A. िरस्कलरइन के अलावा िीसरा व्यक्टति कौन था ?


B. जैकरर्य A. एम.ई. हॉफ

C. ओस्टीर् B. िरल्टर िेटन


D. दलॉटरइट C. जैक वकल्बी

D. ररबटय नोबी
उत्तर -(a)

उत्तर --(b)
104. चाल्सा बैबेज ्ारा भनभिाि

एनाभलभटकल इं जन ।ै? 108. IC आभवष्कार कब हुआ था?


A. इलेरररवनक A. 1960

B. इलेक्तररकल B. 1971
C. मैकेवनकल C. 1959

D. टे क्तक्नकल D. 1940

उत्तर -(c) उत्तर --(c)

105. प्रथि िणना करने वाला यंत्र ।ै? 109. Cobol का भवकास भकस पीढ़ी िें

A. वर्फरें स इां जन हुआ था ?


B. िरस्कल A. 3rd

C. केिुलेटर B. 2nd
D. अबेकस C. 1st

D. 4th
उत्तर --(d)
उत्तर --(b)
106. प।ला िाइक्रोप्रोसेसर "Intel up
4004” को भकसने बनाया था ? 110. Basic का भवकास भकसी पीढ़ी िें

A. एम. ई. हॉफ हुआ था ?


B. ररबटय नोबी A. 1st

C. जैक वकल्बी B. 3rd


D. इनमें से कोई नहीां। C. 2nd

D. 4th
उत्तर --(a)
उत्तर --(b)

JOIN OUR TELEGRAM CHANNEL - https://t.me/rakshaacademy


लेखक – अभिषेक भिवारी सूचना एवं संचार प्रौद्योभिकी

111. प्रथि व्यवसाभयक िाइक्रो उत्तर --(b)


क्‍प्यूटर कौन सा ।ै ?
115. Binary System का आभवष्कार
A. ENIAC
भकसने भकया था ?
B. ABC Computer
A. र्ॉ. ग्रेर्ष हरिर
C. IBM-I
B. जैक वकल्बी
D. Apple II
C. जरन फ्लेवमांग
उत्तर -(d) D. एर्र अगस्तर

112. वैक्यूि ट्यूब का भवकास भकस उत्तर (d)

व्यक्टति ने भकया था ?
116. AI के जनक भनम्नभलक्टखि िें से
A. एम. ई. हॉफ
कौन ।ै
B. जरन एिॉस फ्लोवनांग
A. एम. ई. हरफ
C. र्ॉ. ग्रेर्ष हरिर
B. र्र. विजयन भरस्कर
D. जैक वकल्ली
C. जरन मैकरथथ
उत्तर --(b) D. इनमें से कोई नहीां।

113. असेम्बली िाषा का आभवष्कार उत्तर -(c)

भकसने भकया था ?
117. जान िैकाथी ्ारा AI के भलए
A. एम.ई. हॉफ
खोजी िई प।ली िाषा कौन सी थी?
B. जरन फ्लोवनांग
A. LTSP
C. र्ॉ. ग्रेर्ष हरिर
B. COBOL
D. जैक वकल्ली
C. PASCAL
उत्तर -(c) D. C+

114. IBM व ।ाव ा आइकेन ्ारा उत्तर -a)

भनभिाि प।ला भवदय यु ि यांभत्रक क्‍प्यूटर था


118. Apple Company ्ारा िोबाईल
?
के भलए बनाया िया AI कौन सा ।ै?
A. IBM 1
A. Assitance
B. IBM-mark I
B. MAC
C. IBM-PCXT
C. SIRI
D. इनमें से कोई नहीां।
D. Watson

JOIN OUR TELEGRAM CHANNEL - https://t.me/rakshaacademy


लेखक – अभिषेक भिवारी सूचना एवं संचार प्रौद्योभिकी

उत्तर --(c) B. 1992


C. 2000
119. भनम्नभलक्टखि िें कौन से AI के
D. 1980
उदा।रण ।ै

A. Machine Learning उत्तर --(d)


B. Automatin
123. नीभि आयोि ने भकसके साथ
C. Robotics
भिलकर ICTAT की स्थापना की
D. उिरोक्त सभी
A. Intel
उत्तर --(d) B. Tata Institute for fundamental

Research
120. IBM ्ारा भनभिाि भवश्व का प।ला
C. IBM
Voice Command आिाररि िीीन कौन
D. a और b दोनोां
सी ।ै जो िणिीय आप्रेीन परकािा कर
सकिी ।ै। उत्तर --(d)

A. Watson
124. ICTAI का िुख्यालय क।ां पर ।ै
B. Deep Blue
A. वदल्ली
C. Shoe Box
B. मुिई
D. इनमें से कोई नहीां।
C. बैंगलौर
उत्तर --(c) D. है दररबरद

121. Arend Hintze ्ारा पररिाभषि उत्तर -(c)


AI के प्रकार िें से कौन सा AI का प्रकार
125. भनम्नभलक्टखि िें से AI के भलये क्या
न।ी ं ।ै
सत्य न।ी ं ।ै
A. सरमरन्य AI
(1) Machine Learning AI का ।ी उदा।रण ।ै।
B. िूणय प्रवतवक्यरत्मक
C. ात्म चेतन (2) िक्टस्तष्क चेिन AI का प्रकार न।ी ं ।ै।
D. मक्तस्तष्क वसर्द्रां त
(3) ऐसे AI जो भकसी भवीेष प्रकार के टास्क के
उत्तर --(a) भलये बने ।ोिे ।ड उन्हें सािान्य AI क।िे ।ड

122. जापान ने Fifth Generation A. केिल 1


भकस वषा Start भकया था? B. केिल 2
A. 1981 C. केिल 3

JOIN OUR TELEGRAM CHANNEL - https://t.me/rakshaacademy


लेखक – अभिषेक भिवारी सूचना एवं संचार प्रौद्योभिकी

D. 2 और 3 दोनोां B. 1956
C. 1950
उत्तर --(d)
D. 1970
126. ऐसे AI जो केवल एक टास्क को
उत्तर --(b)
करने के भलये बनाये जािे ।ड उन्हें क।िे ।ै

A. कमजोर AI 130. वचुाअल एजेण्टयस (Agents) SIRI


B. सरमरन्य AI को कब Introduce भकया िया था

C. िूणय प्रवतवक्यरत्मक A. 2018


D. सीवमत स्मवत B. 2019

C. 2016
उत्तर --(a)
D. 1992
127. किजोर AI का उदा।रण
उत्तर --(c)
भनम्नभलक्टखि िें से कौन सा ।ै -

A. SIRI 131. LISP का पूरा नाि क्या ।ै


B. Cortana A. List Processing

C. Google Assistance B. List Problem solving


D. उिरोक्त सभी C. Least Processing

D. Line Processing
उत्तर --(d)
उत्तर -(a)
128. IBM ्ारा भनभिाि सुपर क्‍प्यूटर
________ भजसने िैरी कास्परोव को Chess 132. नीभि आयोि ्ारा AI के भलये

िें िें ।राया था? जारी की ियी ररपोट कौन सी ।ै


A. Watson A. AI for Good

B. Deep Blue B. Al for India


C. Shoe box. C. AI for All

D. Mark - I D. इनमें से कोई नहीां।

उत्तर --(b) उत्तर -(c)

129. AI को पररिाभषि करने वाले जान 133. िेलंिाना ने भकस वषा को AI वषा के

िैकाथी, भजन्होने AI को पररिाभषि भकया कप िें बनाने का कैसला भलया ।ै


था A. 2020

A. 1981 B. 2021

JOIN OUR TELEGRAM CHANNEL - https://t.me/rakshaacademy


लेखक – अभिषेक भिवारी सूचना एवं संचार प्रौद्योभिकी

C. 2019 137. PC Program के ीु क करने वाला


D. 2018 भवश्व भवद्यालय ।ै

A. BHU
उत्तर --(a)
B. Delhi University
134. नीभि आयोि ्ारा AI के भलये
C. Davv
जारी की ियी ररपोटा पर प।ला ।स्तािर
D. इनमें से कोई नहीां।
करने वाला राज्य ।ै
उत्तर -(a)
A. म.प्र.
B. महरररिर 138. CBSE भकस अं िराा ष्ट्रीय कंपनी के

C. केरल साथ भिलकर छात्रों को AI सीखाने के


D. तेलांगरनर भलये सेण्टर की स्थापना करे िा।

A. Intel
उत्तर -(d)
B. IBM
135. "Al for Excellence" Center
C. Microsoft
स्थाभपि करने वाली संस्था ।ै
D. इनमें से कोई नहीां।
A. नीवत ायोग
उत्तर -(a)
B. NIC

C. CBSE 139. RAISE 2020 िें E का ििलब ।ै


D. IIT BHU A. Empowerment

B. Encourage
उत्तर -(b)
C. Education
136. िेलंिाना भकसके स।योि से
D. इनमें से कोई नहीां।
।ैदराबाद िें एक Center स्थाभपि करे िा
उत्तर -(a)
जो AI िथा Robotics िें ीोि काया
करे िा। 140. नीभि आयोि ने भकसकी अध्यििा

A. नीवत ायोग िें AI कायाक्रि की कपरे खा िैयार करने


B. Intel के भलये एक सभिभि का िठन भकया

C. Nasscom A. अरूण जेटली


D. IBM B. के सुिमण्डयम के

C. रविर्ांकर प्रसरद
उत्तर -(c)
D. ररजीि कुमरर

JOIN OUR TELEGRAM CHANNEL - https://t.me/rakshaacademy


लेखक – अभिषेक भिवारी सूचना एवं संचार प्रौद्योभिकी

उत्तर (d) C. IIT गुिरहरटी


D. IIT – BHU
141. नीभि आयोि ्ारा जारी “AI for
All” Reoprt कब जारी की ियी उत्तर -(a)

A. Feb-2018
145. CBSE ने भकस किा से AI पढ़ाने
B. Mar-2018
के भलये एक भवषय का कैसला भलया ।ै
C. जुलरई 2018 -
A. 10th
D. जून-2018
B. 8th
उत्तर -(d) C. 11th

142. BHU ने AI िें नया PG Program


चालू करे िा, इसके भलए भकसकी

अध्यििा िें एक सभिभि का भनिाारण


भकया ।ै

A. प्रो. ान्नद मोहन


B. प्रो. प्रसरद

C. प्रो. कुलदीि वसांह


D. इनमें से कोई नहीां।

उत्तर -(a)

143. RAISE 2020 कब आयोभजि


D. 12th
भकया जायेिा

A. 5-9 Oct 2020 उत्तर -(c)

B. 5-9 Sep 2020


146. AI Powered chat bot का
C. 20-22 Oct 2020
इस्तेिाल करने वाला प।ला TV कायाक्रि
D. इनमें से कोई नहीां।
A. कौन बनेगर करोर्िवत

उत्तर --(a) B. सत्यमेि जयते


C. मैं कुछ भी कर सकती हूँ
144. दे ी का प।ला संस्थान भजसने AI
D. इनमें से कोई नहीां।
के भलए अलि भ पाटा िेंट की स्थापना की
A. IIT Delhi उत्तर -(c)
B. IIT रूड़की

JOIN OUR TELEGRAM CHANNEL - https://t.me/rakshaacademy


लेखक – अभिषेक भिवारी सूचना एवं संचार प्रौद्योभिकी

147. नीभि आयोि ने Intel और Tata C. Deep Blue


Institute of Fundamental Research D. Mark-I

के स।योि से कौन सा सेन्टर स्थाभपि


उत्तर -(a)
भकया
151. AI युति ISRO का रोबोट भजसका
A. ICAR
प्रयोि ििनयान िें भकया जा र।ा ।ै
B. ICAI
A. रक्षरवमत्र
C. ICTAI
B. व्योम वमत्र
D. ICARI
C. भररत वमत्र
उत्तर -(c)
D. इनमें से कोई नहीां।
148. DRDO ्ारा
उत्तर --(b)
स्थाभपि___________जो रिा िेत्र िें Al और
152. IBM की प्रश्न उिर वाली िीीन
Robotics के कील्ड िें ररसचा करे िा।
कौन सी ।ै -
A. ICAR
A. Watson
B. ICTAI
B. Deep Blue
C. Center for excellence
C. Mark I
D. इनमें से कोई नहीां।
D. PC-XT
उत्तर -(d)
उत्तर -(a)
149. ALEXA भकस कम्पनी का Virtual
153. Microsoft company का
Agents ।ै
virtual Agents AT कौन सा ।ै
A. IBM
A. Alexa
B. AMAZON
B. SIRI
C. Apple
C. Cortana
D. इनमें से कोई नहीां।
D. Assitance
उत्तर -(b)
उत्तर -(c)
150. AI युति िारि का प।ला रोबोट
154. IIT भकस िल्टीनेीनल कम्पनी के
भजसे बडभकंि सेल के भलये उपयोि भकया
स।योि से AI को बढ़ावा दे ने ररसचा लैब
जा र।ा ।ै
की स्थापना करें िे
A. लक्ष्मी
A. मरइक्ोसॉफ्ट
B. Watson
JOIN OUR TELEGRAM CHANNEL - https://t.me/rakshaacademy
लेखक – अभिषेक भिवारी सूचना एवं संचार प्रौद्योभिकी

B. Intel के सुिमण्डयम A. 2006


C. अमेजन इां टरनेट सवियसेज Pvt.Ltd. B. 2004

D. Face book.. C. 2010


D. 2020
उत्तर -(c)
उत्तर -(b)
155. "िड कुछ िी कर सकिी हूाँ एक
टी.वी कायाक्रि भजसिे AI युति Chatbot

का प्रयोि भकया िया ।ै व। Chatbot ।ै -


159. ऐसा AI जो िानव की िर।
A. र्र. स्नेहर Chatbot
संवेदनीील ।ो, आत्मचेिना ।ो भजसके
B. वमत्र Chatbot
अंदर कीभलग्स ।ो, क।िे ।ै
C. मवहलर Chatbot
A. िूणय प्रवतवक्यरत्मक
D. इनमें से कोई नहीां।
B. मक्तस्तष्क वसर्द्रां त
उत्तर --(a) C. ात्मचेतन

D. सरमरन्य AI
156. AI के भलये बनायी ियी प्रोग्राभिंि

िाषा कौन सी ।ै उत्तर --(b)


A. LISP
160. NLP िे L का ििलब ।ै
B. PASCAL
A. Language
C. COBOL
B. List
D. ALGOL
C. Least
उत्तर -(a) D. Level

157. जान िैकाथी ने AI ीब्द का प्रयोि उत्तर -(a)

प।ली बार कब भकया िया था


161. क्‍प्यूटर के काया करने का
A. 1956
भसद्ांि ।ै
B. 1950
A. Input
C. 1960
B. Output
D. 1959
C. Process

उत्तर -(a) D. उियुयक्त में से सभी

158. Honda Asimo एक पसानल उत्तर -(d)

रोबोट का आभवष्कार हुआ था

JOIN OUR TELEGRAM CHANNEL - https://t.me/rakshaacademy


लेखक – अभिषेक भिवारी सूचना एवं संचार प्रौद्योभिकी

162. भनम्नभलक्टखि िें से कौन सा उत्पाद D. हरर्य र्ररइि


पेभटयि ब्रां नाि बेचा जािा ।ै -
उत्तर --(a)
A. मोबरइल वचि
166. भजसे क्‍प्यूटर का ब्रेन िी क।िे
B. कम्प्यूटर वचि
।ड, ाटा प्रोसेभसंि करिा ।ै
C. कम्प्यूटर
A. मदर बोर्य
D. मरइक्ोप्रोसेसर
B. रै म
उत्तर -(d)
C. मेमोरी
D. सी.िी.यू

उत्तर --(d)

163. क्‍प्यूटर के सिी िाभकाक िथा 167. िीीन िें किाण्ड को कैरी करने
अंकिभणिीय पररकलन भकये जािे ।ड? की भवभि को क्या क।िे ।ड

A. प्रणरली बोर्य द्वररर A. एक्तिक्ूवटां ग


B. केन्द्रीय वनयांत्रक Unit द्वररर B. वर्इमूवलांग

C. सेन्टरल प्रोसेवसां ग यूवनट द्वररर C. प्रोसेवसांग


D. मदरबोर्य द्वररर D. इनमें से कोई नहीां।

उत्तर --(c) उत्तर --(a)

164. प्रोसेसर िें एक कन्टर ोल यूभनट और 168. ाटा को स्थानान्तररि करने वाली
एक _______ीाभिल ।ोिा ।ै? बस को क्या क।िे ।ड

A. कन्टर ोल यूवनट A. र्रटर बस


B. अथयमेवटक एिां लरवजक यूवनट B. कांटर ोल बस

C. रे म C. ाां तररक बस
D. रोम D. एर्रेस बस

उत्तर --(b) उत्तर -(a)

165. क्‍प्यूटर के िािों को जोड़ने वाला 169. CPU का क्या काया ।ै


िुख्य सभकाट बो ा कौन सा ।ोिा ।ै A. वर्लीट करनर

A. मदर बोर्य B. प्रोसेवसांग करनर


B. फरदर बोर्य C. करप्ट करनर

C. मैच बोर्य D. एवर्ट करनर

JOIN OUR TELEGRAM CHANNEL - https://t.me/rakshaacademy


लेखक – अभिषेक भिवारी सूचना एवं संचार प्रौद्योभिकी

उत्तर --(b) उत्तर --(a)

170. क्‍प्यूटर का प्राथभिक काया ।ै 174. ाटा पर अंकिणिीय और


ाटा का िें बदलना। िाभकाक भक्रयायें करिा ।ै

A. इनफॉमेर्न A. CPU
B. टे बल्स B. CU

C. फरइल्स C. ALU
D. ग्ररफ्स D. मेमोरी

उत्तर -(a) उत्तर -(c)

171. िेिोरी के अनुक्रि िें सबसे ऊंचा 175. ALU का काया ।ै


स्थान भकसका ।ोिा ।ै A. गणतीय वक्यरओां ि तकय की वक्यर को करनर

A. प्ररथवमक मेमोरी B. चलवचत्रोां कर चलरनर


B. रवजस्टर C. गरने चलरनर

C. वद्वतीयक D. लेटर वलखनर


D. CPU
उत्तर --(a)
उत्तर -(b)
176. CPU िें उपक्टस्थि आं िररक िेिोरी

।ै
A. रवजस्टसय कर समूह
172. प्राथभिक िेिोरी जिा करिी ।ै
B. मरइक्ोप्रोसेसर
A. केिल र्रटर
C. ALU र्रटर स्टोर
B. केिल प्रोग्ररम
D. बस
C. केिल िररणरम

D. उिरोक्त सभी उत्तर --(a)

उत्तर -(d) 177. आउटपुट भ वाइस का प्रयोि


करके आप __________ कर सकिे ।ड
173. िाइक्रो प्रोसेसर को _______ िी
A. र्रटर इनिुट
क।िे ।ड
B. र्रटर स्कैन
A. वचि
C. र्रटर स्टोर
B. IC
D. र्रटर व्यू यर वप्रांट
C. रूल

D. टर रां वजस्टर उत्तर --(d)

JOIN OUR TELEGRAM CHANNEL - https://t.me/rakshaacademy


लेखक – अभिषेक भिवारी सूचना एवं संचार प्रौद्योभिकी

178. भबटय स का एक सिू। जो क्‍प्यूटर D. CPU


को भकसी भवीेष काया करने को क।िा ।ै,
उत्तर --(c)
क।लािा ।ै
182. इन्स्ट्रक्शन का सिू।, जो क्‍प्यूटर
A. इन्स्ट्रक्शन कोर्
को भनदे भीाि करिा ।ै क।लािा ।ै -
B. एक्ुमुलेटर
A. प्रोग्ररम
C. मरइक्ो ािरे र्न
B. हरर्य िेयर
D. रवजस्टर
C. लरवजक
उत्तर --(a)
D. CPU

179. व। िेिोरी इकाई जो सीिे CPU से


उत्तर -(a)
कम्यूभनकेट करिी ।ै क।लािी ।ै
183. िीीन साइभकल को कौन भनयंभत्रि
A. मेन मेमोरी
करिा ।ै।
B. िेन र्ररइि
A. कन्टर ोल यूवनट
C. बरह्य मेमोरी
B. मेमोरी यूवनट
D. ये सभी
C. लरवजक यूवनट
उत्तर --(a)
D. इनमें से कोई नहीां।

180. क्‍प्यूटर पर क।ां पर ाटा को


उत्तर -(a)
जोड़ा या िुलना की जािी ।ै
184. क्‍प्यूटर िंत्र िें “बस” से क्या
A. हरर्य वर्स्क
ििलब ।ै
B. फ्लरिी वर्स्क
A. अांकगवणतीय तकय इकरई
C. CPU
B. सरमरनरिर र्रटर िथ
D. मेमोरी वचि
C. एर्े प्टर करर्य
उत्तर -(c)
D. इनमें से कोई नहीां।

उत्तर -b)

181. भसस्टि यूभनट के िेन सभकाट बो ा


185. CPU िथा पेररकेरल के िध्य की
को क।िे ।ड
कम्यूभनकेीन लाइन को क।िे ।ै।
A. कम्प्यूटर प्रोग्ररम
A. बस
B. कन्टर ोल यूवनट
B. लरइन
C. मदर बोर्य
C. मीवर्यर

JOIN OUR TELEGRAM CHANNEL - https://t.me/rakshaacademy


लेखक – अभिषेक भिवारी सूचना एवं संचार प्रौद्योभिकी

D. ये सभी D. सेम्यूर के

उत्तर --(a) उत्तर -(b)

186. भ भजटल घड़ी िें भकस प्रकार का


क्‍प्यूटर पाया जािा ।ै?
190. भनम्नभलक्टखि िें से कौन सा
A. मेनिेम
क्‍प्यूटर ।ा ा वेयर न।ी ।ै?
B. सुिर कम्प्यूटर
A. मरउस
C. नोट कम्प्यूटर
B. वप्रांटर
D. एिेटेर् कम्प्यूटर
C. मरनीटर

उत्तर -(d) D. एक्सेल

187. भनम्नभलक्टखि िें से कौन सबसे बड़े उत्तर -(d)


सबसे ि।ंिे और सबसे िेज क्‍प्यूटर ।ै?
191. क्‍प्यूटर के भलये IC भनम्नभलक्टखि
A. िसयनल कम्प्यूटर
िें से भकस पदाथा की बनी ।ोिी ।ै?
B. सुिर कम्प्यूटर
A. लीफ
C. लैिटरि
B. क्ोवमयम
D. नोटबुक
C. जमेवनयम

उत्तर -(b) D. वसवलकरन

188. भनम्न भलक्टखि िें से कौन सा काया उत्तर --(d)


क्‍प्यूटर ्ारा न।ी ं भकया जािा ।ै?
192. दे ी का प्रथि क्‍प्यूटर सािर
A. कम्प्यूवटां ग
भजला ।ै?
B. प्रसांस्करण
A. अनरय कुलम
C. समझनर
B. विल्लुिुरम
D. ाउट बुवटां ग
C. भीरूिल्लूर
उत्तर -(c) D. मल्लिुरम

189. िारि िें सुपर क्‍प्यूटर के भपिा उत्तर -(d)


के कप िें भकसे जाना जािा ।ै?
193. िारि का प।ला क्‍प्यूटर क।ां
A. रघन ुनरथ मरर्ेलकर
स्थाभपि भकया िया था?
B. विजयन भरस्करन
A. वदल्ली
C. नांदन नीलेिणथ
B. बैंगलौर
JOIN OUR TELEGRAM CHANNEL - https://t.me/rakshaacademy
लेखक – अभिषेक भिवारी सूचना एवं संचार प्रौद्योभिकी

C. कलकिर B. IBM-PC
D. बनयिुर C. (C) Cray

D. ब्लू जीन
उत्तर --(c)

उत्तर --(d)
194. क्‍प्यूटर सािरिा भदवस िनाया

जािा ।ै? 198. भवश्व के प।ले सुपर क्‍प्यूटर का


A. 1 December नाि था ?

B. 2 December A. िरम 8000


C. 1 January B. सवमट

D. 12 January C. क्े – I
D. इनमें से कोई नहीां।
उत्तर -(b)

उत्तर -(c)
195. भवश्व िें सवााभिक क्‍प्यूटर वाला

दे ी ।ै? 199. िेनफ्रेि क्‍प्यूटर का व। टभिानल


A. चीन भजसके िाध्यि से Data को Access भकया

B. भररत जािा ।ै, क।िे ।ै


C. जरिरन A. Terminal

D. स. ररज्य अमेररकर B. Dumb


C. Node
उत्तर -(d
D. इनमें से कोई नहीां।
196. क्‍प्यूटर प्रोसेस ्ारा…….. को
उत्तर --(c)
सूचना (Information) िें पररवभिाि
करिा ।ै? 200. ANUPAM सुपर क्‍प्यूटर िारि

A. नांबर की भकस सरकारी संस्था या इं स्टीट्यूट का


B. Data (र्रटर) सुपर क्‍प्यूटर ।ै?

C. Input A. DRDO
D. प्रोसेसर B. BARC

C. SBI
उत्तर -(b)
D. RBI
197. भवश्व का सबसे िेज सुपर क्‍प्यूटर
उत्तर -(b)
IBM _________का ।ै?

A. एकर

JOIN OUR TELEGRAM CHANNEL - https://t.me/rakshaacademy


लेखक – अभिषेक भिवारी सूचना एवं संचार प्रौद्योभिकी

201. िाइक्रो क्‍प्यूटर का भवकास करने B. सुिर कम्प्यूटर


वाली प।ली कम्पनी कौन थी? C. वर्वजटल कम्प्यूटर

A. IBM D. नोटबुक कम्प्यूटर


B. HP
उत्तर (b)
C. Dell
205. June 2020 के आिार पर भवश्व
D. Sony
का सबसे िेज सुपर क्‍प्यूटर ।ै?
उत्तर -(a)
a. प्रत्युर्ष
202. एक क्‍प्यूटर भजसे पोटे बल b. वतयरन्िे ॥

क्‍प्यूटर न।ी ं िाना जािा ।ै व। ? c. Summit


A. वमनी कम्प्यूटर d. वमवहर

B. लैिटॉि
उत्तर -(c)
C. टै बलेट
206. परि ब्र।ि जो िारि का एक सुपर
D. ये सभी
क्‍प्यूटर ।ै, क।ां स्थाभपि भकया िया ।ै ?
उत्तर -(a)
A. IISER PUNE
203. क्‍प्यूटर B. IISER बैंगलौर

(1) आं कड़ों को स्टोर करने वाली एक सिि C. IIT गुिरहरटी


युक्टति ।ै। D. IIT दे हली

(2) पूणा िोपनीयिा बनाये रखने िें सिि ।ै।


उत्तर -(b)
(3) किी-किी वायरस ्ारा प्रिाभवि ।ोिा ।ै।
207. परि ब्र।ि भकस वषा बनकर िैयार
उपरोति िें से कौन सा कथन सत्य ।ड।
हुआ था ?

A. 1 ि 2 A. 2003
B. 1 2 ि 3 B. 2020

C. 1, 3 C. 1991
D. ये सभी D. 2004

उत्तर -(c) उत्तर -(b)

204. भनम्न िें से अलि को प।चाभनये, 208. C-DAC िें A का ििलब ।ै


बाभक िीन एक प्रकार से सिान ।ै? A. Article

A. मरइक्ो कम्प्यूटर B. Automation

JOIN OUR TELEGRAM CHANNEL - https://t.me/rakshaacademy


लेखक – अभिषेक भिवारी सूचना एवं संचार प्रौद्योभिकी

C. Automatic उत्तर -(c)


D. Advance
213. िेनफ्रेि क्‍प्यूटर िें भकस प्रकार
उत्तर -(d) के ऑपरे भटं ि भसस्टि का प्रयोि ।ोिा ।ै ?

A. बैच प्रोसेवसांग
209. PACE सुपर क्‍प्यूटर क।ा
B. िैरेलल प्रोसेवसांग
स्थाभपि भकया िया ।ै ?
C. टरइम र्ेयररां ग
A. BARC
D. इनमें से कोई नहीां
B. DRDO
C. ISRO उत्तर -(c)

D. SBI
214. C-DAC के ारे िर जनरल कौन
उत्तर (b) ।ै ?

A. प्रो रजत मूनर


210. C-DAC का िुख्यालय क।ां ।ै ?

A. िुणे
B. वदल्ली

C. मुिई
D. कलकिर

उत्तर -(a)

211. PeTa Flops बराबर ।ोिा ।ै ?


A. 1012 F

B. 1010 F
C. 1015 F

D. 103 F B. अय्यर

C. र्ॉ. विजयन
उत्तर -(c)
D. र्ॉ. हे मांत दरबररी
212. भिनी क्‍प्यूटर का आभवष्कार
उत्तर -(a)
DEC ने कब भकया था ?
A. 1966 215.. दे ी का प।ला पेटाफ्लाप्स सुपर क्‍प्यूटर

B. 1942 कौन सा ।ै
C. 1965
A. र्रस्त्र I
D. 1980
B. िरम युिर II
JOIN OUR TELEGRAM CHANNEL - https://t.me/rakshaacademy
लेखक – अभिषेक भिवारी सूचना एवं संचार प्रौद्योभिकी

C. िरम िह्म C. सेलवनयम


D. िरम ईर्रन D. इनमें से कोई नहीां

उत्तर -(a) उत्तर -(a)

June 2020 Ranking के अनुसार िारि का 218. कोटाग्राकी िें सोलर सेल का

सबसे िेज सुिार क्‍प्यूटर ।ै? प्रयोि भकस प्रकार भकया जािा ।ै
A. मरिक
A. वमवहर
B. सांसुचन
B. िरम िह्म
C. कांर्क्तरि
C. िरम ईर्रन
D. इनमें से कोई नहीां
D. प्रत्युर्ष
उत्तर -(a)
उत्तर -(d)
219. भनम्नभलक्टखि िें से भकस ायो का
215. चोर सूचक घण्टी िें भकसका प्रयोि
प्रयोि वोल्टे ज भनयंत्रक के कप िें उपयोि
भकया जािा ।ै?
करिे ।ड -
A. Photo diode
A. जेनर र्रयोर्
B. LED
B. फोटो र्रयोर्
C. Diode
C. र्रयोर्
D. Zener diode
D. टर रां वजस्टर
उत्तर -(b)
उत्तर -(a)
216. Amplifier िें कौन सी भ वाईस
220. टर ांभजस्टर का आभवष्कार कब हुआ
का प्रयोि की जािी ।ै
।ै
A. टर रां वजस्टर
A. 1950
B. टर रयोर् िरल्व
B. 1952
C. a और b दोनोां
C. 1948
D. इनमें से कोई नहीां
D. 1949
उत्तर -(c)
उत्तर -(c)
217. अवरति भकरणों के के ससूचन िें
221. लेजर भसद्ांि का भवकास करने
भकस पदाथा का प्रयोि भकया जािा ।ै
वाले वैज्ञाभनक कौन ।ै
A. इां वर्यन एण्टीमनी
A. र्ैलो
B. लेर् सल्फरइर्
JOIN OUR TELEGRAM CHANNEL - https://t.me/rakshaacademy
लेखक – अभिषेक भिवारी सूचना एवं संचार प्रौद्योभिकी

B. प्रो चरल्सय सी टरउन्स C. 1987


C. a और b दोनोां D. उिरोक्त में से कोई नहीां

D. a और b दोनोां नहीां
उत्तर -(a)

उत्तर -(c)
226. लेजर अद्ा चालक का आभवष्कार

222. लेजर के जनक क।े जािे ।ड - कब हुआ


A. सैमुअल मरसय A. 1965

B. प्रो चरल्सय सी टरउन्स B. 1961


C. वथयोर्र मैमेन C. 1966

D. र्ैलो D. 1967

उत्तर -(c) उत्तर -(a)

223. प।ला िैस लेजर कब अक्टस्तत्व िें 227. िेसर का अभवष्कार भकसने भकया

आया A. गीगरर
A. 1960 B. गोरर्न

B. 1961 C. टरउन्स
C. 1962 D. उिरोक्त सभी

D. 1984
उत्तर -(d)
उत्तर -(a)
228. RADAR -
224. िारि िें लेजर प्रौद्योभिकी के जन A. Radio detect and Range.

कौन क।े जािे ।ै B. Radio detection and Ranging


A. र्ॉ. होमी जहरां गीर भरभर C. Ray detection and Ranging

B. ररजर रमन्नर D. उिरोक्त में से कोई नही


C. जनदीर्ष चन्द्र बसु
उत्तर -(b)
D. इनमें से कोई नहीां
229. टर ांभजस्टर का आभवष्कार भकसने
उत्तर -(a)
भकया
225. राजा रिन्ना प्रौद्योभिकी संस्थान की A. जरन बररर्ीन

स्थापना कब हुई B. िरल्टर बैटरेन


A. 1984 C. विवलयम र्रकली

B. 1961 D. इन सभी ने।

JOIN OUR TELEGRAM CHANNEL - https://t.me/rakshaacademy


लेखक – अभिषेक भिवारी सूचना एवं संचार प्रौद्योभिकी

उत्तर -(d) उत्तर -(a)

230. प्रौद्योभिक भवकास पररषदय का 234. आिुभनक रोबोभटक्स के भपिािा।


िठन कब हुआ। भकसे क।िे ।ड

A. 1971 A. ईर्षरक ावर्षमोि


B. 1972 B. कररे ल करिेक

C. 1973 C. जोसेफ एां जेल बगयर


D. 1974 D. जरजय र्ी िोल

उत्तर -(c) उत्तर -(c

231. रोबोभटक्स के अंििाि भनम्नभलक्टखि 235. दु भनया का प।ला एन्ड्राय रोबोट


िें से कौन सा िेत्र आिा ।ै कौन सा ।ै

A. मैकेवनकल A. इबवसनर रोबोट


B. इलेक्तररकल B. कोटोमेट

C. इलेरररवनक C. िरबोट
D. उिरोक्त सभी D. उिरोक्त में से कोई नहीां

उत्तर -(d) उत्तर -(a)

232. रोबोट का सवाप्रथि प्रयोि एवं 236. प्रथि औद्योभिक रोबोट का


प्रकाीन भकस व्यक्टति ने भकया। आभवष्कार भकसने भकया ।ै

A. कररे ल करिेक A. जोसेफ एां जेल बगयन


B. ईर्षरक ावर्षमोि B. जरजय सी र्े िोल

C. जोसेफ कप्पेक C. a और b दोनोां


D. इनमें से कोई नहीां D. A और b दोनोां नहीां

उत्तर -(a) उत्तर -(c)

233. रोबोभटक्स का भसद्ांि

भनम्नभलक्टखि िें भकस व्यक्टति ने भदया ।ै


237. सोभकया जो भक भवश्व का
A. ईजरक ावर्मोि
सवााभिक उन्नि रोबोट ।ै क।ां का रोबोट
B. कररे ल करिेक
।ै
C. जोसेफ एां जेल बगयर
A. सांयुक्त ररज्य अमेररकर
D. उिरोक्त में से कोई नहीां
B. जरिरन

JOIN OUR TELEGRAM CHANNEL - https://t.me/rakshaacademy


लेखक – अभिषेक भिवारी सूचना एवं संचार प्रौद्योभिकी

C. हरां गकरां ग B. FINSAS


D. चीन C. इब वसांनर

D. सोवफयर
उत्तर -(c)

उत्तर -(b)
238. दु भनया का प।ला सजीव रोबोटय स

भकस दे ी ने बनाया ।ै 242. CAIR िें । का ििलब क्या ।ै


A. हरां गकरां ग A. Industrial.

B. भररत B. Intelligence
C. जरिरन C. Innovation

D. सांयुक्त ररज्य अमेररकर D. उिरोक्त में से कोई नहीां

उत्तर (d) उत्तर -(b)

239. भनम्नभलक्टखि िें से कौन सा एक 243. CAIR का िुख्यालय क।ां ।ै

पभ।ये वाला रोबोट ।ै A. बैंगलौर


A. बरल्ड रोबोट B. वदल्ली

B. इब वसनर रोबोट C. है दररबरद


C. जन बोट् स D. उिरोक्त में से कोई नही

D. उिरोक्त में से कोई नही


उत्तर -(a)
उत्तर -(a)
244. प्रथि िारिीय औद्योभिक कंपनी
240. ी.एन.ए पर आिाररि रोबोटय स भजसने औद्योभिक रोबोट का भनिााण

क।लािे ।ै भकया
A. जेनबरट् स A. भररतीय इलेरररवनक वलवमटे र्

B. चैटबोट् स B. CAIR
C. न्यूबोट् स C. DRDO

D. उिरोक्त में से कोई नहीां D. उिरोक्त में से कोई नहीां

उत्तर -(a) उत्तर -(a)

241. DRDO ने………. नािक रोबोट 245. प।ला िारिीय औद्योभिक रोबोट

का आभवष्कार भकया ।ै जो बा कदी सुरंिों कौन सा ।ै।


का पिा लिा सकिा ।ै A. BRABO

A. दक्ष B. चतुरोबोट

JOIN OUR TELEGRAM CHANNEL - https://t.me/rakshaacademy


लेखक – अभिषेक भिवारी सूचना एवं संचार प्रौद्योभिकी

C. प्रज्ञरन D. इनमें से कोई नहीां


D. गरूड़
उत्तर -(c)
उत्तर -(a)
250. भकसकी स।ायिा से इलेिराभनक
246. CAIR ्ारा भवकभसि रोबोट कौन िाध्यि से व्यापार भकया जािा ।ै

सा ।ै A. इां टरनेट
A. रोबोसेन B. Telephone

B. स्नेक C. Pager
C. a और b दोनोां D. केवबल

D. A और b दोनोां नहीां
उत्तर -(a)
उत्तर -(c)
251. E-Commerce का िात्पया ।ै
247. िारिीय रोबोभटक सिुदाय की A. इां टरनेट कर व्यरिरररक लेखर जोखर करनर

स्थापना कब हुयी B. इां क्तिर्ष में लेखर जोखर करनर


A. 10 July 2011 C. इां टरनेट में एकरउां ट सीखनर

B. 11 Aug 2011 D. उिरोक्त सभी


C. 9 July 2011
उत्तर -275-(a)
D. 11 July 2012
252. __________ ने 1979 िें प।ले
उत्तर -271-(a)
Online Shopping System को
248. CAIR के भनदे ीक ।ै demonstrate भकया था ।

A. Dr. UK Singh A. Michael Aldrish


B. Dr. P.K. Upadhayay B. Wilhelm stinit 2

C. Dr.Shdini Chaturvedi C. Tim Berner lee


D. Dr subrat rashit D. इनमें से कोई नहीां

उत्तर -(d ) उत्तर -(a)

249. E-Commerce िें E भनम्नभलक्टखि िें 253. विािान िें दु भनया िें सबसे ज्यादा
से भकसके भलये आिा ।ै Revenue Generate करने वाली e -

A. Economics Commerce कंपनी ।ै


B. Essential A. Alibaba.com

C. Electronic B. Amazon

JOIN OUR TELEGRAM CHANNEL - https://t.me/rakshaacademy


लेखक – अभिषेक भिवारी सूचना एवं संचार प्रौद्योभिकी

C. Jingdong B. B2C
D. E bay C. B2B

D. C2B
उत्तर -(b)

उत्तर -(c)
254. CMS -

A. Core Managing System 258. Online पुराने सािान की खरीद-


B. Content management System करोख्त करने वाली कंपनी OLX

C. Core management System भनम्नभलक्टखि िें से भकस Segment के


D. Cashe management System अंििाि आिी ।ै

A. C2C
उत्तर -(d)
B. C2B
255. Amazon भकस Segment के
C. B2B
अंििाि आिी ।ै
D. उिरोक्त में से कोई नही
A. B2B
उत्तर -(a)
B. B2C

C. C2B 259. भनम्नभलक्टखि िें से कौन सी कंपनी


D. B2G इं न्वेटरी के अंििाि आिी ।ै

A. अलीबरबर
उत्तर --(b)
B. अमेजन
256. E-Commerce स।ी िायने िें अथा
C. विलमकरटय
।ै
D. स्नैिर्ील
A. Business करनर
उत्तर -(a)
B. Goods और Services को Sell करनर

C. Electronically businees करनर 260. ीासन ्ारा भकसी योजना के भलये


D. इनमें से कोई नहीां लोिो (Logo), टै िलाइन इत्याभद के भलये

प्रभियोभििा आयोभजि करना भकस


उत्तर -(c)
सेििेंट के अंििाि आिा ।ै
257. E-Commerce का व। प्रकार
A. C2C
भजसिं एक Company internet के
B. B2A
िाध्यि से भकसी दू सरी कंपनी को अपने
C. C2A
प्रो ि सेल करिी ।ै
D. उिरोक्त मे से कोई नही
A. B2C

JOIN OUR TELEGRAM CHANNEL - https://t.me/rakshaacademy


लेखक – अभिषेक भिवारी सूचना एवं संचार प्रौद्योभिकी

उत्तर -(c) D. बैंगलौर

261. ई-कािसा कंपनी क्टफ्लपकाटा का उत्तर -288-(c)


।े क्वाटा र भनम्नभलक्टखि िें से भकस जि। ।ै
265. ऑनलाइन प्रापाटी की ीभजंि
A. नोएर्र
करने वाली ई-कािसा कंपनी
B. बैंगलौर
99Acrcs.com के संस्थापक कौन ।ै
C. मुिई
A. विजय र्ेखर र्मरय
D. कोलकरतर
B. सांजीि वबखचांदरनी
उत्तर -(b) C. कुणरल िहल

D. रोवहत बांसल
262. ई-कािसा कंपनी अिेजन की
स्थापना कब हुयी थी? उत्तर -(b)

A. 1994
266. Just Dial कंपनी जो भकस
B. 1980
service या सेवा को बेचने का काया करिी
C. 1983
।ै भनम्नभलक्टखि िें भकस सेििेंट के अंििाि
D. 1991
आिी ।ै
उत्तर -(a) A. B2B Portal

B. C2C Portal
263. भनम्न भलक्टखि िे कौन सी कंपनी
C. B2A Portad
एक कैीन ई-कािसा कंपनी ।ै
D. B2C Portal
A. Maynka
B. Flipkart उत्तर -(d)

C. Amazon
267. टर े वल पैकेज प्रोवाइ करने वाली
D. उिरोक्त मे से कोई नही
ई-कािसा कंपनी यात्रा (Yatra) कंपनी का
उत्तर -(a) ।े ऑभकस क।ां ।ै

A. कलकिर
264. िारि की सबसे बड़ी िोबाईल
B. गुरूग्ररम
िुििान एवं ई-कािसा कंपनी Paytm का
C. बैंगलौर
।े क्वाटा र भनम्न भलक्टखि िें से भकस जि। ।ै
D. उिरोक्त में से कोई नही
A. कलकिर
B. वदल्ली उत्तर -(b)
C. नोएर्र

JOIN OUR TELEGRAM CHANNEL - https://t.me/rakshaacademy


लेखक – अभिषेक भिवारी सूचना एवं संचार प्रौद्योभिकी

268. भनम्नभलक्टखि िें से कौन सी कंपनी उत्तर -(a)


स्टॉक िाकेट िें भलस्टे ट कंपनी ।ै
272. ई-कािसा कंपनी स्नैप ील का
A. Yatra.com
।े ऑभकस भनम्न भलक्टखि िें से भकस जि।
B. Flipkart
।ै
C. Smapd coy
A. वदल्ली
D. Amazon
B. कोलकरतर
उत्तर -(a) C. नोएर्र

D. उिरोक्त में से कोई नहीां


269. ऑनलाइन ।ोटल बुभकंि प्लेटकािा

कंपनी Make my Trip की स्थापना उत्तर -(a)


भनम्नभलक्टखि िे से कब हुई थी
273. e-seva Project भकस राज्स िें
A. 1999
Implement हुआ ।ै
B. 2001
A. अरूणरां चल प्रदे र्
C. 2002
B. ाां ध्रप्रदे र्
D. 2000
C. हररयरणर

उत्तर -(d) D. ररजस्थरन

270. ई-कािसा कंपनी Flipkart के उत्तर -(a)


विािान िें CEO कौन ।ै
274. Khajane Project भकस राज्य िें
A. कल्यरण कष्णमूवतय
Implement हुआ
B. मुकेर्ष बांसल
A. कनरय टक
C. विनीत सक्सेनर
B. ओवर्र्र
D. उिरोक्त मे से कोई नहीां
C. वबहरर

उत्तर -(a) D. िांजरब

271. भनम्नभलक्टखि िें से कौन ई-कािसा उत्तर -(b)


कंपनी Paytm के संस्थापक ।ै
275. NIC NET का कुल कािा -
A. विजय र्ेखर र्मरय
A. National Information Center Network
B. विनीत सक्सेनर
B. National Information Central Network
C. मुकेर्ष बांसल
C. National Integrated Control Network
D. ध्रुि वचांगी।
D. इनमें से कोई नहीां

JOIN OUR TELEGRAM CHANNEL - https://t.me/rakshaacademy


लेखक – अभिषेक भिवारी सूचना एवं संचार प्रौद्योभिकी

उत्तर -(a) उत्तर -(a)

276. सािारण ीब्दों िें नेटवकों का 280. े जी व्हील भप्रटर का प्रकार ।ै


नेटवका क।लािा ।ै A. लेसर

A. इन्टरनेट B. र्रट मैवटर क्स


B. इन्टर रनेट C. मैन्युअल

C. इथरनेट D. इम्पैर
D. उिरोक्त में से कोई नहीां
उत्तर -(b)
उत्तर -(a)
281. वल्डा वाइ बेव एक ।ाइपर

277. सबसे िीिी इन्टरनेट कनेिन सेवा िीभ या भसस्टि ।ै क्योभकं –


कौन सी ।ै – A. हरइिर फरस्ट है

A. र्ी.एस.एल B. दू सरे कम्प्यूटर सांसरधनोां को जोड़तर है


B. र्रयल अि सवियस C. िीवर्यो सरमग्री को केिल प्ररप्त करने में

C. लीज्ड लरइन प्रयोग की जरती है


D. केबल मरर्े म D. उिरोक्त में से कोई नहीां

उत्तर -(b) उत्तर -(b)

278. भकस प्रकार के क्‍प्यूटर का प्रयोि 282. क्‍प्यूटर िे भलंक जानकारी के


सवााभिक भकया जािा ।ै संग्र। को जो इं टरनेट के िाध्यि से

A. सुिर कम्प्यूटर उपलब्ध ।ोिी ।ै ।ै ----- क।ां जािा ।ै


B. वमनी कम्प्यूटर A. बेि सियर

C. मेनफेम कम्प्यूटर B. बेि स्टोर


D. मरइक्ो कम्प्यूटर C. िल्डय िरइर् बेि

D. बेि इन्फवमर्षन
उत्तर - (d)

उत्तर -(c)
279. िारिीय ररिोट सेभसंि संस्थान

क्टस्थि ।ै - 283. ब्लूटूथ िकनीक अनुिभि दे िी ।ै


A. दे हररदू न A. केबल मोबरइल फोन िर सां केत सांचररण

B. कोलकरतर B. लँर्लरइन फोन से मोबरइल फोन के वलये


C. नोएर्र सांप्रेर्षण
D. उिरोक्त में से कोई नहीां C. उिग्रह से टे लीविजन सम्प्रेर्षण

JOIN OUR TELEGRAM CHANNEL - https://t.me/rakshaacademy


लेखक – अभिषेक भिवारी सूचना एवं संचार प्रौद्योभिकी

D. उिकरणो के बीच िरयरलेस सांचररण C. वर्स्टै न्ट ािरे टर सवियस


D. उिरोक्त मे से कोई नही
उत्तर -(a)
उत्तर -(a)
284. भनम्नभलक्टखि िें से कौन सी सूचना
प्रौद्योभिकी पररिाभषकी ।ै – 288. जंक ई िेल को िी क।िें ।ड -

A. सरइबर स्पेस A. स्पूफ


B. अिलोर् B. स्पूल

C. प्रकरर्षीय भण्डररण C. वस्नफर क्तिप्ट


D. मोर्े म D. स्पैम

उिर – साइबर स्पेस उत्तर -(d)

285. भनयिो का सेट क।लािा ।ै – 289. एक भनभिि एब्यूजर ्ारा बार बार
A. प्रोटोकरल्स एक ।ी िेल ई िेल संदेष िेजना क।लािा

B. ार.एफ.सी.एस. ।ै –
C. स्टै ण्डसय A. ई मेल स्पूवफांग

D. इनमें से कोई नही B. ई मेल स्पैवमांग


C. ई मेल बरक्तिग
उत्तर -(a)
D. उिरोक्त मे से कोई नही
286. प्रोटोकालों का व। सिू। जो सिी
उत्तर -(c)
टर ांसभिीन एक्सचेंजेंस को इं टरनेट िें एक
ओर से दू सरी ओर िेजने को पररिाभषि 290. िूिल ने भकस बेब ब्राउजर को

करिा ।ै, क।लािा ।ै – भवकभसि भकया ।ै


A. CSMA/CD A. इन्टरनेट एक्सिोलर

B. TCP/IP B. फरयर फरक्स


C. OFDDI C. सफररी

D. उिरोक्त में से कोई नही D. क्ोम

उत्तर -(b) उत्तर -(d)

287. साइबर ला की ीब्दावली िें 291. भनम्नभलक्टख िे से कौन सा बेब

ी.ओ.एस. का अथा ।ै – ब्राउजर न।ी ।ै –


A. वर्नरइल ाफ सवियस A. ओिेरर

B. वर्स्क ािरे वटां ग वसस्टम B. गूगल एप्स

JOIN OUR TELEGRAM CHANNEL - https://t.me/rakshaacademy


लेखक – अभिषेक भिवारी सूचना एवं संचार प्रौद्योभिकी

C. वििरल्डी C. िरइरल इिोटै र् ररकरर्य यूजर सल्डय


D. मोवजलर फरयरफरक्स D. उिरोक्त में से कोई नही

उत्तर -(b) उत्तर -(b)

292. बेब क्राउलर को इस कप िें िी

जाना जािा ।ै –
A. वलांक र्रयरे ररी

B. सचय ाप्टीमरइजर
C. बेब स्परइर्र

D. बेब मैनेजर

उत्तर -(c)

293. WiMax भनम्नभलक्टखि िें से भकससे

सम्बंभिि ।ै
A. जैि प्रौद्योवगकी
295. संचार नेटवका प्रणाली िें
B. अांतररक्ष प्रौद्योवगकी
कायरवाल का प्रयोि भनम्न िें से भकससे
C. वमसरइल प्रौद्यौवगकी
बचािा ।ै –
D. सांचरर प्रौद्योवगकी
A. अनरवधकत ाक्मण

उत्तर -(d) B. र्रटर वर्रिन ाक्मण


C. अवग्न ाक्मण
294. VIRUS –
D. िरयरस ाक्मण
A. िेरी इां टेलीजेंट ररजल्ट अांवटल सोसय
B. िरइटल इन्फमेर्षन ररसोस अांर्र सीज उत्तर -(a)

JOIN OUR TELEGRAM CHANNEL - https://t.me/rakshaacademy

You might also like